Breast Reconstruction 01-22 Flashcards

1
Q

An otherwise healthy 45-year-old woman undergoes bilateral breast reconstruction with free deep inferior epigastric perforator-based flaps. Only the medial deep inferior epigastric vein was anastomosed to the medial internal mammary vein using a 3.5-mm venous coupler. Her right breast flap has become increasingly congested. The cutaneous Doppler signals are strong, and examination of the pedicle shows good flow through both arterial and venous anastomoses. There is no kinking or hematoma. Which of the following is the most appropriate next step in management?

A) Additional anastomosis of the lateral deep epigastric vein
B) Additional anastomosis of the superficial inferior epigastric vein
C) Revise the arterial anastomosis
D) Revise both anastomoses
E) Revise the venous anastomosis

A

The correct response is Option B.

The most appropriate next step in management is to supercharge the flap using an additional anastomosis of the superficial inferior epigastric vein. This is a clinical example of persistent superficial venous system dominance. This is due to either the superficial system being the dominant venous drainage of the abdominal wall and the absence of connections between the superficial and deep venous systems, or the lack of an adequate number of perforators in the flap. In either case, the venous blood is not getting from the superficial system into the deep system, which is the egress given that the deep system is the only system anastomosed.

There is no technical issue with the anastomosis or venous thrombosis; therefore, revising the deep anastomoses is not required. Rather, it is an intra-flap venous system issue that requires the superficial system to be drained somehow. This is accomplished by anastomosing the superficial inferior epigastric vein to either the deep system in the flap or to another drainage system in the chest like the intercostal system. It is interesting to note that the two reasons for the above issue (i.e., inadequate perforator selection or anatomic lack of communicating vessels between the superficial and deep system), though distinct entities, clinically present similarly and are treated in the same way. The incidence of venous congestion secondary to persistent superficial system dominance is about 0.9%.

How well did you know this?
1
Not at all
2
3
4
5
Perfectly
2
Q

An otherwise healthy 45-year-old woman develops an infection following implant-based breast reconstruction. Which of the following gram-negative species is most likely to be involved with the infection?

A) Enterobacter spp
B) Escherichia coli
C) Proteus spp
D) Pseudomonas spp
E) Staphylococcus spp

A

The correct response is Option D.

The most likely gram-negative species associated with implant-based breast reconstruction infection is Pseudomonas spp.

In general, gram-positive organisms (about 70%) are most commonly associated with such infections, in which Staphylococcus species are the most commonly cultured (51%). Gram-negative species are associated with about 27% of the infections, of which Pseudomonas were the most commonly cultured. These data guide the empiric antibiotics that should be used initially at presentation. Staphylococcus species are the most common bacteria involved with breast implant infections; however, they are gram-positive bacteria.

How well did you know this?
1
Not at all
2
3
4
5
Perfectly
3
Q

An otherwise-healthy 45-year-old woman presents for discussion regarding autologous breast reconstruction following mastectomy. Medical history includes abdominoplasty following the birth of her last child. After examination, the surgeon determines that she is a good candidate for bilateral lumbar artery perforator flaps. During the preoperative discussion, the surgeon compares the risks, benefits, and expected outcomes between lumbar artery perforator flaps and deep inferior epigastric artery perforator (DIEP) flaps, which are the surgeon’s typical initial choice for autologous tissue transfer. Which of the following complications is associated with lumbar artery perforator flaps when compared with DIEP flaps?

A) Higher breast dissatisfaction rate
B) Higher hematoma rate
C) Higher secondary revision rate
D) Higher total flap loss rate

A

The correct response is Option D.

Lumbar artery perforator (LAP) flaps have recently been shown to be a good autologous breast reconstruction option when deep inferior epigastric perforator (DIEP) flaps are not available. LAPs do carry a higher total flap loss rate with reports of 6.6 to 9% compared with less than 2% for DIEPs. Based on the microvascular compromises, which are reported to be more venous than arterial, the take-back rate on LAPs is also expectantly higher than DIEPs. Hematoma and secondary revision rates are very similar between the two groups. Based on BREAST-Q data, the satisfaction rate in the breast reconstruction is also very similar between the two groups. This is not the case in satisfaction rate of the donor site, where responses were lower in LAPs compared with DIEPs. This is likely due to a higher donor site complication rate in the form of seromas and postoperative pain.

How well did you know this?
1
Not at all
2
3
4
5
Perfectly
4
Q

A 56-year-old woman presents with significant capsular contracture and deformity of her left reconstructed breast 3 years after undergoing bilateral mastectomy, left-sided radiation therapy, and subsequent two-stage implant reconstruction. A photograph is shown. She has a history of abdominoplasty. Which of the following is the most reliable way to achieve an acceptable reconstruction in this patient?

A) Conversion to prepectoral plane with acellular dermal allograft
B) Deep inferior epigastric perforator flap
C) Implant exchange with capsulectomy and acellular dermal allograft
D) Implant exchange with capsulotomy
E) Latissimus dorsi flap with expander

A

The correct response is Option E.

In this patient with a history of left-sided radiation and a severely contracted and deformed implant, the most reliable way to achieve an acceptable reconstruction would be to convert to some form of autologous reconstruction. Given her prior history of abdominoplasty, a deep inferior epigastric perforator flap is not feasible. A latissimus dorsi flap with an expander is a reliable option. Implant exchange with capsulotomy would have an extremely high risk for recurrence of capsular contracture and deformity. Capsulectomy with acellular dermal allograft replacement would have a fairly high risk for complications due to adding yet more foreign body to a radiated tissue bed, and there would still be a high risk for recurrent capsular contracture. Conversion to a prepectoral plane with acellular dermal allograft would have a potentially higher risk for complications because even more foreign body is required to fully wrap the implant.

How well did you know this?
1
Not at all
2
3
4
5
Perfectly
5
Q

A 42-year-old woman presents 1 week following exchange of tissue expander for permanent implant, in which skin glue was used for closure. She reports an itchy rash around her incision. Medical history includes right breast cancer, for which she has undergone mastectomy and placement of a pre-pectoral textured tissue expander with acellular dermal allograft coverage. She is afebrile. A photograph is shown. Which of the following is the most likely cause of this rash?

A) Anaplastic large-cell lymphoma infiltration of the skin
B) Antiphospholipid antibody-mediated urticaria
C) Cyanoacrylate exposure and dermatitis
D) Lymphatic disruption leading to red breast syndrome
E) Staphylococcus aureus cellulitis

A

The correct response is Option C.

The patient has recently had surgery and is doing well except for her skin rash. The rash appears to be localized to the incision. Contact dermatitis to surgical adhesives is a common problem. There is growing recognition that surgical glues containing cyanoacrylate may be associated with a 10 to 15% rate of rashes secondary to Type IV allergic hypersensitivity reactions. These typically appear within 3 to 7 days and are treated by removal of the surgical glue and topical steroid cream. Patients who have previously been exposed to surgical glue are more at risk for developing sensitivity.

Surgical site infection is a concern following implant placement. However, a Staph cellulitis is typically more diffuse and has a different appearance more consistent with erythema rather than a rash. Antiphospholipid syndrome (APS) may be associated with a rash and other systemic symptoms; however, there is no indication within the history that this patient suffers from that condition. Additionally, the timing and location of the rash along the incision point to other local causes rather than APS. Lymphatic disruption is one theory to explain red breast syndrome, which is a condition that has been reported in relation to acellular dermal matrix placement. This is typically more diffuse and usually occurs in the acute setting following acellular dermal matrix placement. Anaplastic large-cell lymphoma is a rare type of cancer that may be associated with breast implants and may present with swelling and a rash. There is a higher rate of this condition in patients who have had textured devices. The mean onset for this condition is 8 years after exposure to the breast implant. This patient’s clinical course is not typical of anaplastic large cell lymphoma.

How well did you know this?
1
Not at all
2
3
4
5
Perfectly
6
Q

A 40-year-old woman with cancer of the right breast undergoes a bilateral skin-sparing mastectomy and immediate reconstruction with placement of submuscular tissue expanders. After completion of the bilateral mastectomies, the breast skin flaps appear to be dusky with some bruising of the skin edges. Which of the following pharmacologic agents has been shown to decrease the rate of mastectomy flap necrosis?

A) Indocyanine green
B) Lidocaine with epinephrine
C) Nitroglycerin paste
D) Papaverine

A

The correct response is Option C.

Nitroglycerin paste has been shown to decrease the rate of mastectomy flap necrosis.

Topical nitroglycerin ointment application has been shown in a prospective randomized clinical trial to decrease the incidence of mastectomy flap necrosis in immediate tissue expander breast reconstruction.

Papaverine is incorrect since it is a vasodilator that relaxes smooth muscles in blood vessels. This agent is useful during microvascular surgery to decrease vasospasm of arteries and veins but has not been used to decrease the rates of mastectomy flap necrosis.

Indocyanine green is incorrect since it is a dye used for diagnostic purposes during fluorescence angiography. It can be administered intravenously and be used to assess tissue perfusion intraoperatively, which can be helpful for diagnosis of mastectomy flap necrosis. It does not change tissue perfusion, however.

Lidocaine with epinephrine is incorrect since it is an analgesic agent with a vasoconstrictor. This can be used intraoperatively for pain control and hemostasis. At more dilute concentrations, it can be used for tumescent injection during mastectomy. This has not been shown to decrease the rates of mastectomy flap necrosis.

How well did you know this?
1
Not at all
2
3
4
5
Perfectly
7
Q

A 51-year-old woman with history of right breast invasive ductal carcinoma (IDC) status post right skin-sparing mastectomy, axillary node dissection, and adjuvant radiation therapy presents to discuss delayed breast reconstruction. Her recent positron emission tomography scan was negative for distant metastasis. Her pathology report shows positive estrogen receptor, positive progesterone receptor, negative human epidermal growth factor receptor, and an IDC 2.5 cm in size with three positive axillary lymph nodes. Based on these findings, which of the following is the TNM classification for this tumor?

A) T1 N2 M0
B) T1 N3 M1
C) T2 N1 M0
D) T2 N3 M0
E) T3 N2 M0

A

The correct response is Option C.

The tumor node metastasis (TNM) classification helps to stage tumors. Based on the information given about this breast cancer patient, she would be staged as T2 (primary tumor is 2.5 cm), N1 (three axillary nodes involved), and M0 (no evidence of metastatic disease on systemic radiologic workup. Please see the TNM classification system below.

Tumor staging: Tx - tumor cannot be assessed; T0 - no evidence of primary tumor; T1 - tumor less than 2 cm; T2 - tumor greater than 2 cm but less than 5 cm; T3 - tumor greater than 5 cm; T4 - tumor with direct extension into chest wall or skin.

Node staging: Nx - nodes cannot be assessed; N0 - no nodal involvement; N1 - one to three ipsilateral nodes involved; N2 - four to nine ipsilateral or internal mammary nodes involved; N3 - ten or more ipsilateral nodes.

Metastasis staging: Mx - distant spread cannot be assessed; M0 - no distant spread; M1 - cancer has spread to distant organs or liver.

How well did you know this?
1
Not at all
2
3
4
5
Perfectly
8
Q

A 51-year-old woman presents for breast reconstruction. She was recently diagnosed with left-sided breast implant-associated anaplastic large cell lymphoma (BIA-ALCL). She has bilateral submuscular textured implants that were placed 8 years ago. The patient would also like to have the contralateral implant removed during the left-sided surgery. Which of the following best describes the chance of finding BIA-ALCL pathology in this patient’s contralateral breast?

A) 1.6%
B) 4.6%
C) 7.6%
D) 10.6%
E) 13.6%

A

The correct response is Option B.

The risk for breast implant-associated anaplastic large cell lymphoma (BIA-ALCL) has been estimated to be between 1:1000 and 1:30,000 women with textured breast implants. Management consists of removal of the implant, complete surgical excision of the surrounding implant capsule, and excision of any suspicious lymph nodes. According to National Comprehensive Cancer Network guidelines, removal of the contralateral implant can be considered because in 4.6% of reported cases, the pathology disclosed BIA-ALCL in the contralateral breast.

Once full diagnostic workup and staging are performed, the options for reconstruction need to be discussed with the patient. A patient with resectable disease, stage IA to stage IC, can be offered resection and immediate reconstruction with either a smooth implant or autologous reconstruction, or the patient can choose to have resection followed by repeat imaging in 3 to 6 months and delayed reconstruction.

Stage IA is T1 N0 M0 disease confined to effusion or a layer on the luminal side of the capsule with no lymph node involvement and no distant spread.

Stage IC is T3 N0 M0 cell aggregates or sheets infiltrating the capsule, no lymph node involvement, and no distant spread.

In advanced disease, stages IIA to IV, surgical resection and chemotherapy followed by repeat positron emission tomography CT scan in 6 to 12 months is recommended. If the patient has no signs of metastatic activity, then delayed reconstruction with either a smooth implant or autologous reconstruction are both options.

How well did you know this?
1
Not at all
2
3
4
5
Perfectly
9
Q

A 45-year-old woman presents for bilateral immediate breast reconstruction. An abdominal flap is planned. The patient demands reconstruction with a flap that has the lowest abdominal wall morbidity since she is a yoga and pilates instructor. Which of the following autologous flaps utilized for breast reconstruction results in the lowest rate of abdominal wall morbidity?

A) Deep inferior epigastric perforator flap
B) Free transverse rectus abdominus myocutaneous flap
C) Muscle-sparing free transverse rectus abdominus myocutaneous flap
D) Pedicled transverse rectus abdominus myocutaneous flap
E) Superficial inferior epigastric artery flap

A

The correct response is Option E.

Of all the abdominally based flaps listed for breast reconstruction, only the superficial inferior epigastric artery (SIEA) flap does not violate the abdominal fascia. Each of the other flaps are associated with abdominal bulge or hernia formation. Pedicled and free transverse rectus abdominus myocutaneous (TRAM) flaps both involve the harvest of significant abdominal muscle and are associated with more abdominal wall dysfunction than with deep inferior epigastric perforator (DIEP) flaps. It is unclear whether there is a difference in abdominal wall morbidity following muscle-sparing free TRAMs versus DIEPs. However, only the SIEA flap offers no muscle dissection or violation of the abdominal fascia.

How well did you know this?
1
Not at all
2
3
4
5
Perfectly
10
Q

A 45-year-old woman with a history of breast cancer presents for a discussion of breast reconstruction with silicone breast implants. She inquires about the current screening recommendations for implant rupture. Which of the following postoperative screening recommendations should the surgeon communicate to this patient?

A) CT scan at 3 years followed by CT, MRI, or ultrasound every 2 years thereafter
B) Mammogram every 5 years
C) MRI at 2 years then every 3 years thereafter
D) MRI at 3 years then every 2 years thereafter
E) MRI or ultrasound at 5 to 6 years then every 2 to 3 years thereafter

A

The correct response is Option E.

As a critically important part of the informed consent process, practitioners should be very familiar with updated information from the FDA. The screening recommendations for implant rupture for women with silicone gel-filled implants were recently updated (September 2020). The current FDA recommendations are that asymptomatic patients should have the implants evaluated with either ultrasound or MRI 5 to 6 years after implantation and then every 2 to 3 years thereafter. This new recommendation replaces the old recommendation by adding ultrasound as an accepted modality and extending the time until the first evaluation. The older recommendation was MRI evaluation at 3 years, with follow-up evaluation being performed every 2 years. These recommendations apply to women who have cosmetic augmentation and breast reconstruction. CT scan is not an acceptable modality for implant evaluation. Since this patient has had mastectomies and breast reconstruction, mammography is not indicated.

How well did you know this?
1
Not at all
2
3
4
5
Perfectly
11
Q

Breast cancer cells can spread directly to the cranial cavity and brain via the vertebral venous plexus (Batson plexus). Through which of the following veins is it possible for these cells to reach this plexus?

A) Axillary
B) Azygos
C) Brachiocephalic
D) Intercostal
E) Internal thoracic

A

The correct response is Option D.

Hematogenous spread through the intercostal veins is the most likely way for breast cancer to reach the internal vertebral venous plexus. Although 75% of the lymph from the breast drains to the axillary nodes with most of the remaining lymph draining to the parasternal nodes, these lymphatic channels are not the most likely way to transmit cancer to the internal vertebral venous plexus. This plexus of veins would be most likely to receive cancer cells transmitted through the blood. The internal thoracic vein drains blood from the breast in the opposite direction of the vertebral column. The thoracoacromial artery supplies blood to the breast, as opposed to draining the breast, and therefore would not provide a route for spreading cancer.

How well did you know this?
1
Not at all
2
3
4
5
Perfectly
12
Q

A 56-year-old woman presents with right breast swelling 10 years after undergoing bilateral mastectomy and reconstruction with bilateral submuscular textured implants with an acellular dermal matrix sling. Examination shows right-sided periprosthetic fluid collection. Breast implant-associated anaplastic large cell lymphoma is suspected. Analysis of the fluid will most likely show which of the following tumor markers?

A) Anaplastic lymphoma kinase
B) CD3
C) CD5
D) CD30
E) Human germinal center-associated lymphoma (HGAL)

A

The correct response is Option D.

Breast implant-associated anaplastic large cell lymphoma (BIA-ALCL) has been associated primarily with textured implants. Thought to be related to chronic inflammation from subclinical infection, the presentation is typically with a late-onset seroma. Workup of a late-onset seroma includes aspiration of the seroma and cytological/immunohistochemical analysis. The predictive value is most significant on the initial aspiration because of the presumed larger concentration of malignant cells; as the seroma reforms, there is a hypothesized dilutional aspect.

BIA-ALCL neoplastic cells are strongly CD30-positive, but CD30 is a necessary but insufficient criterion to make the diagnosis since it can be displayed in nonmalignant situations as well (such as in viral infection).

While anaplastic lymphoma kinase (ALK) is associated with systemic ALCL in 60% of cases, it has not been associated with BIA-ALCL. Thus, while ALK assessment is considered necessary to the analysis, it is expected to be negative. BIA-ALCL neoplastic cells variably lose expression of CD3 and CD5. Compared with other forms of ALCL, BIA-ALCL is associated with a more indolent course, although a subset of patients do have an aggressive course. Human germinal center-associated lymphoma (HGAL) is associated with B-cell lymphoma.

How well did you know this?
1
Not at all
2
3
4
5
Perfectly
13
Q

A 62-year-old woman undergoes left mastectomy with axillary lymph node dissection and tissue expander reconstruction. A drain is placed adjacent to the tissue expander and in the axilla. An intravenous catheter is placed in her left arm, and blood pressure is measured from same arm. BMI is 42 kg/m2. Which of the following risk factors is most significantly associated with lymphedema in this patient?

A) Compression from a blood pressure cuff
B) Drain placement in the axilla
C) Obesity
D) Placement of an intravenous catheter
E) Tissue expander reconstruction

A

The correct response is Option C.

Obesity (BMI >30 kg/m2) is the most significant risk factor for lymphedema from the listed choices. Patients who undergo axillary lymphadenectomy and radiation are at an approximately 30% risk for acquiring lymphedema. Obese individuals are at higher risk. In addition, obesity-induced lymphedema is a phenomenon in which patients who reach a BMI of 50 to 60 kg/m2 have a high risk for lymphedema, in particular bilateral lower extremity lymphedema, without any other inciting events.

Compression from a blood pressure cuff is unlikely to cause lymphedema. Lymphedema is primarily treated with compression (pneumatic, static). Evidence is lacking that an intravenous catheter increases the risk for lymphedema. Tissue expander reconstruction or any breast reconstruction does not increase the risk for lymphedema and neither does drainage tube placement.

How well did you know this?
1
Not at all
2
3
4
5
Perfectly
14
Q

A 76-year-old woman presents with swelling of her right reconstructed breast, which was performed with a textured implant 14 years ago. Ultrasound demonstrates an uncomplicated fluid collection. The patient denies pain, fevers, or chills. Enrichment of which of the following cell surface markers within the aspirated fluid is suggestive of breast implant-associated anaplastic large cell lymphoma?

A) CD3
B) CD15
C) CD30
D) CD34
E) CD45

A

The correct response is Option C.

Breast implant-associated anaplastic large cell lymphoma (BIA-ALCL), a type of T-cell lymphoma, is associated with textured implants. Initial diagnosis is often made following ultrasound. CD30 is a cell surface marker detected on T-cells associated with BIA-ALCL. The other cell surface markers are not associated with BIA-ALCL. These cells are also anaplastic lymphoma kinase (ALK) negative. CD3 and CD15 are nonspecific lymphocyte markers not related to BIA-ALCL. CD34 is an endothelial cell marker. CD45 is a marker expressed on all leukocytes.

How well did you know this?
1
Not at all
2
3
4
5
Perfectly
15
Q

A 49-year-old woman who underwent first-stage left breast reconstruction with a tissue expander presents for follow-up to discuss expander to implant exchange. The reconstructed side has more volume and superior fullness compared with the native breast, and the patient prefers the reconstructed side. A photograph is shown. In addition to implant exchange, which of the following would most likely give the patient the greatest satisfaction regarding the appearance of her breasts?

A) Left breast fat grafting
B) Left breast flap reconstruction
C) Right breast implant augmentation
D) Right breast mastopexy
E) No additional procedures

A

The correct response is Option C.

The patient shown in the initial photograph complains of lack of symmetry between the reconstructed breast and the native, contralateral side. The volume and superior fullness in the reconstructed breast are a result of an implant-based reconstruction. Studies have shown improved patient satisfaction with breast reconstruction when contralateral augmentation is performed for symmetry. For this patient who was happy with an implant-based reconstruction, symmetry was best achieved with a contralateral breast augmentation. In the additional image shown, she is pictured after tissue expander to implant exchange, nipple-areola complex reconstruction, and right breast augmentation. The other choices are less ideal and would not necessarily give the patient the symmetry or satisfaction she desires.

How well did you know this?
1
Not at all
2
3
4
5
Perfectly
16
Q

A 47-year-old woman comes to the emergency department because of wound separation. History includes immediate prepectoral breast reconstruction with placement of left tissue expander, and she is currently undergoing radiation therapy. Physical examination shows an approximately 0.5-cm area of incisional dehiscence with exposure of the device. Vital signs are stable, and inflammatory markers are mildly elevated. Which of the following is the most appropriate next step in management?

A) Admit the patient for intravenous antibiotic therapy and monitor the wound
B) Apply occlusive dressing and discharge the patient on oral antibiotic therapy
C ) Close the wound at bedside in the emergency department and discharge the patient on oral antibiotic therapy
D) Perform expander removal in the operating room
E) Perform operative replacement of the expander

A

The correct response is Option D.

Studies have shown that postmastectomy radiation therapy almost doubles the risk of complications, including wound breakdown. While attempts at implant salvage are tempting, they are often ultimately unsuccessful and may potentially delay necessary radiation treatments. Therefore, in the event of implant exposure during adjuvant therapy, the device should be removed and the mastectomy pocket closed over a surgical drain. This strategy will allow for the shortest delay in further oncologic treatments and does not preclude reconstruction at a later date. Closure of the wound over an exposed device should not be attempted, as any exposed device is assumed to be colonized, if not grossly infected. Although antibiotics may temporize the infectious process, it is unlikely that successful wound healing and retention of the prosthesis will occur after suppressive antimicrobials are stopped.

How well did you know this?
1
Not at all
2
3
4
5
Perfectly
17
Q

An otherwise healthy, 45-year-old woman presents for breast reconstruction. She underwent a mastectomy 1 year ago with no immediate reconstruction followed by post-mastectomy radiation therapy. She does not want anything performed to the contralateral breast and does not want to have an implant. She has a history of an abdominoplasty. The plastic surgeon plans to perform stacked profunda artery perforator flaps for the unilateral reconstruction. Which of the following is the most appropriate option for the recipient vessels?

A) Ipsilateral and contralateral antegrade internal mammary vessels
B) Ipsilateral antegrade and retrograde internal mammary vessels
C) Ipsilateral antegrade and retrograde thoracodorsal vessels
D) Ipsilateral antegrade internal mammary and thoracoacromial vessels
E) Ipsilateral antegrade thoracodorsal and thoracoacromial vessels

A

The correct response is Option B.

It has been demonstrated that the retrograde intermammary vessels are a viable and reliable choice for stacked flap reconstruction. The benefit of using these as the recipient vessels is that it does not add another recipient site to the operation.

Ipsilateral and contralateral antegrade internal mammary vessels are incorrect because the patient does not want to have any procedures performed on her contralateral breast, although this has been described as an option for autologous breast reconstruction.

The thoracodorsal vessels, prior to development of the internal mammary vessels as the preferred option for autologous breast reconstruction, were commonly used for recipients, but utilizing these can negate the option of a latissimus dorsi as a salvage procedure in the setting of flap failure, especially if the plastic surgeon were to use the antegrade and retrograde vessels. Ipsilateral antegrade internal mammary and thoracodorsal vessels would be correct as well, but it was not given as an option. Thoracoacromial vessels are not typically used in autologous breast reconstruction, aside from being a lifeboat for venous outflow.

How well did you know this?
1
Not at all
2
3
4
5
Perfectly
18
Q

Which of the following legislative acts mandates insurance coverage of breast reconstruction after total mastectomy as well as coverage of any associated symmetry procedures for the contralateral breast?

A) Affordable Care Act
B) Breast Cancer Patient Education Act
C) SB-255 Amendment to the Knox-Keene Health Care Service Plan Act
D) Women’s Health and Cancer Rights Act

A

The correct response is Option D.

The Women’s Health and Cancer Rights Act of 1998 (WHCRA) is a federal law that mandates the coverage of breast reconstruction after mastectomy as well as coverage of any associated symmetry procedures for the contralateral breast. Although this federal law was enacted more than 20 years ago, there are still significant disparities in access to breast reconstruction and a lack of education regarding the options available for breast reconstruction.

The Breast Cancer Patient Education Act of 2015 is a federal law that requires the Secretary of Health and Human Services to implement an educational campaign to inform breast cancer patients about access, availability, and options for breast reconstruction after mastectomy. SB-255 is an amendment in the state of California to the Knox-Keene Health Care Service Plan Act that includes “lumpectomy” for treatment of breast cancer in the definition of “mastectomy” and mandates access to insurance coverage of breast reconstruction after lumpectomy in the state of California. It is important to note that the WHCRA only mandates coverage of breast reconstruction after mastectomy, not after breast conservation therapies such as lumpectomy. In fact, there is no current legislation mandating insurance coverage for all types of breast reconstruction, including breast reduction, mastopexy, or implant complications after aesthetic surgeries.

The Affordable Care Act (ACA) is a comprehensive health care reform law enacted in March of 2010. The three primary goals of the ACA are to make affordable health insurance available to more people, to expand the Medicaid program to cover all adults with income below 138% of the federal poverty level, and to support innovative medical care delivery methods designed to lower the costs of health care generally. There is nothing specifically in reference to breast cancer or breast reconstruction care within the ACA.

How well did you know this?
1
Not at all
2
3
4
5
Perfectly
19
Q

A 56-year-old woman with breast cancer undergoes bilateral mastectomy and immediate breast reconstruction with deep inferior epigastric perforator (DIEP) flaps. She has an uneventful recovery and is discharged home on postoperative day 4. Which of the following best describes the expected postoperative changes to the lower extremity venous system during this patient’s hospital stay?

A) There is decrease in diameter of the common femoral vein
B) There is increase in flow velocity of the common femoral vein
C) There is no change to the venous circulation
D) There is persistent venous stasis through the day of discharge
E) There is venous stasis, influenced by unilateral versus bilateral flap reconstruction

A

The correct response is Option D.

There is an increased risk of deep venous thrombosis associated with autologous breast reconstruction. Studies have tried to correlate autologous breast reconstruction with decreased venous return and stasis.

When compared with baseline levels, the common femoral veins have increased diameter and decreased flow velocity. These changes persist through the day of discharge. Since lower abdominal tissue is harvested and tight abdominal closure is performed in both unilateral and bilateral autologous breast reconstruction, there is no difference in venous stasis postoperatively when comparing both groups.

How well did you know this?
1
Not at all
2
3
4
5
Perfectly
20
Q

A 43-year-old woman with a history of Stage I T1N0 invasive ductal carcinoma of the right breast presents 15 years after undergoing bilateral mastectomy, negative right sentinel lymph node biopsy, and staged expander to implant breast reconstruction. Over the past 3 months, her left reconstructed breast has become significantly larger than her right. On examination, there are no skin lesions, palpable masses, or axillary lymphadenopathy. Which of the following is the most appropriate next step in the workup of this patient?

A) Ultrasound
B) Mammography
C) PET-CT
D) MRI

A

The correct response is Option A.

The patient presents 15 years after bilateral mastectomy and staged expander to implant breast reconstruction with a new finding of left reconstructed breast enlargement. This may be due to a large late periprosthetic fluid collection, Baker Classification Grade III and IV capsular contracture can give the impression of breast enlargement because of the increased projection and vertical height of the breast contour. Any patient presenting with a late periprosthetic fluid collection should be evaluated for breast implant-associated anaplastic large cell lymphoma (BIA-ALCL). In this patient, it is unknown what type of implant she has; therefore, radiologic evaluation is warranted.

Physical examination and a breast ultrasound would distinguish between these two causes of perceived enlargement of a breast after implant-based breast reconstruction. Ultrasound can also identify capsular masses and provides an opportunity to aspirate the fluid collection, which often provides symptomatic relief for the patient, as well as an opportunity to perform cytology to analyze lymphocyte numbers and characteristics, in addition to flow cytometry to measure anaplastic lymphoma kinase negativity and CD30 cell surface receptor positivity.

Presentation with BIA-ALCL can take many years to develop, with a median 10 years to fluid collection development. Surgical treatment for BIA-ALCL can vary with the stage of presentation, but BIA-ALCL confined to the periprosthetic fluid can be effectively treated with capsulectomy and implant removal. The majority of cases of BIA-ALCL present with late-onset seroma (66-80%), whereas the second most common presentation is an isolated mass within the capsule around the implant.

PET-CT is often performed as part of a staging evaluation after the diagnosis of BIA-ALCL is confirmed. Mammography is a breast cancer screening tool, but has no role in post-mastectomy evaluation.

A prone breast MRI (or ultrasound) is recommended 5 to 6 years after silicone breast implant placement and every 2 years thereafter to evaluate the integrity of the implant shell, but has no role in the management of BIA-ALCL.

How well did you know this?
1
Not at all
2
3
4
5
Perfectly
21
Q

An otherwise healthy 41-year-old woman is scheduled to have bilateral prophylactic NSM for a BRCA1 mutation. Which of the following is an advantage of prepectoral implant reconstruction when compared with submuscular implant reconstruction?

A) There are significantly more acute postoperative surgical complications
B) There is a higher revision rate
C) There is a lower rate of animation deformity
D) There is a lower rate of upper pole rippling and implant palpability
E) There is more pain and functional impairment

A

The correct response is Option C.

The patient is a young, healthy, thin woman presenting for bilateral breast reconstruction following prophylactic mastectomies because of increased lifetime risk of breast cancer. She will maintain her entire breast skin envelope because she has planned nipple-sparing mastectomies and does not wish to increase her breast size with the operation. She has questions about the plane of implant placement above or below the pectoralis muscle and the answer choices provide the opportunity to counsel her on direct to implant versus staged expander to implant breast reconstruction.

Because of the lack of camouflage afforded by placement of the pectoralis muscle over the upper pole of the breast implant, prepectoral breast reconstruction is associated with increased rippling of the upper portion of the breast. This patient is thin according to BMI and so will have an increased risk of upper pole rippling that can be improved with subsequent fat grafting, provided she has adequate donor sites for fat harvest.

Several patient series reporting on early outcomes after prepectoral breast reconstruction have used a partially submuscular patient cohort as a comparison for acute postoperative complications. The overwhelming majority of these series showed no significant differences in acute postoperative outcomes and revision rates between techniques.

Postoperative functional assessments have demonstrated less pain and earlier return of function after prepectoral implant placement compared to submuscular implant placement. Re-siting of submuscular implants to a prepectoral plane have demonstrated resolution of animation deformity.

Algorithms for patient selection have slowly evolved over the past five years with the only constant recommendation being that the ideal candidate has a BMI less than 30 kg/m2. Earlier series emphasized mastectomy skin flap thickness, whereas more recent studies focus on skin flap viability. Earlier studies considered radiation treatment a contraindication, whereas more recent reports demonstrate the negative impact of radiation on pectoralis major fibrosis in submuscular reconstruction as compared with prepectoral implant placement.

How well did you know this?
1
Not at all
2
3
4
5
Perfectly
22
Q

Lactiferous ducts in the human embryo are derived from which of the following types of cell populations?

A) Endodermal
B) Epithelial
C) Mesenchymal
D) Neural crest
E) Pluripotential

A

The correct response is Option B.

As early as 6 weeks of gestation, discrete areas of epithelial proliferation occur on the ventral thorax. These areas evolve into buds that then canalize and form secondary buds that give rise to lactiferous ducts. These, along with secretory acini that originate in puberty, constitute the parenchyma of the breasts. These ducts are surrounded by mesodermal-derived mesenchymal cells, which in turn develop into the breast stroma composed of smooth muscle cells, capillary endothelial cells, and adipocytes. Neural crest cells give rise to pigmented cells and other structures but are not involved with embryogenesis of the breasts. Endodermal cell lines produce the lining of the gut and contribute to development of the liver and pancreas but not the breasts. Pluripotential cells do not comprise the parenchyma nor the stroma of the developing breast.

How well did you know this?
1
Not at all
2
3
4
5
Perfectly
23
Q

Which of the following hormones in the mammary gland is critical for breast development during puberty?

A) Cortisol
B) Insulin-like growth factor-1 (IGF-1)
C) Platelet-derived growth factor (PDGF)
D) Prolactin
E) Vascular endothelial growth factor-C (VEGF-C)

A

The correct response is Option B.

While pubertal breast development depends on an estrogen surge, it is important to remember that breast development occurs downstream. The surge triggers pituitary growth hormone to stimulate mammary gland production of insulin-like growth factor-1 (IGF-1). IGF-1, in turn, causes proliferation and anti-apoptosis through a signaling cascade. Disruption of any factor along this pathway will result in abnormal breast development. Laron syndrome, for example, is an autosomal recessive disorder in which growth hormone insensitivity can cause delayed puberty as well as short stature. While high levels of prolactin can cause breast growth (and are thought to be responsible for ultimate breast development in Laron syndrome), prolactin is not in the mammary glands themselves.

In addition to the critical role IGF-1 plays during breast development, it also plays an important role in breast development during pregnancy. High levels of IGF-1 are also associated with increased breast volume and early-onset breast cancer. High serum levels of IGF-1 are associated with both bad prognosis and increased mortality in breast cancer. Cortisol is a stress hormone not related to breast development. Platelet-derived growth factor has a primary role in blood vessel development, as does vascular endothelial growth factor (VEGF). VEGF-C has a function in lymphangiogenesis.

How well did you know this?
1
Not at all
2
3
4
5
Perfectly
24
Q

A 55-year-old woman has ductal carcinoma in situ located approximately 6 cm from the nipple on the lateral upper quadrant of the left breast. Which of the following is the minimum margin of excision if she decides to proceed with lumpectomy followed by radiation therapy?

A) 1 mm
B) 2 mm
C) 5 mm
D) 1 cm
E) 2 cm

A

The correct response is Option B.

Consensus guidelines recommend a 2-mm minimum margin for patients undergoing breast-conserving therapy. Margins over 2 mm are not associated with further risk of local recurrence.

Primary treatment options for women with ductal carcinoma in situ (DCIS) are lumpectomy plus whole breast radiation with or without boost, total mastectomy - with or without SLNB with optional reconstruction, or lumpectomy alone. For patients with DCIS treated with breast conserving therapy, which includes lumpectomy followed by radiation, margins of at least 2 mm are associated with a reduced risk of ipsilateral breast tumor recurrence relative to narrower negative margin widths. However, a negative margin width wider than 2 mm is not supported by the evidence. An analysis of specimen margins and specimen radiographs should be performed to ensure that all mammographically detectable DCIS has been excised. In addition, a postexcision mammogram should be considered where appropriate.

How well did you know this?
1
Not at all
2
3
4
5
Perfectly
25
Q

A 54-year-old woman presents for follow-up examination after undergoing left mastopexy for improved symmetry 6 months ago. Medical history includes breast cancer. The patient reports some discomfort overlying a firm 1.5-cm mass that is palpable in the subcutaneous plane of the left breast. Which of the following is the most appropriate next step in management?

A) Fine-needle aspiration
B) Surgical excision
C) Ultrasound imaging
D) Warm compresses
E) Observation only

A

The correct response is Option C.

The most appropriate next step in management is to obtain imaging. Ultrasound can identify and characterize lesions such as fat necrosis, which is the most likely cause of this patient’s palpable mass in the postoperative period. Such imaging can confirm the physical examination findings and guide the best course of treatment (ie, aspiration versus excision). Observation or warm compresses are inadequate in addressing these findings or in obtaining a diagnosis. Once the benign nature and characteristics of the lesion are better understood, then treatment with bedside aspiration or formal excision of the mass can be undertaken.

How well did you know this?
1
Not at all
2
3
4
5
Perfectly
26
Q

According to the 2017 US breast cancer mortality data, three states with appropriately powered data achieved statistically equivalent mortality rates between non-Hispanic African American and non-Hispanic Caucasian women. Which of the following factors was most likely implicated in the improvement in mortality rates in the non-Hispanic African American women in these states?

A) Non-Hispanic African American women are more affluent
B) Non-Hispanic African American women are younger
C) Non-Hispanic African American women have fewer “triple-negative” aggressive breast cancers
D) Non-Hispanic African American women have more access to screening mammography and in situ diagnosis

A

The correct response is Option D.

Social determinants of health include where a person resides, what exposures are present, what diet is eaten, how much stress is present, and other lifetime experiences. Social determinants of health affect who gets disease and how well they can be cared for when they have disease.

According to the 2020 cancer statistics, death from female breast cancer has declined overall by 40% in the United States since 1989.

The 2017 breast cancer statistics look at breast cancer in a state-by-state fashion demonstrating that those gains in survival have not been uniformly spread among the populations, with non-Hispanic African American women suffering higher death rates by age group, staging, and oncotyping groups.

While there are no data to discuss why non-Hispanic African American women tend to develop cancer younger or develop more “triple negative” breast cancers, there has been evidence presented in the 2017 breast cancer statistics that better access to care may improve the stage at diagnosis. In seven states, the mortality rates were statistically the same. In three of those states—Massachusetts (an early state to establish mandatory health insurance), Connecticut, and Delaware—there were sufficient cohort numbers of non-Hispanic African American women with an in situ diagnosis, which is used as a proxy for access to screening mammography. In the four other states, there was statistical equivalence, but they were underpowered. The implication of these data is that better access to mammography/care lowers mortality for non-Hispanic African American women with breast cancer. These studies did not address affluence, but they did rule out age as a factor.

How well did you know this?
1
Not at all
2
3
4
5
Perfectly
27
Q

A 56-year-old woman is evaluated 6 hours after undergoing bilateral breast reconstruction with a deep inferior epigastric perforator (DIEP) flap. Doppler examination shows strong arterial signals in both flaps. The right breast appears bluish with a capillary refill time of 1 second compared to 3 seconds on the left side. Which of the following is the most appropriate next step?

A) Administration of tissue plasminogen activator
B) Application of leeches
C) Application of nitroglycerin ointment
D) Exploration in the operating room
E) Observation

A

The correct response is Option D.

The patient described has evidence of venous congestion. The reported incidence of venous congestion in free tissue breast reconstruction ranges from 2 to 20%. Causes include venous thrombosis, inadequate perforator selection, and superficial venous system dominance with lack of sufficient communication to the deep system. Signs of venous compromise include the following: cyanotic/blue color, brisker than normal capillary refill, increased tissue turgor, cooler temperature compared to normal skin (greater than 2 degrees), rapid bleeding of dark blood with pinprick, and absence of continuous venous Doppler signal. The most appropriate course of action in this scenario is emergent exploration in the operating room to assess the vascular pedicle for thrombosis, compression from hematoma, kinking, or superficial system dominance. Flap salvage rate is directly tied to timing of exploration, with higher salvage rates in flaps explored within 6 hours of identification of compromise.

Early recognition and rapid exploration of compromised flaps are the most important factors predicting flap salvage, so observation would be unacceptable. Tissue plasminogen activator is useful if diffuse clotting is suspected within the flap, but should only be given locally within the flap. Leeches can be a useful adjunct postoperatively after employing the other maneuvers described above, but would not resolve the underlying problem in this case. Application of topical nitroglycerin can improve venous congestion in random skin flaps, but has no role in the management of acute microvascular thrombosis.

How well did you know this?
1
Not at all
2
3
4
5
Perfectly
28
Q

An otherwise healthy 54-year-old perimenopausal woman is scheduled for a mastectomy for biopsy-proven right-sided grade 2 ductal carcinoma. According to the National Comprehensive Cancer Network (NCCN) guidelines, postmastectomy radiation therapy will be the standard of care for this patient if she has which of the following surgical outcomes?

A) 1-cm surgical margins, four positive axillary lymph nodes
B) 1-cm surgical margins, one positive axillary sentinel node
C) 1-mm surgical margins, no positive axillary nodes
D) 5-mm surgical margins, no positive axillary nodes
E) 5-mm surgical margins, three positive axillary nodes

A

The correct response is Option A.

Traditionally, the need for radiation therapy has been a contraindication for implant-based reconstruction, and autologous reconstruction is the conservative gold standard for women with advanced cancer needing postmastectomy radiation. More recently, there have been reports of successful implant based reconstruction in the setting of postmastectomy radiation that have similar complication profiles and good oncologic outcomes compared with autologous reconstruction. Protocols vary between those that radiate the expander and then expand, and those that expand and then radiate the permanent implant. Being able to anticipate which patient will require postmastectomy radiation is essential for joint decision making about breast reconstruction with the patient prior to her mastectomy.

By National Comprehensive Cancer Network (NCCN) guidelines, relative indications for postmastectomy radiation therapy include: positive sentinel node with unknown status of other axillary nodes, one to three positive nodes on permanent histology, and close surgical margins (less than 5 mm). Postmastectomy radiation is recommended as the standard of care in the situations of positive surgical margins with the inability to get clear margins and four or more positive lymph nodes.

How well did you know this?
1
Not at all
2
3
4
5
Perfectly
29
Q

A healthy 45-year-old woman with a history of breast malignancy underwent bilateral mastectomy and reconstruction with tissue expanders followed by exchange for cohesive silicone gel implants eight years ago with routine postoperative MRI surveillance. She comes to the office to report pain and tightness in the right breast that has gradually increased over the past month. On examination, temperature is 36.8°C (98.2°F), blood pressure is 112/76 mmHg, and heart rate is 68 bpm. The right breast appears fuller than the left breast; otherwise, the right implant is in a symmetric position with the left side. The skin is otherwise normal in appearance, and there is no tenderness on palpation. Which of the following is the most appropriate next step in management?

A) MRI of the right breast to assess the integrity of the implant
B) One week of an oral antibiotic and prednisone taper
C) Operative exploration, culture, and replacement of implant
D) Referral of the patient back to her medical and surgical oncologists
E) Ultrasound of the right breast and fine-needle aspiration of any fluid

A

The correct response is Option E.

Patients that present with a late seroma should be evaluated for possible Breast implant associated Anaplastic Large Cell Lymphoma (BI-ALCL). A late seroma is usually accepted as occurring 1 year following surgery: however there are cases of BI-ALCL seroma that have presented as early as 4 months.

The first step in evaluation for BI-ALCL is an ultrasound followed by fine needle aspiration is indicated. The fluid requires evaluation beyond routine cell cytology. Immunohistochemistry test for CD30 was the most commonly positive marker for BI-ALCL. Immunohistochemistry stains specific antigens in cells by binding to this antigen in an antibody/antigen reaction. The specific stain can then be seen under light microscopy. CD30 antibody labels anaplastic large cell lymphoma cells. CD30 is a transmembrane cytokine receptor belonging to the tumor necrosis factor receptor family and characteristically stains ALCL cells.

MRI for implant integrity and referral to her Oncologist may be needed but it is not the most appropriate next step. BIA-ALCL needs to be ruled out. Immediate operative exploration is not indicated before fluid aspiration and immunohistochemistry evaluation. Antibiotics and prednisone is not indicated in this patient without evidence of infection or inflammation (red breast syndrome).

How well did you know this?
1
Not at all
2
3
4
5
Perfectly
30
Q

In women undergoing prosthetic breast reconstruction complicated by an expander/implant infection, which of the following is the most common gram-negative bacteria isolated from cultures?

A) Escherichia coli
B) Klebsiella
C) Proteus
D) Pseudomonas
E) Serratia

A

The correct response is Option D.

Tissue expander/implant-based breast reconstruction remains the most common form of reconstruction after mastectomy. One of the most potentially devastating complications of this form of breast reconstruction is an implant infection with need for removal of the expander/implant. The mean reported incidence of implant infection after breast reconstruction is 8%, with a range of 1 to 35%. When cultures are obtained, the most common causative bacteria on microbiology examination are gram-positive organisms (41 to 83%), specifically, Staphylococcus species (56 to 76.5%). Gram-negative bacteria accounted for 15.3 to 28.6%, with Pseudomonas (10.7 to 14%) being the most common gram-negative bacteria present on microbiology examination.

How well did you know this?
1
Not at all
2
3
4
5
Perfectly
31
Q

A patient with a history of breast cancer undergoes nipple-sparing mastectomy of the right breast with immediate implant-based reconstruction. Ten months after surgery, the patient starts to recover sensitivity at the nipple. Which of the following nerves is most likely providing sensitivity to the nipple-areola complex in this patient?

A) Anterior branch of the fourth intercostal nerve
B) Anterior branch of the second intercostal nerve
C) Lateral branch of the fifth intercostal nerve
D) Lateral branch of the fourth intercostal nerve
E) Lateral branch of the second intercostal nerve
F) Lateral branch of the third intercostal nerve

A

The correct response is Option A.

The nipple and areola of the breast are innervated by both the anterior and lateral cutaneous branches of the third, fourth, or fifth intercostal nerves. The anterior and lateral cutaneous branches of the second and sixth intercostal nerves innervate breast skin only.

In anatomical studies conducted in female cadavers, the fourth intercostal nerve’s lateral cutaneous branch supplied the nipples in 93% dissected breasts. The third and fifth intercostal lateral branches were found to innervate the nipple alone in 3.6%. However, the fourth intercostal lateral branch penetrates the deep fascia in the midaxillary line, takes an inferomedial course to reach the midclavicular line, and continues through the glandular tissue towards the posterior surface of the nipple. Thus, when a mastectomy is performed, this lateral branch is the most likely one to be dissected.

On the other hand, the anterior cutaneous branches take a superficial course, as they run in the subcutaneous tissue close to the skin and reach the nipple from the lateral side. According to this, the anterior branch of the fourth intercostal nerve is most likely providing sensitivity to the nipple-areolar complex after nipple-sparing mastectomy.

How well did you know this?
1
Not at all
2
3
4
5
Perfectly
32
Q

A 53-year-old woman is evaluated for left-sided nipple reconstruction after mastectomy. She has scars on the left breast from a previous breast biopsy, as well as from the mastectomy itself. Nipple reconstruction must be designed around the scars. In single-pedicle nipple reconstruction, which of the following provides the blood supply to the pedicle?

A) Internal mammary artery perforators
B) Posterior intercostal arteries
C) Subdermal plexus
D) Superior intercostal artery
E) Thoracoacromial artery perforators

A

The correct response is Option C.

Single-pedicle nipple reconstructions, which include such techniques as the skate flap, star flap, C-V flap, and opposing tab flaps as well as other variations, create nipples from remaining mastectomy skin through adjacent tissue transfer. The flap derives its blood supply from the subdermal plexus.

The creation of the flap must keep this blood supply in mind. The flap design must avoid previous scars at the flap base and must integrate the subcutaneous fat at the base of the pedicle.

The internal mammary artery supplies the breast itself and the nipple-areola complex, and the thoracoacromial artery supplies the pectoralis muscle and the breast. The posterior intercostal arteries supply the intercostal spaces. The superior intercostal artery arises from the costocervical trunk, off of the subclavian artery, and supplies the intercostal spaces.

How well did you know this?
1
Not at all
2
3
4
5
Perfectly
33
Q

A 41-year-old woman presents with right breast lobular carcinoma in situ (LCIS) involving a 1-cm area with no palpable axillary nodes. According to the TNM staging system, which of the following is this patient’s T classification?

A) Tx
B) Tis
C) T0
D) T1a
E) None; there is no TNM staging for LCIS

A

The correct response is Option E.

Lobular carcinoma in situ (LCIS) has been removed from the staging classification system in the 8th edition and is no longer included in the pathologic tumor in situ (pTis) category. LCIS is treated as a benign entity with an associated risk for developing carcinoma in the future but not as a malignancy capable of metastases. There is a small subset of LCIS that has high-grade nuclear features and may exhibit central necrosis. This subset has been referred to as pleomorphic LCIS and has histologic features that partially overlap the features of ductal carcinoma in situ (DCIS), including the potential to develop calcifications detectable by mammography. The expert panel debated whether to include this variant of LCIS in the pTis category; however, there are insufficient data in the literature regarding outcomes and reproducible diagnostic criteria for this LCIS variant. Cases exhibiting DCIS and LCIS are classified as pTis (DCIS).

How well did you know this?
1
Not at all
2
3
4
5
Perfectly
34
Q

A 44-year-old woman presents in evaluation for breast reconstruction with biopsy-proven left breast-infiltrating ductal carcinoma after routine mammography discovered a 7-cm lesion. She has been referred to medical oncology and genetic testing is pending. Her past medical history is significant for hypertension and scleroderma. On examination, she has grade I ptosis and wears a size 34A brassiere. During the consultation, the patient reports a strong preference for lumpectomy and oncoplastic reconstruction over total mastectomy. Which of the following is most likely to increase this patient’s chances of qualifying for breast-conserving therapy?

A) Active scleroderma
B) BRCA-1 gene mutation
C) Multicentric tumor
D) Preoperative chemotherapy
E) Small-sized breasts

A

The correct response is Option D.

Preoperative chemotherapy could increase this patient’s chances of qualifying for locoregional treatment (partial mastectomy or lumpectomy). Studies have shown that breast conservation rates are improved with preoperative systemic therapy, which can also render inoperable tumors resectable. Other potential benefits of preoperative (neoadjuvant) chemotherapy include providing important prognostic information based on response to therapy, minimizing the extent of axillary surgery, and allowing time for genetic testing and reconstructive planning prior to surgery. A small-sized breast would likely provide insufficient uninvolved breast tissue for breast-conserving therapy after resection of a large (7 cm) mass. The same applies to multicentric tumors.

Whole breast irradiation is strongly recommended after lumpectomy, with studies showing a favorable effect in reducing the 10-year risk of recurrence (19% versus 35%) and the 15-year risk of breast cancer death (21% versus 25%). Therefore, patients with (relative) contraindications to radiation therapy, such as lupus or scleroderma (connective tissue disease involving the skin), should ordinarily be offered total mastectomy, particularly if this resolves the need for radiation therapy. While radiation therapy would likely still be considered for this particular patient even after total mastectomy (tumor size greater than 5 cm), the diagnosis of scleroderma itself does not increase her chances of qualifying for breast conservation surgery. BRCA-1 gene mutation and other genetic predispositions to breast cancer are relative contraindications for breast-conserving therapy. These patients may be considered for prophylactic bilateral mastectomy for risk reduction.

How well did you know this?
1
Not at all
2
3
4
5
Perfectly
35
Q

A 54-year-old woman is evaluated for nipple-areola complex reconstruction after mastectomy and silicone implant-based reconstruction. During discussion of the risks and benefits of a C-V flap, the patient asks about the long-term results of different techniques. Which of the following is the most likely long-term complication of a single-pedicle nipple-areola reconstruction?

A) Atrophic scarring
B) Delayed nipple necrosis
C) Hypertrophic scarring
D) Implant exposure
E) Loss of projection

A

The correct response is Option E.

Single-pedicle nipple reconstructions, which include techniques such as the skate flap, star flap, C-V flap, and opposing tab flaps as well as other variations, create nipples from remaining mastectomy skin through adjacent tissue transfer. The flaps derive their random-pattern blood supply from the subdermal plexus.

The creation of the flap must keep this blood supply in mind. The surgical technique must avoid previous scars at the base of the flap design and must integrate the subcutaneous fat at the base of the pedicle.

While hypertrophic and atrophic scarring can occur, they are not the most common long-term effects, and are more a function of patient characteristics than flap characteristics.

Implant exposure can occur with scar breakdown, but this is an early rather than a late complication.

Delayed nipple necrosis is technically not correct because the nipple is no longer present, and is not correct of the nipple reconstruction because necrosis of the flaps, if it occurs, usually occurs early.

How well did you know this?
1
Not at all
2
3
4
5
Perfectly
36
Q

A 45-year-old woman presents with right breast cancer and is planning a nipple-sparing mastectomy and tissue expander placement. She is specifically interested in a carbon dioxide–based expander. Which of the following is a disadvantage of this device compared with a saline tissue expander?

A) Extrusion
B) Inability to deflate
C) Increase in wound dehiscence
D) Increase in wound infection
E) Possible device dislocation

A

The correct response is Option B.

The carbon dioxide-based tissue expander (AeroForm) is a fixed-volume device and has an inability to deflate the expander.

In a prospective, multicenter, randomized controlled trial comparing carbon dioxide–based expanders and saline tissue expanders, there were no statistically significant differences in rates of wound infection, extrusion, device dislocation, or wound dehiscence. Advantages of the carbon dioxide–based expander include a more rapid expansion process and a shorter time to implant exchange. The device is self-contained and patient-controlled, so there are no needles required and possibly fewer physician office visits.

How well did you know this?
1
Not at all
2
3
4
5
Perfectly
37
Q

A 55-year-old woman with a BRCA gene mutation elects to undergo bilateral mastectomy with reconstruction using a deep inferior epigastric perforator flap. BMI is 41 kg/m2. Physical examination shows both supra- and infraumbilical adiposity with excess skin and a mature cesarean delivery scar. This patient has the highest risk for which of the following early postoperative complications?

A) Abdominal wall bulge
B) Abdominal wall hernia
C) Delayed wound healing of donor site
D) Fat necrosis of the flap
E) Flap failure

A

The correct response is Option C.

The highest risk is for delayed wound healing of the donor site. Because of the patient’s morbid obesity and prior cesarean delivery scar, she has the highest risk for some form of wound breakdown or prolonged wound healing. This risk can be as high as 50 to 60% for morbidly obese patients. These trends are similar in patients following reconstruction with a transverse rectus abdominis musculocutaneous (TRAM) flap.

To reduce these risks, minimal undermining is recommended and only done if necessary. Techniques to preserve all cutaneous perforators will help reduce the risk associated with closure of the donor sites.

While morbid obesity can be associated with increased abdominal wall thickness, there is no correlation with the occurrence of abdominal wall bulge or hernia. The risks for these complications are less than 2%.

Patients with morbid obesity can have shorter operative times, but there is no correlation with overall flap failure, with rates reported to be less than 1%. This is also seen in pedicled and free TRAM flap reconstructions.

Rates of fat necrosis of the flap can be as high as 10 to 15% in patients undergoing reconstruction, but this risk is not affected by body habitus or body mass index and is lower than the risk for delayed wound healing.

How well did you know this?
1
Not at all
2
3
4
5
Perfectly
38
Q

A 44-year-old previously healthy woman comes to the clinic because of a 2-week history of a painless mass in the left breast. She initially felt this mass while taking a shower. Her mother was diagnosed with fibrocystic changes. The patient denies alcohol consumption and smoking cigarettes. Examination of the left breast shows a 5-cm mobile, painless mass in the left upper external quadrant without nipple discharge, skin retractions, or color changes. Examination of a specimen obtained on biopsy discloses a phyllodes tumor, and surgical excision of the lesion is planned. Which of the following is the most important factor to prevent local recurrence after surgery?

A) Adjuvant radiotherapy
B) Concurrent axillary node dissection
C) Postoperative chemotherapy
D) Surgical margins less than or equal to 0.5 cm
E) Wide surgical margins

A

The correct response is Option E.

In a young woman who has no history of breast cancer, presents with a painless mass, and has a mammogram suggestive of fibroadenoma but a core needle biopsy showing stromal hypercellularity with atypical spindle cells and a high mitotic rate, a phyllodes tumor must be suspected.

Phyllodes tumors are uncommon fibroepithelial breast tumors that behave like benign fibroadenomas, although they have a high propensity to recur locally. More aggressive tumors can metastasize distantly. Surgery is the preferred treatment for this condition. In this context, surgical margins greater than or equal to 1 cm have been associated with a lower recurrence rate in borderline and malignant tumors.

Axillary lymph node involvement is rare. Wide local excision or mastectomy with appropriate margins is the preferred clinical intervention.

Based on limited data, the role of systemic chemotherapy in phyllodes tumors is limited. Patients with benign or borderline phyllodes tumors are usually cured with surgery and should not be offered chemotherapy unless they develop unresectable metastases.

Local recurrence rate is higher after excision with narrower margins than broader ones. The efficacy of postoperative adjuvant radiotherapy for a breast phyllodes tumor is not clear. In clinical practice, the utilization of adjuvant radiotherapy for a phyllodes tumor appears to be modest.

How well did you know this?
1
Not at all
2
3
4
5
Perfectly
39
Q

A 35-year-old woman presents with unilateral swelling that has developed over the past 3 months. She underwent bilateral nipple-sparing mastectomy with immediate implant reconstruction with textured, round silicone gel implants 8 years ago. Ultrasound confirms periprosthetic seroma without any masses. Which of the following is the most appropriate next step in the management of this patient?

A) Core needle biopsy
B) Fine-needle aspiration
C) Implant removal and capsulectomy
D) MRI
E) Positron emission tomography (PET) scan

A

The correct response is Option B.

The clinical scenario is concerning for breast implant–associated anaplastic large-cell lymphoma (BIA-ALCL). Aspiration of the fluid seen on ultrasonound and pathologic evaluation is necessary to confirm the diagnosis. Following the National Comprehensive Cancer Network guidelines, initial workup of an enlarged breast should include ultrasound evaluation specifically for a fluid collection, a breast mass, or enlarged regional lymph nodes (axillary, supraclavicular, and internal mammary).

MRI is appropriate for cases where ultrasound is indeterminate or requires further confirmation. This patient does not have an identifiable mass amenable to core biopsy. Positron emission tomography (PET) scan is beneficial in confirmed cases to identify associated masses, chest wall involvement, regional lymphadenopathy, and/or metastasis. Implant removal and capsulectomy is appropriate once the diagnosis is confirmed.

How well did you know this?
1
Not at all
2
3
4
5
Perfectly
40
Q

A 50-year-old woman presents to the clinic to discuss breast reconstruction after bilateral mastectomy. She is interested in free tissue transfer. She has a diagnosis of systemic lupus erythematosus treated with chronic steroid therapy and wants to know if she is an appropriate candidate for free flap reconstruction. Which of the following statements best describes the surgical risks for this patient with lupus compared with the general population?

A) Higher rate of free flap failure
B) Higher risk of a thromboembolic event
C) Similar rate of hernias after abdominally based free flaps
D) Similar rates of infection

A

The correct response is Option B.

The statement which best describes the surgical risks for a patient with lupus undergoing free tissue transfer for breast reconstruction is that the patient has a higher risk of a thromboembolic event than the average patient.

The rate of free flap failure in patients with lupus is similar to patients without lupus. Chronic steroid use increases the risk of wound healing complications in patients with lupus, rather than increases the risk of free flap failure. Additionally, patients with lupus have an increased risk of abdominal wall bulge and hernia after abdominally based free flaps compared with the average population. Chronic steroid use also suppresses the immune system, predisposing patients treated with steroids to increased rates of infection compared to patients not taking steroids.

How well did you know this?
1
Not at all
2
3
4
5
Perfectly
41
Q

A 19-year-old woman with a medical history significant for Poland syndrome and a BMI of 19 kg/m2 undergoes first-stage breast reconstruction with a tissue expander that is complicated by extrusion and infection 40 days after implantation. A photograph is shown. Attempts at implant salvage are made. The presence of which of the following factors is most likely to lead to decreased salvage rates?

A) BMI of 19 kg/m2
B) Culture-positive Staphylococcus sp
C) Hemoglobin A1c of 6.5%
D) Prepectoral placement of the device
E) Use of acellular dermal matrix

A

The correct response is Option B.

It has been shown that successful breast device salvage in breast reconstruction is possible if caught early. However, there are associated factors with failure, including culture-positive Staphylococcus (epidermidis or aureus), as demonstrated by several studies. Other associated risk factors for failure include obesity, poorly-controlled diabetes, smoking, history of radiation therapy, postoperative seroma, and early contamination of the implant with biofilm formation. Therefore, prompt and aggressive intervention is warranted in these situations where the device is threatened by either infection and/or exposure. This includes both surgical and antimicrobial options.

In a 2017 study, prepectoral and subpectoral placement demonstrated comparable complications. Acellular dermal matrix did not increase failure rates.

How well did you know this?
1
Not at all
2
3
4
5
Perfectly
42
Q

A 42-year-old woman with a history of a cesarean delivery from a low-transverse abdominal incision is scheduled to undergo a unilateral deep inferior epigastric perforator (DIEP) flap breast reconstruction. BMI is 28 kg/m² and the distance from nipple to sternal notch is 24 cm per side. This patient’s history of cesarean delivery is most likely to have which of the following effects?

A) Decreased abdominal seroma
B) Decreased flap venous congestion
C) Increased flap arterial thrombosis
D) Increased flap fat necrosis
E) No overall effect

A

The correct response is Option B.

Pfannenstiel incisions are the preferred access for cesarean deliveries. They are not a contraindication for abdominal-based flaps for breast reconstruction because the deep inferior epigastric circulation is not disturbed. However, the superficial epigastric circulation may be divided. The net result appears to be a more robust venous circulation with a protective effect against fat necrosis in the flap. This venous division causes a delay-type phenomenon—during healing increased branches are formed from the superficial epigastric circulation, and there is some evidence of new connections to the venae comitantes of the deep epigastric venous circulation.

There is evidence, however, for an increased rate of abdominal healing problems, including seroma (15% versus 6%), wound healing problems, and fat necrosis in the abdomen. There is no evidence for an effect on the arterial circulation of the flap.

How well did you know this?
1
Not at all
2
3
4
5
Perfectly
43
Q

A 50-year-old woman comes to the office 6 weeks after undergoing right mastectomy and immediate placement of a tissue expander. She reports swelling and redness of the right breast. A photograph is shown. Which of the following factors is most predictive of implant salvage failure in this patient?

A) Culture positive for Pseudomonas species
B) Elevated body mass index
C) Periprosthetic seroma
D) Presence of cellulitis
E) Previous irradiation

A

The correct response is Option A.

Immediate implant-based reconstruction has become increasingly popular over the past two decades, accounting for over 70% of all reconstructions in the United States. The benefits of immediate reconstruction are numerous, including decreased recovery/number of required procedures and increased patient psychological well-being and aesthetic outcome. However, the complication (seroma, mastectomy flap necrosis, loss of implant, and infection) rates after implant-based reconstruction remain relatively high. Infection rates in the reported literature range from 2.5 to 24%.

Historically, periprosthetic infection or implant exposure mandated immediate implant removal. However, numerous studies over the past several decades have demonstrated implant salvage rates of 37.3 to 73% depending on the methods employed. Several studies have looked at the predictive factors that increase the risk of a failed salvage attempt. Salvage was typically defined as administration of systemic antibiotics (oral or intravenous), removal of the infected implant, partial/total capsulectomy, pocket curettage, implant pocket irrigation with antibiotic solution, and placement of a new device.

Factors associated with implant salvage failure include an elevated white blood cell count, elevated temperature, deep-seated pocket infection (purulent periprosthetic fluid), and atypical pathogens such as methicillin-resistant Staphylococcus aureus (MRSA) and Pseudomonas species. Spear et al. showed that 93.9% of mild implant infections (localized cellulitis) could be salvaged compared with a 30% salvage rate in the severe infection group. Factors such as smoking, chemotherapy, previous irradiation, mastectomy skin necrosis, increased BMI, and use of acellular dermal matrix (ADM) have demonstrated increased rates of implant-related infections, but these factors have not been demonstrated to increase the risk of implant salvage failure.

How well did you know this?
1
Not at all
2
3
4
5
Perfectly
44
Q

A 68-year-old woman comes to the office for a delayed breast reconstruction. She had right breast cancer and a mastectomy followed by chemotherapy and radiation therapy 1 year ago. BMI is 35 kg/m2. Medical history includes well-controlled type 2 diabetes mellitus, a previous cesarean section through a low transverse incision, and an open cholecystectomy through a subcostal incision. The patient requests autologous reconstruction, but the surgeon is not comfortable performing a free flap. Which of the following is the most appropriate method for reconstruction?

A) Bipedicled transverse rectus abdominis musculocutaneous (TRAM) flap
B) Contralateral pedicled TRAM flap
C) Ipsilateral pedicled TRAM flap
D) Surgical delay procedure followed by contralateral pedicled TRAM flap
E) Surgical delay procedure followed by ipsilateral pedicled TRAM flap

A

The correct response is Option D.

In this obese patient with right breast cancer and a previous subcostal incision, a delay procedure with a contralateral transverse rectus abdominis musculocutaneous (TRAM) flap is the most appropriate method for reconstruction. Although a contralateral TRAM flap can be performed without a delay procedure, it has been shown that the addition of a delay procedure decreases risks of ischemia to the flap. In addition, a delay procedure may also reduce risks of abdominal wall complications. In this patient, the subcostal incision excludes right-sided pedicled flap reconstruction, so an ipsilateral pedicled TRAM would not be the right choice, nor would a bipedicled TRAM flap.

Although there is controversy on which patients should have a delay procedure, the use of this technique has usually been limited to high-risk patients and to those requiring large amounts of tissue. Multiple reports have shown that obese patients undergoing a delay can decrease the risks of tissue related ischemia. In a paper by Wang et al., the delay procedure was performed at least 14 days prior to the reconstruction; however, other studies have shown improvements at 7 days. The procedure described consists of ligation of both deep inferior epigastric arteries and veins bilaterally accessed from an inferior flap incision. This can also be done laparoscopically. Some authors advocate more extensive incisions and elevating portions of the flap; however, there is little data to show that this is effective or necessary.

How well did you know this?
1
Not at all
2
3
4
5
Perfectly
45
Q

A 52-year-old woman undergoes autologous breast reconstruction with unilateral deep inferior epigastric perforator (DIEP) flaps. According to the Hartrampf model of perfusion zones, if the lateral row perforator vessels are used, in which chronological order will the flap zones be perfused?

A) I – II – III – IV
B) I – III – II – IV
C) II – I – III – IV
D) II – I – IV – III
E) IV – III – II – I

A

The correct response is Option B.

In medial perforator-based flaps, the zones are perfused in the order I – II – III – IV (A) as shown in the image. In lateral perforator-based flaps, however, the zones are perfused in the order I – III – II – IV (B).

How well did you know this?
1
Not at all
2
3
4
5
Perfectly
46
Q

A 64-year-old woman who is postmenopausal asks why she has not been prescribed hormone replacement therapy with estrogen and progestin like her mother was. Supplementation with these hormones is associated with an increased risk for which of the following?

A) Coronary artery disease
B) Diabetes
C) Endometrial cancer
D) Invasive breast cancer
E) Osteoporosis

A

The correct response is Option D.

Hormone replacement therapy has fallen out of favor because of a risk profile that is believed to exceed the potential benefits. Combined estrogen and progestin supplementation is thought to be associated with an increased risk for invasive breast cancer but may decrease the risk for diabetes and osteoporosis. It is thought to not impact the risk for coronary artery disease or endometrial cancer.

How well did you know this?
1
Not at all
2
3
4
5
Perfectly
47
Q

A 46-year-old woman who is 5 ft 7 in (170 cm) tall and weighs 135 lbs (61 kg) is evaluated one year following bilateral nipple-sparing mastectomy and immediate reconstruction with placement of 350-mL smooth, round silicone gel implants beneath the pectoralis major muscle. Since the surgery, she has experienced hyperdynamic deformity of her breasts. On physical examination, the breast reconstruction appears natural, and there is significant movement of the breasts when the patient flexes her chest. Which of the following is the most appropriate management for this patient?

A) Inject botulinum toxin into the pectoralis major muscle
B) Inject triamcinolone-40 into the areas of tenderness using ultrasound guidance
C) Move the implants to the prepectoral plane and cover them fully with acellular dermal matrix
D) Perform a breast MRI to assess for rupture of the implants
E) Refer the patient to a physical therapist for range of motion, massage, and ultrasound treatments

A

The correct response is Option C.

This patient is experiencing significant movement because her implants were placed beneath the pectoralis major muscles. While reconstruction options are limited in this otherwise healthy and very thin patient who is not a good candidate for fat grafting or pedicled or free tissue transfer, placing implants over the pectoralis major muscles and covering the implants fully with acellular dermal matrix would be the most appropriate method of reconstructing her breasts and addressing her concerns.

Physical therapy and muscle relaxants are unlikely to produce long-term improvement. An MRI would likely be nondiagnostic, and even if her implants were ruptured, change to a prepectoral plane is still indicated. Botulinum toxin type A is likely not as effective for long-term significant improvement as reoperation. Triamcinolone would not be effective for hyperdynamic deformity.

How well did you know this?
1
Not at all
2
3
4
5
Perfectly
48
Q

The capsules from patients with breast implant–associated anaplastic large-cell lymphoma (ALCL) have significant presence of which of the following bacteria?

A) Escherichia coli
B) Ralstonia pickettii
C) Staphylococcus aureus
D) Pseudomonas aeruginosa
E) Serratia marcescens

A

The correct response is Option B.

Most concerning in the past two decades is the incidence of breast implant–associated anaplastic large-cell lymphoma (ALCL). This entity was first diagnosed and associated with breast implants in 1997, and is almost only associated with a history of textured implants and/or tissue expanders. The most common presentation of these patients is late seroma, with some patients presenting with mass, tumor erosion, or lymph node metastasis. A recent review of the world literature on this entity include the following: (1) 173 cases were documented, (2) no cases were found in patients with documented smooth devices only (although this remains controversial, as the data in many cases are incomplete), (3) there may be an associated genetic predisposition as suggested for cutaneous T-cell lymphoma, and (4) the cause is likely multifactorial.

Bacterial biofilm is thought to be an inciting factor for the development of both breast-implant related ALCL and Non-Tumor related capsule contractures. The capsules from patients with tumor had significant presence of Gram-negative bacteria (Ralstonia species) compared to nontumor capsules (Staphylococcus species). Such data may support the bacterial induction model, as there are also other types of implant-associated lymphomas.

How well did you know this?
1
Not at all
2
3
4
5
Perfectly
49
Q

A 45-year-old woman who is obese is considering unilateral mastectomy and reconstruction of the left breast because of invasive ductal carcinoma. Which of the following patient characteristics is associated with the lowest risk for complications from a nipple-sparing mastectomy?

A) BMI of 41 kg/m2
B) Grade III ptosis of the breast
C) Nipple retraction
D) Tumor distance from nipple of 5 cm
E) Tumor size of 6 cm

A

The correct response is Option D.

Nipple-sparing mastectomy is increasing in popularity. To decrease the risk for surgical complications as well as oncologic complications, smaller tumors located further from the nipple in patients without morbid obesity or severe ptosis are considered better candidates for treatment with nipple-sparing mastectomy. Clinical involvement of the nipple, including retraction, would suggest that nipple-sparing mastectomy should not be performed.

References

How well did you know this?
1
Not at all
2
3
4
5
Perfectly
50
Q

Which of the following is the most likely chronic effect of post-mastectomy radiation therapy?

A) Desquamation
B) Dyspigmentation
C) Edema
D) Erythema
E) Ulceration

A

The correct response is Option B.

Radiation therapy induces tissue injury that can be categorized as acute or chronic. The spectrum of acute injury includes erythema, edema, desquamation, hyperpigmentation, and ulceration, ranging from mild to severe. Acute radiation dermatitis occurs in upward of 85% of treated patients. Chronic injury involves skin atrophy, dryness, telangiectasia, dyspigmentation, and dyschromia. In the breast, it leads to chronic fibrosis of the skin and subcutaneous tissues. This fibrosis and surrounding injury can lead to pain and restricted movement of the arm. The chronic changes from radiation can take months to years to fully manifest.

How well did you know this?
1
Not at all
2
3
4
5
Perfectly
51
Q

A 43-year-old woman who is BRCA-positive is scheduled to undergo bilateral mastectomy. Tissue expander–based reconstruction is planned. Which of the following is the optimal duration of antibiotic prophylaxis for this patient?

A) No preoperative antibiotic
B) One preoperative antibiotic dose and another dose during skin closure
C) One preoperative antibiotic dose, followed by 24 hours of treatment while in the hospital
D) One preoperative antibiotic dose, followed by 24 hours of treatment while in the hospital and then discharge on oral antibiotics until drains are removed
E) One preoperative antibiotic dose, followed by 24 hours of treatment while in the hospital and then maintenance on oral antibiotics until tissue expanders are exchanged

A

The correct response is Option C.

The overall complication rate in breast reconstructive surgery is as high as 60%. Infection rates can exceed 20%, much higher than in clean elective surgery. The CDC guidelines suggest only 24 hours of peri-operative antibiotics beginning thirty minutes prior to skin incision. However, not all plastic surgeons agree with this. A 2013 meta-analysis found when comparing combined patient cohorts receiving no antibiotics, antibiotics for less than 24 hours, and antibiotics for greater than 24 hours, the average infection rates were 14.4, 5.8, and 5.8%, respectively. This demonstrated that the administration of antibiotics made a difference, however duration beyond 24 hours did not.

A study was published in 2013 evaluating the difference in surgical site infection between two different prophylactic antibiotic durations (24 hours and until drain removal). In this prospective, randomized, controlled non-inferiority trial, 24 hours of antibiotics is equivalent to extended oral antibiotics for surgical-site infection in tissue expander immediate breast reconstruction patients.

How well did you know this?
1
Not at all
2
3
4
5
Perfectly
52
Q

A 45-year-old woman comes to the office for consultation about immediate bilateral breast reconstruction of a right-sided tumor measuring 2.5 cm. Biopsy reveals a HER-2/neu negative invasive ductal carcinoma without lymphovascular invasion. The patient requests nipple-sparing mastectomy. Physical examination shows a palpable mass is located in the right upper outer quadrant approximately 1.5 cm from the nipple-areola complex and is freely mobile. There is no lymphadenopathy on exam. Based on current literature, which of the following best describes this patient’s candidacy for the requested procedure?

A) Good candidate based on current presentation
B) Not a candidate because of lymph node status
C) Not a candidate because of tumor location
D) Not a candidate because of tumor pathology
E) Not a candidate because of tumor size

A

The correct response is Option C.

Nipple-sparing mastectomy or total skin-sparing mastectomy is becoming an increasingly popular choice for women because of the excellent cosmetic outcomes and the ability to save the nipple-areola complex that may provide psychological benefits with increased patient satisfaction as well. Nipple-sparing mastectomy appears to be oncologically safe with low risks of cancer recurrence in the literature thus far. However, there has been little long-term follow-up, so this approach is still somewhat controversial because the oncologic safety and locoregional recurrence have not been examined definitively. Although certain centers are pushing the envelope regarding the use of this technique in a wide range of patients, the current literature supports the following exclusion criteria:

A. Tumor size greater than 5 cm B. Tumor location less than 2 cm from the nipple C. Evidence of axillary disease D. Tumor involvement on retroareolar biopsy E. Lymphovascular invasion, human epidermal growth factor receptor-2 positivity, and/or HER-2/neu positivity on biopsy

The current patient’s tumor was found to have a tumor-to-nipple distance of 1.5 cm which is a relative contraindication to nipple-sparing mastectomy in this case.

How well did you know this?
1
Not at all
2
3
4
5
Perfectly
53
Q

A 54-year-old woman undergoes bilateral immediate tissue expander–based breast reconstruction. BMI is 36 kg/m2. On postoperative day 10, examination shows bilateral breast erythema; empiric antibiotics are started. In addition to Staphylococcusspecies, which of the following bacteria should be treated as the next most likely pathogen?

A) Bacteroides fragilis
B) Enterococcus faecalis
C) Mycobacterium marinum
D) Pasteurella multocida
E) Pseudomonas aeruginosa

A

The correct response is Option E.

After Staphylococcus aureus and S. epidermidis, Pseudomonas aeruginosa is among the the next most common sources of breast infections. It is common in hospitalized or immunocompromised patients, as well as patients with foreign body devices such as catheters or implants. It is a gram negative rod, and common antibiotic treatments include advanced β-lactams (piperacillin, ceftazidime), carbapenems, quinolones, and aminoglycosides. Dual coverage is often recommended in severe infections. In the case of breast implant infections, if there is not rapid improvement on antibiotic therapy, or if significant systemic symptoms develop (vital sign instability, high white blood cell count, fever, renal impairment), then surgical washout and device removal is mandatory. In patients without systemic symptoms, wash out and new implant placement can be an option in carefully selected and counseled patients. Other breast pathogens include Escherichia coli, Propionibacterium, and Corynebacterium.

More than 300,000 breast implant procedures are performed each year in the United States. In reconstructive cases, the infection rate averages 6% and the explantation rate 3% (range, 1.5 to 8%). Preventative measures include proper patient selection, preoperative MRSA management when carriers are suspected, routine presurgery chlorhexidine washes, proper antibiotic timing presurgery and continuation of antibiotics in implant reconstruction cases for at least 24 hours (though the optimal treatment duration has not yet been determined).

None of the other bacteria listed are common in breast infections, though all are common pathogens. Bacteroides are anaerobic gram-negative rods that are common in gut flora and feces.

Enterococcus faecalis is a frequent cause of nosocomial infection, with a high prevalence of multi-drug resistance. It is a gram-positive coccus, and is not commonly seen in breast surgery patients as it primarily colonizes the digestive tract.

Mycobaterium marinum is a rare pathogenic cause of hand infections from injuries that occur in aquatic environments.

Pasteurella multocida is a frequent cause of animal bite infections, particularly from cats and dogs.

How well did you know this?
1
Not at all
2
3
4
5
Perfectly
54
Q

A 62-year-old woman is evaluated because of a new 2 × 2-cm open area near her left axillary fold. Medical history is significant for left breast cancer previously treated with bilateral mastectomies, left axillary node dissection, and adjuvant chemoradiation 10 years ago. A photograph is shown. She has been compliant with postoperative oncologic surveillance and has had no recent trauma. Which of the following underlying conditions is most likely responsible for her current presentation?

A) Empyema with spontaneous drainage
B) Lymphedema drainage tract
C) Osteoradionecrosis of the underlying rib(s)
D) Recurrent breast cancer
E) Skin ulceration from intertriginous shearing forces

A

The correct response is Option C.

The effects of ionizing radiation are permanent and may present either acutely or in delayed fashion, even years after the original radiation insult. The mechanism of injury from this radiation is through free radical production which, in turn, directly damages the DNA. In the acute period, the effects of radiation may manifest themselves as erythema and edema of the skin, vasodilation with endothelial edema, and lymphatic obliteration. This eventually leads to capillary thrombosis and subsequent inadequate tissue oxygenation. Over time, nonhealing ulcers can spontaneously develop, sometimes years later.

Although recurrent cancer is always a concern in patients with a personal history of cancer, proper, regular, and thorough surveillance can often detect recurrences early, especially in compliant patients. Most recurrences occur within the first 5 years.

Abscesses usually present initially with pain, erythema, and localized fluctuance, and often with associated fever and/or malaise. Spontaneous necessitation to the skin would also result in purulent drainage.

Lymphedema can be a chronic condition after mastectomy and axillary node dissection, and is usually manifested as generalized edema of the ipsilateral upper extremity. Sinus tract formation is rare.

Intertriginous shearing would most often present as superficial epidermal loss with possible superinfection with yeast due to moisture.

How well did you know this?
1
Not at all
2
3
4
5
Perfectly
55
Q

Which of the following factors is most likely to lead to a patient undergoing breast reconstruction after mastectomy?

A) BRCA1/BRCA2 status
B) Insurance coverage through Medicaid
C) Living close to a reconstructive surgeon
D) Patient age
E) Residence in the southeastern United States

A

The correct response is Option C.

Several large population-based studies in different regions of the United States show that provision of breast reconstruction is not uniform among all mastectomy patients. The two biggest factors are geographic distance from reconstructive surgeons and insurance status. Patients who live farther away from providers of reconstruction and those uninsured or with Medicaid have a much lower incidence of reconstruction compared to the overall cohorts. Most studies also show BRCA positive and younger patients tend to have a higher incidence of bilateral reconstruction.

How well did you know this?
1
Not at all
2
3
4
5
Perfectly
56
Q

A 48-year-old woman is evaluated for bilateral microsurgical breast reconstruction. Compared with the deep inferior epigastric perforator (DIEP) flap, the superficial inferior epigastric artery (SIEA) flap places the patient at a greater risk for which of the following complications?

A) Abdominal bulge
B) Donor site dehiscence
C) Fat necrosis
D) Flap failure
E) Umbilical necrosis

A

The correct response is Option D.

Several comparative studies have reported a higher incidence of anastomotic thrombosis and failure with the superficial inferior epigastric artery (SIEA) flap than with flaps based on the deep inferior epigastric artery (DIEA). These failure rates range from 7.35 to 14%. Most of these failures were arterial in nature. Since SIEA flaps do not require an incision into the anterior rectus sheath or rectus muscle, bulges do not occur. Reported fat necrosis rates are similar between SIEA and DIEA flaps. There is no evidence for a difference in donor site dehiscence or umbilical necrosis rates.

How well did you know this?
1
Not at all
2
3
4
5
Perfectly
57
Q

Which of the following is most commonly associated with alloplastic breast reconstruction in the setting of adjuvant radiation therapy?

A) Decreased rates of implant rupture
B) Decreased rates of seroma
C) Increased patient satisfaction
D) Increased risk for capsular contracture

A

The correct response is Option D.

Alloplastic breast reconstruction increases the risk for capsular contracture in the setting of adjuvant radiation therapy. It can also increase the risks for seroma, wound healing complications, and infections. Radiation would not decrease the rates of implant rupture and would not increase patient satisfaction.

How well did you know this?
1
Not at all
2
3
4
5
Perfectly
58
Q

A 65-year-old woman with a history of left mastectomy for breast cancer 10 years ago undergoes biopsy of a suspicious lesion of the right breast found on a recent mammogram. Examination of the biopsy specimen confirms a right breast carcinoma. This lesion most likely originated from which of the following structures?

A) Adipose tissue
B) Areolar skin
C) Lactiferous duct
D) Lymph node
E) Pectoralis major muscle

A

The correct response is Option C.

Women who were previously treated for breast cancer are at increased risk for development of a metachronous lesion of the contralateral breast. Cancers of the breast are typically adenocarcinomas, arising from the glandular tissue such as the ducts or lobules. Paget disease of the breast would involve the areolar skin but is fairly uncommon. Sarcomas arising from the connective tissue (such as adipose or muscle) are also rare. Breast adenocarcinomas do not originate from lymphatic tissue.

How well did you know this?
1
Not at all
2
3
4
5
Perfectly
59
Q

A 48-year-old woman undergoes immediate unilateral breast reconstruction with a free deep inferior epigastric artery perforator (DIEP) flap. At the conclusion of the procedure, the flap skin paddle is noted to have venous congestion. Upon reexploration, the venous anastomosis appears patent with venous outflow detected by handheld pencil Doppler evaluation, but the flap continues to have venous congestion with brisk capillary refill. Which of the following is the most appropriate next step in management?

A) Apply leeches postoperatively
B) Loosely re-inset the flap and monitor closely
C) Perform a second venous anastomosis using the superficial inferior epigastric vein
D) Perform a second venous anastomosis using the vena comitans
E) Revise the venous anastomosis using a hand-sewn technique

A

The correct response is Option C.

Preservation of the superficial inferior epigastric veins (SIEV) during flap harvest is a useful preventive measure in microsurgical free tissue transfer operations. These veins can serve as important lifeboats to augment venous outflow in the setting of venous congestion. Typically, if a free flap demonstrates venous congestion, the inset should be taken down and the pedicle, recipient vessels, and anastomoses should be interrogated. Simple issues, such as mechanical compression or twisting of the vein, should be ruled out. Next, the SIEV should be inspected. If engorged, the flap is likely reliant on superficial outflow, and this vein should be connected to a recipient vessel to augment the venous outflow of the flap. Options for recipient veins include an anterograde branch on the pedicle vena comitans, or in a retrograde fashion to the vena comitans that was not used in the initial set of anastomoses.

In this case scenario, the flap continued to demonstrate venous congestion intraoperatively. This makes it unlikely that tension or pressure from the inset of the flap was causing the venous outflow obstruction. Furthermore, leech therapy is not indicated for a free flap with global venous congestion.

The venous coupling device is safe and effective for the anastomosis of veins in DIEP flap surgery. It has not been associated with patency rates that are different from hand-sewn anastomoses. The coupling device, however, has been shown to reduce the microsurgery time.

The use of one or two veins in microsurgical free tissue transfer is a topic that has been debated for several years. While some studies indicate that the use of two venous connections may reduce the velocity of blood flow across the anastomosis, there is not sufficient data to support differences in flap outcomes or thrombotic events. Therefore, the routine use of a second vein is largely up to surgeon preference.

How well did you know this?
1
Not at all
2
3
4
5
Perfectly
60
Q

A 43-year-old woman undergoes the second stage of tissue expander–based breast reconstruction. Exchange of the tissue expander for a smooth round silicone implant is planned along with a superior capsulotomy and fat grafting to the upper pole for contour improvement. Which of the following is an increased risk associated with fat grafting to the breast in this patient?

A) Anaplastic large cell lymphoma
B) Benign lesions
C) Hypopigmentation
D) Infection
E) Recurrent breast cancer

A

The correct response is Option B.

Autologous fat grafting is a widely accepted technique in breast reconstruction. A large systematic review recently confirmed the oncologic safety of this technique but did report a significant incidence of benign lesions including cysts and calcifications. Fat grafting is not associated with an increased risk of recurrent breast cancer, infection or hypopigmentation.

How well did you know this?
1
Not at all
2
3
4
5
Perfectly
61
Q

A 36-year-old woman with invasive ductal carcinoma of the left breast comes to the office to discuss options for immediate breast reconstruction. Examination shows that the tumor is positive for estrogen receptor (ER) and progesterone receptor (PR) expression. Family history is negative for breast and ovarian cancer, and genetic testing shows no abnormalities. Bilateral mastectomy is planned. Which of the following outcomes is most likely in this patient following contralateral prophylactic mastectomy of the right breast?

A) Decreased number of anticipated overall operations
B) Decreased risk of recurrence of the known cancer
C) Increased cure rate for her known cancer
D) Increased possibility of future breast-feeding
E) Increased risk of surgical site complications

A

The correct response is Option E.

Contralateral prophylactic mastectomy (CPM) has become increasingly common in recent years. Women who are at higher than average risk of developing breast cancer may have a clear oncologic benefit from CPM in terms of risk reduction. This would include women who are carriers of high-risk genetic mutations such as BRCA1 and BRCA2, women with a strong family history of breast cancer, and young women with high risk of aggressive tumors. However, for women with average risk of contralateral breast cancer (2 to 6% over 10 years), studies have not supported an oncologic benefit to CPM, attitudes of the lay public and media attention notwithstanding.

Since the surgeries of each breast, involving both mastectomy and reconstruction, are largely (although not completely) independent of one another, the risk of surgical complications to one side or the other is nearly additive, resulting in an approximate doubling of those risks.

CPM has not been shown to improve the cure rate for the known cancer treated with therapeutic mastectomy.

CPM has not been shown to reduce risk of recurrence for the known cancer on the primary side.

CPM would not be expected to decrease the number of anticipated operations. Given the increased likelihood of complications, one may reasonably expect the same number of operations or greater (for treatment of complications), but not fewer.

Removal of both breasts eliminates the ability to breast-feed in the future.

How well did you know this?
1
Not at all
2
3
4
5
Perfectly
62
Q

A 54-year-old woman with breast cancer undergoes a skin-sparing mastectomy with tissue expander reconstruction. Adjuvant chemotherapy and subsequent radiation therapy have been recommended. Chemotherapy proceeds during tissue expansion. Radiation may be performed either before or after the implant exchange procedure. When compared with radiating the permanent implant, radiating the tissue expander is most likely to increase the risk of which of the following?

A) Cancer recurrence
B) Capsular contracture
C) Device rupture
D) Explantation
E) Radiation dermatitis

A

The correct response is Option D.

Cordeiro et al. updated the largest series of women undergoing two-stage implant breast reconstruction who require postmastectomy radiation. The authors found that radiating the tissue expander, as opposed to the permanent implant, increased the rate of reconstructive failure by 46%. However, aesthetic results were better and capsular contracture was less frequent. There were no differences in patient-reported outcomes. This study confirmed the earlier findings of Nava et al. regarding reconstruction failure (explantation). There are no known differences in cancer recurrence between the two approaches discussed.

How well did you know this?
1
Not at all
2
3
4
5
Perfectly
63
Q

A 52-year-old woman comes to the office to discuss revision of breast reconstruction following mastectomy for breast cancer. She is undergoing adjuvant treatment with an agent that interferes with her body’s natural mechanisms that promote native breast growth, but she cannot remember its name. She is most likely being treated with which of the following agents?

A) Alkylating agent (cyclophosphamide)
B) Anthracycline (doxorubicin)
C) Aromatase inhibitor (anastrozole)
D) Platinum agent (cisplatin)
E) Taxane (paclitaxel)

A

The correct response is Option C.

Aromatase inhibitors such as anastrozole impair conversion of androgens to estrogens. Estrogens promote normal breast tissue growth as well as growth of many breast cancers. The other options (taxanes, anthracyclines, alkylating agents, and platinum agents) are all chemotherapeutic agents that do not particularly target hormones involved in normal breast growth mechanisms.

How well did you know this?
1
Not at all
2
3
4
5
Perfectly
64
Q

Improvement in which of the following is an advantage of nipple-areola complex reconstruction?

A) Breast feeding ability
B) Breast mound shape
C) Reactivity to touch
D) Satisfaction with reconstruction
E) Sexual sensation

A

The correct response is Option D.

Satisfaction with reconstruction, quality of life, and feeling of completeness with reconstruction have all been shown to be linked to nipple and/or areola reconstruction. The reconstruction does not restore nipple function; therefore, breast feeding, sexual sensation, and reactivity to touch are not accomplished with nipple and/or areola reconstruction. Often the nipple reconstruction, if using local tissue techniques, may actually flatten the anterior aspect of the breast and is not thought to positively impact the breast mound shape.

How well did you know this?
1
Not at all
2
3
4
5
Perfectly
65
Q

A 75-year-old woman is evaluated after undergoing bilateral autologous breast reconstruction with deep inferior epigastric artery perforator (DIEP) flaps 2 days ago. The patient had been recovering well until this morning, when she developed acute respiratory insufficiency with hypoxia, tachycardia, and tachypnea. Pulmonary embolism is suspected, and pulmonary CT angiography has been ordered. Baseline creatinine level is 1.1 mg/dL. Which of the following steps should be taken to prevent contrast-induced nephropathy in this patient?

A) Hydration with hydroxyethyl starch (colloid)
B) Hydration with 0.9% saline
C) No preventive measures are needed
D) Pretreatment with the antioxidant N-acetylcysteine
E) Pretreatment with 0.9% saline and diphenhydramine

A

The correct response is Option B.

Although the risk of contrast-induced nephropathy (CIN) is overall relatively low, geriatric patients are at higher risk for this complication secondary to their high prevalence of risk factors for renal dysfunction. In this patient population, even normal laboratory creatinine concentrations are not always indicative of normal glomerular filtration rate (GFR), and every effort should be made to prevent the development of CIN. Important prophylactic strategies to avoid this adverse effect include optimization of fluids and avoidance of dehydration, use of non-ionic contrast media, and avoiding repeated exposure to contrast at close intervals.

This geriatric patient should be well hydrated prior to undergoing exposure to this high-contrast study. Minimizing fluid and performing no intervention will increase the patient’s risk of developing CIN even with a normal creatinine concentration. Antioxidants such as N-acetylcysteine, vasodilators such as dopamine, and colloid solutions such as Hespan have not proven to be beneficial in preventing renal dysfunction. Pretreatment with 0.9% saline and diphenhydramine is used in patients who have allergies to the contrast dye to minimize the allergic reaction and plays no role in preventing CIN.

How well did you know this?
1
Not at all
2
3
4
5
Perfectly
66
Q

A 46-year-old woman with cancer of the right breast comes to the office to discuss a right mastectomy with immediate implant reconstruction and a symmetry procedure for the contralateral breast. Physical examination shows bilateral Grade 3 ptosis with volume symmetry. The patient currently wears a size 34C brassiere and desires to remain the same size. Which of the following is the most appropriate procedure for the contralateral breast?

A) Augmentation mammaplasty
B) Fat injections
C) Mastopexy
D) Reduction mammaplasty

A

The correct response is Option C.

Contralateral breast procedures are frequently necessary to achieve symmetry following mastectomy and reconstruction. Options to achieve symmetry include mastopexy, reduction mammaplasty, or augmentation mammaplasty combined with mastopexy. Volume symmetry can be achieved through reduction mammaplasty or fat injections. Positional asymmetry of the contralateral breast and the nipple-areola complex may require correction with mastopexy. Augmentation mammaplasty with mastopexy may be indicated for the correction of volume and positional asymmetry. The most appropriate contralateral procedure in a patient with symmetric volumes of the breast and ptosis of the contralateral breast is mastopexy.

67
Q

A 35-year-old woman with a Stage T2 infiltrating ductal carcinoma is scheduled to undergo a skin-sparing, right total mastectomy and a nipple-sparing, left prophylactic mastectomy. The possibility of adjuvant radiation therapy to the right breast depends on the final surgical pathology. The patient has a history of smoking. BMI is 28 kg/m2. She wears a brassiere with a D cup and would like the postoperative result to be of a similar size. Which of the following immediate bilateral reconstructive techniques is most appropriate for this patient?

A) Abdominal-based free flaps
B) Gluteal-based free flaps
C) Latissimus dorsi myocutaneous flaps and silicone implants
D) Silicone implants and acellular dermal matrix
E) Tissue expanders and acellular dermal matrix

A

The correct response is Option E.

For this patient in whom postoperative radiation therapy is possible, the best first-stage, immediate reconstructive approach is placement of tissue expanders with acellular dermal matrix. The outcome of immediate autologous flap reconstruction may be compromised if subjected to adjuvant radiation therapy and is best delayed until after such treatment has been rendered. Although successful, cost-effective outcomes are possible with a single-stage, direct-to-implant approach, this patient has risk factors for early revision and implant failure due to her large breasts and history of smoking.

68
Q

After a nipple-sparing mastectomy, which of the following branches of an intercostal nerve predominantly provides remaining sensation to the nipple-areola complex?

A) Anterior branch of the fifth
B) Anterior branch of the fourth
C) Lateral branch of the fifth
D) Lateral branch of the fourth
E) Lateral branch of the third

A

The correct response is Option B.

The cutaneous innervation of the female breast is derived medially from the anterior cutaneous branches of the first to sixth intercostal nerves and laterally from the lateral cutaneous branches of the second to seventh intercostal nerves. The nipple-areola complex is physiologically innervated by the lateral and anterior branches of the third to fifth intercostal nerves. The fourth intercostal nerve has further shown to be most consistent in various anatomical studies.

However, the anterior branches take a superficial course within the subcutaneous tissues of the medial breast while the lateral branches take a deep course within the pectoral fascia and reach the nipple via the breast parenchyma. Therefore, the lateral branches are most likely resected during mastectomy and contribute little to the postoperative innervation of the nipple-areola complex.

69
Q

Following a skin-sparing mastectomy, a 39-year-old woman undergoes deep inferior epigastric perforator (DIEP) flap breast reconstruction. To augment flap sensation, the anterior sensory branch of the fourth intercostal nerve is coapted to which of the following nerves within the DIEP flap?

A) Genitofemoral
B) Iliohypogastric
C) Ilioinguinal
D) Intercostal
E) Lateral femoral cutaneous

A

The correct response is Option D.

The third, fourth, and fifth intercostal nerves are responsible for innervation of the majority of the breast. The anterior branch of the fourth intercostal nerve provides most erogenous sensation to the nipple. Sensation to the lower abdomen arises from segmental cutaneous branches of the intercostal nerve, which travel through the rectus abdominis muscle. T10 provides sensation to the dermatome, including the periumbilical region, and is most commonly used. The iliohypogastric nerve provides sensation to the lateral gluteal region. The ilioinguinal nerve provides sensation to the upper medial thigh. The genitofemoral nerve provides sensation to the upper anterior thigh and mons pubis. The lateral femoral cutaneous nerve provides innervation to the lateral thigh and is not used for this purpose.

70
Q

Which of the following is a risk factor for hormone-sensitive breast cancer?

A) Breast-feeding
B) Early age at first pregnancy
C) Early menopause
D) Late menarche
E) Post-menopausal obesity

A

The correct response is Option E.

Post-menopausal obesity is associated with increased adipose production of estrogen, which can increase the risk for hormone-sensitive breast cancer. Other options (late menarche, early menopause, and breast feeding) decrease the number of menstrual cycles, and therefore may decrease the risk of breast cancer. Early age at first pregnancy is also associated with decreased risk for hormone-sensitive breast cancers.

71
Q

A 54-year-old woman with BRCA mutation is scheduled for a bilateral nipple-sparing mastectomy. Which of the following would lead to the highest risk for postoperative nipple necrosis?

A) Grade 2 breast ptosis
B) Periareolar mastectomy incision
C) Previous excisional breast scar
D) Tumor greater than 1.5 cm
E) Use of smooth round gel implants

A

The correct response is Option B.

Nipple-sparing mastectomy and direct to implant reconstruction is becoming increasingly popular in the setting of prophylactic mastectomies. The criteria for nipple-sparing mastectomies have been increased to tumors not involving the nipple-areola complex, with some surgeons requiring a distance of 2 cm from the nipple and others espousing just a clean surgical margin at the nipple. Nipple-sparing mastectomies do not require recreating the breast envelope and therefore direct to implant reconstruction is possible. Increased risk for incidence of nipple necrosis in the setting of direct to implant reconstruction is associated with mastectomy incision involving the areola. The use of smooth round gel implants, grade 2 ptosis, tumor size, or previous breast biopsy scars are not associated with nipple necrosis.

72
Q

An otherwise healthy 16-year-old girl comes to the office because of a painless mass in the left breast. Physical examination of the left breast discloses a circumscribed firm, rubbery, 3-cm mass without overlying skin changes, and no axillary lymphadenopathy. Results of a pregnancy test are negative. Which of the following is the most likely diagnosis?

A) Common fibroadenoma
B) Giant fibroadenoma
C) Lactating adenoma
D) Phyllodes tumor
E) Tubular adenoma

A

The correct response is Option A.

A common fibroadenoma is the most likely diagnosis of this patient. Common fibroadenoma is the most common breast tumor in adolescent females and present between the ages of 14 and 16.

Juvenile fibroadenoma is a variant of fibroadenoma and is usually seen in adolescents and young adults. It is associated with a normal stromal/epithelial balance, which distinguishes it from phyllodes tumor, and has both stromal and epithelial hyperplasia. In addition, juvenile fibroadenomas are characterized by rapid growth.

A giant fibroadenoma is a clinical diagnosis, rather than a pathologic diagnosis. It is characterized by its size, usually greater than 5 cm.

Complex fibroadenoma is characterized by fibrocystic changes on glandular tissue with underlying features of common fibroadenoma on pathologic analysis.

A tubular adenoma has glandular proliferation on pathologic analysis, and while it is a subtype of fibroadenoma, it is not as common as common fibroadenoma. A lactating adenoma, similarly, is defined by the presence of secretory hyperplasia of lobules on pathologic analysis. Lactating adenomas are so defined because of the histologic presence of secretory hyperplasia, and they commonly occur in pregnant or lactating women. Many lactating adenomas will spontaneously regress.

A phyllodes tumor is typically a large, rapidly growing lesion and can be either benign or malignant. It is rare in adolescents, but when found, is usually aggressive. Phyllodes tumor is related to fibroadenoma and is distinct from other forms of breast cancer. Treatment is wide local excision.

73
Q

A 30-year-old woman comes to the office because of pain in the left breast. Two weeks ago, she underwent core needle biopsy of a breast mass that was diagnosed as benign. Family history does not include breast cancer. She does not smoke cigarettes. On examination, the left breast is erythematous and tender to palpation, and the skin of the breast is retracted laterally. There is a palpable, rope-like mass that courses longitudinally along the breast. Which of the following is the most appropriate diagnosis and treatment of this lesion?

A) Breast abscess, perform incision and drainage of the mass
B) Fibrocystic changes, perform biopsy
C) Mastodynia, treat with 10-day course of broad-spectrum antibiotics
D) Nipple papilloma, perform diagnostic mammography
E) Superficial thrombophlebitis, manage with analgesics

A

The correct response is Option E.

Mondor disease, or superficial thrombophlebitis of the breast, involves the superficial veins of the breast and anterior chest wall. It may occur following surgery, core biopsy, irradiation, or trauma. Clinical manifestations include pain, redness and swelling, and the presence of a thickened tender cord. This condition usually resolves in 4 to 6 weeks with symptomatic treatment using pain relief.

Nipple papillomas may be identified as a mass on breast imaging or may be found incidentally. They frequently present with bloody nipple discharge. While not concerning in and of themselves, these lesions may harbor areas of atypia or ductal carcinoma in situ, and are treated with core needle biopsy.

While a breast abscess is possible after a diagnostic procedure such as a biopsy, it would present as localized swelling, tenderness, and induration. The skin retraction and rope-like mass would not be present.

Fibrocystic changes in the breast present as a solitary mass and may cause patients to seek medical attention because of associated pain. They may fluctuate in size and tenderness during a patient’s menstrual cycle. Because no breast mass can be definitively declared benign on physical examination alone, imaging and/or biopsy may be considered.

Breast pain in the absence of a finding on physical examination may have a number of causes including menstrual changes, breast hypertrophy, diet, hormone replacement therapy, ductal ectasia, mastitis, malignancy, and hidradenitis. While the patient does have breast pain, the other physical findings rule out mastodynia alone.

74
Q

Oral contraceptive use increases the incidence of which of the following histologic risk factors for breast cancer?

A) Apocrine metaplasia
B) Atypical hyperplasia
C) Intraductal hyperplasia
D) Intraductal papilloma
E) Sclerosing adenosis

A

The correct response is Option B.

Patients in whom an otherwise benign breast biopsy shows atypical hyperplasia have a 4.5- to 5-fold increased risk for developing breast cancer. Proliferative breast disease without atypia increases the risk 1.5- to 2-fold. Oral contraceptives have shown to decrease the occurrence of all proliferative forms of benign breast disease without atypia, such as intraductal hyperplasia, intraductal papilloma, and sclerosing adenosis. Apocrine metaplasia is a non-proliferative histologic change and carries no increased risk for breast cancer. Of the choices, only atypical hyperplasia is increased with oral contraceptive use.

75
Q

A 42-year-old nurse is scheduled to undergo elective non–implant-based surgery of the left breast. Medical history includes no abnormalities, and she has no allergies. She smokes 1 pack of cigarettes daily. To decrease the incidence of surgical site infection, which of the following is the most effective perioperative strategy?

A) Administer cefazolin intravenously within 5 minutes of skin incision
B) Administer an insulin drip to keep blood glucose concentration less than 100 mg/dL
C) Have the patient practice complete smoking cessation for 7 days prior to surgery
D) Prescribe nasal mupirocin and chlorhexidine baths for 5 days prior to surgery
E) Use povidone-iodine skin preparation rather than chlorhexidine/isopropyl alcohol

A

The correct response is Option D.

A 2010 randomized controlled trial of over 6700 patients published in the New England Journal of Medicine documented a nearly 60% decrease in Staphylococcus aureus infections in patients if mupirocin was applied twice a day to the nares as well as a full-body wash with chlorhexidine for 5 days prior to surgery. The mean hospital stay was already reduced almost 2 days. A meta-analysis also demonstrated the same findings, with a nearly 45% decrease in surgical site infections (SSIs).

Data would suggest that isopropyl alcohol–containing skin preparations for surgery decrease SSI rates more effectively than povidone-iodine alone.

Perioperative antibiotics should be administered with enough advance time to achieve proper and adequate rates of skin penetration. With cefazolin, this is 30 to 59 minutes before skin incision, with an odds ratio of 1.0 (vs. 2.0 if given within 30 minutes, and 1.7 if given after 60 minutes). Five minutes prior to skin incision is not sufficient for the SSI-reducing effect to be achieved.

Blood glucose control is critical to decreasing SSI rates, with optimal rates usually being quoted as less than 180 mg/dL. However, this patient is not diabetic; she is otherwise healthy, so an insulin drip would not be appropriate. In addition, hypoglycemia can also have detrimental physiologic effects and also should be avoided.

Smoking cessation decreases SSI rates if the patient does not smoke for 4 weeks before or after surgery. A 2012 systematic review and meta-analysis of nearly 500,000 patients demonstrated this. However, 7 days of smoking cessation is insufficient time in advance of surgery to obtain these statistically significant benefits.

76
Q

A 45-year-old woman comes to the office for consultation regarding severe breast asymmetry after undergoing lumpectomy and radiation therapy for ductal carcinoma of the right breast 5 years ago. Physical examination shows the radiated right breast is tight and retracted, and the left breast is ptotic. Left-sided mastopexy for symmetry and autologous fat grafting to the radiated breast are recommended. The patient asks if the procedure will be covered by insurance. Which of the following is the most appropriate response?

A) Fat grafting the right breast will be covered by insurance but the mastopexy will be considered cosmetic and will not be covered
B) Insurance companies rarely cover the cost of immediate breast reconstruction
C) Insurance may not cover the procedure since insurers are not mandated to pay for reconstruction of lumpectomy defects
D) Procedures for both sides will be covered by insurance
E) The left-sided mastopexy will likely be covered but the fat grafting will not be covered

A

The correct response is Option C.

The Women’s Health and Cancer Rights Act, signed into law in 1998, requires insurance plans to cover the cost of breast reconstruction after mastectomy. The law includes all stages of reconstruction as well as contralateral procedures to provide symmetry. An often misunderstood aspect of the Women’s Health and Cancer Rights Act is that it does not apply to individuals undergoing breast conservation therapy (lumpectomy with radiation). As rates of breast conservation therapy have continued to increase (60% of women with stage I cancers), so have significant lumpectomy defects associated with the untoward effects of radiation. In the clinical scenario presented, the patient should be informed that the corrective operation (fat grafting and mastopexy) might not be covered by her insurance company.

77
Q

A 35-year-old woman comes to the office to discuss a recent diagnosis of breast cancer. Recent mammography showed diffuse microcalcifications throughout the breast, and needle biopsy showed infiltrating ductal carcinoma. On physical examination, some retraction of the skin in the lower outer quadrant of the breast is noted. She wears a size 36C brassiere. The patient reports that she is currently considering whether to have lumpectomy and radiation therapy or mastectomy. Which of the following features of this clinical scenario is a CONTRAINDICATION to breast conservation therapy?

A) Breast size
B) Mammographic findings
C) Patient age
D) Skin retraction on physical examination of the breast
E) Tumor pathology

A

The correct response is Option B.

Breast conservation therapy (BCT) refers to breast conserving therapy followed by moderate-dose radiation to eradicate microscopic residual disease. The goal is to provide the equivalent survival of mastectomy while maintaining a cosmetically acceptable appearance with a low rate of recurrence.

When considering breast conservation therapy or mastectomy, the needs and desires of each patient should be addressed. Age alone is not a contraindication to BCT, but overall health and comorbidities should be considered. Histologic subtype and pathology are not contraindications to BCT as long as the tumor is not diffuse and can be safely excised with negative margins. Similarly, breast size needs to be considered along with tumor size and location, but “small” or “large” breasts are not indications or contraindications. While retraction of the skin, nipple, or breast parenchyma is not an absolute contraindication to BCT, as long as negative margins can be safely removed, the cosmetic impact of their involvement should be considered.

There are few absolute contraindications to BCT, but they include:

Multicentric disease with two or more tumors in separate quadrants of the breast such that they cannot be encompassed in a single excision

Diffuse malignant microcalcifications on mammography

A history of prior radiation in the same breast or chest wall

Pregnancy

Persistently positive margins despite re-excision

78
Q

A 52-year-old woman undergoes immediate unilateral breast reconstruction with a free deep inferior epigastric flap. Near completion of the procedure, the flap skin paddle is noted to have venous congestion. On exploration of the pedicle, the anastomosis between the vena comitans and the internal mammary vessel appears patent. Which of the following preventive measures would most likely have averted this issue?

A) Administration of heparin immediately before release of vessel clamps and flap revascularization
B) Anastomosis of the veins using sutures instead of a venous coupling device
C) Preservation and anastomosis of the superficial inferior epigastric vein
D) Routine anastomosis of two venae comitantes per flap
E) Use of near-infrared fluorescence imaging to assess flap blood flow

A

The correct response is Option C.

Preservation of the superficial inferior epigastric veins (SIEV) during flap harvest is a useful preventive measure in microsurgical free tissue transfer operations. These veins can serve as important lifeboats to augment venous outflow in the setting of venous congestion. Typically, if a free flap demonstrates venous congestion, the inset should be taken down and the pedicle, recipient vessels, and anastomoses interrogated. Simple issues, such as mechanical compression or twisting of the vein should be ruled out. Next, the SIEV should be inspected. If it is engorged, it is likely that the flap is reliant on superficial outflow, and this vein should be connected to a recipient vessel to augment the venous outflow of the flap. Options for recipient veins include an anterograde branch on the pedicle vena comitans, or in a retrograde fashion to the vena comitans that was not used in the initial set of anastomoses.

Near-infrared fluorescence imaging technology can assist with flap design and may be useful for assessing the arterial inflow of the flap, but it has not been shown to correlate with flap loss or venous complications.

The use of one or two veins in microsurgical free tissue transfer is a topic that has been debated for several years. While some studies indicate that the use of two venous connections may decrease the velocity of blood flow across the anatomosis, there are no data to support differences in flap outcomes or thrombotic events. Therefore, the routine use of a second vein is largely up to surgeon preference.

Heparin may be a useful adjunct when thrombosis of the arterial or venous anastomosis is excised and a revision of the anastomosis is performed. Without evidence of thrombosis, it is unlikely to have any added benefit.

For venous anastomoses, the use of venous coupling devices has not been associated with patency issues or increased thrombosis rates. Therefore, a hand-sewn anastomosis is not likely to prevent the issue presented in this question.

79
Q

A 36-year-old woman with genetic susceptibility to breast cancer is scheduled to undergo bilateral prophylactic mastectomy. She has elected to proceed with immediate single-stage reconstruction using permanent silicone implants and acellular dermal matrix. Which of the following is the most sensitive method to detect implant rupture in this patient?

A) CT scan
B) Mammography
C) MRI
D) Physical examination
E) Ultrasonography

A

The correct response is Option C.

Implant rupture is one of the most common reasons for implant removal. While implant rupture can be associated with symptoms such as capsular contracture, it is often completely asymptomatic or “silent.” For this reason, screening imaging to detect such ruptures has been recommended.

According to the 2011 Update on the Safety of Silicone Gel-Filled Breast Implants published by the U.S. Food and Drug Administration, it is currently recommended that women with silicone implants get their first MRI 3 years after they receive the implants and every 2 years thereafter to detect silent ruptures.

Since the rate of rupture increases the longer an implant is in place, not screening is unacceptable.

Mammograms can detect extracapsular silicone when an implant ruptures, but they do not detect intracapsular ruptures.

The accuracy of ultrasound largely depends on the skill of the ultrasound technologist, the type of equipment used, and the experience of the interpreting physician. Furthermore, ultrasound is limited in its ability to detect ruptures in the back wall of the implant and in the breast tissue behind it.

CT scans can detect intracapsular silicone gel-filled breast implant rupture, but they are limited in their ability to detect extracapsular ruptures.

80
Q

A 53-year-old woman comes to discuss breast reconstruction after undergoing left mastectomy for ductal carcinoma in situ. She does not require chemotherapy or radiation therapy and does not want surgery on the unaffected breast. She is obese but otherwise healthy with a large, ptotic right breast. Which of the following breast reconstruction techniques is most likely to result in the greatest long-term patient satisfaction?

A) Autologous breast reconstruction with microvascular free tissue transfer from the abdomen
B) Extended latissimus dorsi musculocutaneous flap reconstruction without an implant
C) Immediate reconstruction with cohesive, anatomically shaped silicone gel implant
D) Immediate tissue expander followed by implant-based reconstruction
E) Latissimus dorsi musculocutaneous flap reconstruction with immediate placement of a permanent implant

A

The correct response is Option A.

Multiple studies have supported improved patient satisfaction with autologous breast reconstruction in the setting of a unilateral reconstruction. The clinical scenario involves an obese patient with a large, ptotic breast who does not desire a surgical procedure for symmetry and is the ideal candidate for an autologous reconstruction. Microvascular free tissue transfer of abdominal tissue for breast reconstruction demonstrates improved reliability and decreased fat necrosis compared with pedicled flap reconstruction. Implant-based reconstruction is less likely to provide adequate symmetry in this patient. An extended latissimus dorsi musculocutaneous flap–based reconstruction alone is unlikely to provide enough volume for symmetry and a traditional latissimus flap with implant is unlikely to provide adequate ptosis for symmetry.

81
Q

A 43-year-old woman would like to discuss plans for breast reconstruction after her upcoming unilateral mastectomy. Postoperative radiation therapy is planned. Which of the following is the most likely benefit of tissue expander–based breast reconstruction compared with immediate autologous breast reconstruction using this patient’s abdominal tissue?

A) Better symmetry
B) Improved postoperative sensation
C) A larger, more ptotic breast reconstruction
D) Lower risk of complications from radiation
E) Preservation of the patient’s options for final reconstruction

A

The correct response is Option E.

Immediate breast reconstruction with tissue expanders followed by reconstruction of choice preserves the patient’s skin envelope and keeps open options for definitive final reconstruction of choice whether with an implant or autologous tissue.

Tissue expander-based reconstruction is associated with a higher complication rate in the setting of radiation therapy but preserves abdominal and back tissue as options for autologous reconstruction. Implant-based reconstruction does not provide the advantages of improved symmetry, sensation, or breast ptosis.

82
Q

A 62-year-old woman undergoes breast reconstruction using autologous tissue from the abdomen. Intraoperatively, use of a perforator flap is found to be impossible because of multiple small nondominant perforators. Conversion to a delayed pedicled transverse rectus abdominis muscle flap is planned. Ligating which of the following vessels in this stage will best facilitate future viability of the tissue transferred in the next stage?

A) Deep inferior epigastric
B) Hypogastric
C) Internal mammary
D) Superficial inferior epigastric
E) Superior epigastric

A

The correct response is Option A.

Pedicled transverse rectus abdominis muscle (TRAM) flaps are based on the superior epigastric system, which is often less robust than the deep inferior epigastric system. Therefore, surgical delay by ligation of the deep inferior epigastric system may facilitate overall viability of the transferred tissue. Ligation of the superior epigastric system would make a pedicled TRAM flap unlikely to survive. The internal mammary ligation may also interrupt blood supply to the superior epigastric system, and even if the tissue is fed through collaterals, it would not strengthen the flap. Division of the superficial inferior epigastric system might also help, but it is not as critical as ligation of the deep inferior epigastric system. The hypogastric system does not have a direct impact on the pedicled TRAM tissues.

83
Q

A 35-year-old BRCA-positive woman undergoes bilateral prophylactic nipple-sparing mastectomy via lateral radial incisions with periareolar extensions. She undergoes direct-to-implant reconstruction with an acellular dermal matrix inferior lateral sling. Mastectomy specimens weigh 435 g each, and placement of 450 mL smooth, round, high-profile implants is performed. Postoperatively, 25% partial nipple areolar complex necrosis in both breasts is noted. Which of the following most likely contributed to the necrosis?

A) Bilateral surgery
B) Mastectomy incision
C) Use of acellular dermal matrix
D) Volume of the implants
E) Weight of the mastectomy specimen

A

The correct response is Option B.

Nipple-sparing mastectomies are becoming more prevalent with the rise of prophylactic mastectomies and increased comfort with performing it in presence of in situ and invasive carcinoma. While universal standards for patient selection do not exist, there is consensus that general recommendations include no inflammatory breast cancers, no pathologic nipple discharge, no Paget’s disease, and for many, a 2-cm distance from the tumor to the nipple. Nipple-sparing mastectomy incisions have been described as radial lateral, lateral inframammary fold, and peri-areolar. Because the entire skin envelope is preserved, direct-to-implant reconstruction is possible with inferolateral support from acellular dermal matrices or other scaffolds. Areolar involvement in mastectomy incisions is associated with increased rates of nipple necrosis. The weight of mastectomy specimen, volume of implants, use of acellular dermal matrices or laterality of surgery have not been shown to be associated with nipple necrosis.

84
Q

A 45-year-old woman undergoes bilateral nipple-sparing mastectomy with immediate tissue expander reconstruction for T3N2 breast cancer. She completes radiotherapy and resumes expansion in the office 4 months after surgery. Her husband calls the office noting that the patient is reporting headache and requests additional pain medication when she suddenly demonstrates seizure activity. She is taken to the emergency department for immediate evaluation and stabilization. Which of the following imaging modalities should be used with caution for this patient?

A) CT scan
B) Fluoroscopy
C) MRI
D) Positron emission study
E) Ultrasound

A

The correct response is Option C.

The majority of breast tissue expanders include a ferromagnetic port and are currently labeled as MRI-unsafe due to the potential interaction with the magnetic field of the machine. There are reports of several MRI-related complications: malposition, pain or burning sensation, polarity reversal, port dislodgement, and thermal injuries. However, several subsequent studies have demonstrated modifications to minimize risks in patients requiring diagnostic MRI of either the contralateral breast, perforator mapping, or evaluation of brain/spine metastases. This patient may need MRI evaluation of the brain to diagnose metastatic disease of the brain. While some imaging technicians may be unwilling to perform MRI due to the warnings, studies suggest use of a 1.5T MRI, saline filling, and prone positioning are effective strategies to minimize risks and safely perform MRI rather than explant the device(s).

CT scanning can be safely performed with no contraindication due to expanders. Positron emission studies also do not interfere with expanders. Ultrasound is frequently used to safely assess and treat possible fluid collections or infections associated with expanders. Despite her previous radiation therapy, fluoroscopy can still be safely performed while taking care to minimize radiation exposure to critical structures.

85
Q

An otherwise healthy 38-year-old woman who is a smoker is considering implant-based breast reconstruction following mastectomy. She has been counseled about likely use of acellular dermal matrix when the intermediate tissue expander is placed and wants to further understand why the matrix will be used in her body. The patient should be advised that use of acellular dermal matrix is associated with a decreased risk for which of the following?

A) Cancer recurrence
B) Capsular contracture
C) Seroma
D) Skin flap necrosis
E) Smoking-related complications

A

The correct response is Option B.

Acellular dermal matrix (ADM) use has been increasing over time with tissue expander or implant-based breast reconstruction following mastectomies. Many potential advantages and disadvantages have been studied, and some of the data are contradictory. However, the consensus of the literature indicates that ADMs are associated with decreased capsular contracture rates. There is literature to suggest that seroma rates are increased or remain stable, not decreased, with ADMs. ADMs have not been shown to decrease independent patient risk factors for complications such as tobacco use or to decrease cancer recurrence rates. ADMs also do not appear to improve vascularity of the tissue overlying them when initially placed.

86
Q

A 45-year-old woman undergoes mastectomy. Which of the following anatomical landmarks denotes a limit of the breast and, therefore, a limit to the extent of the mastectomy?

A) Anterior border of latissimus dorsi muscle
B) Inferior origin of pectoralis major muscle
C) Lateral pectoralis minor muscle
D) Superficial fascia of the serratus anterior muscle
E) Supraclavicular lymph node basin

A

The correct response is Option A.

The borders of the breast and, therefore, the limits of the mastectomy are the sternum, clavicle, inframammary fold, and anterior border of the latissimus dorsi muscle. The other options are not designated as borders of the breast tissue.

87
Q

A 57-year-old woman with a history of modified left radical mastectomy followed by adjuvant chemotherapy and radiation therapy is evaluated six months after her last radiation treatment for breast reconstruction. Physical examination shows slight skin redundancy of the chest wall and grade 2 ptosis of the contralateral breast. She wears a size 36D brassiere and does not want to undergo symmetry procedures on the contralateral breast. Which of the following reconstructive options is most likely to achieve breast symmetry in this patient?

A) Deep inferior epigastric perforator flap
B) Latissimus dorsi flap
C) Thoracodorsal artery perforator flap
D) Tissue expander with acellular dermal matrix and staged implant placement
E) Total submuscular tissue expander and staged implant placement

A

The correct response is Option A.

Breast reconstruction requires reconstruction of both the soft-tissue envelope and volume. In the setting of radiated chest wall tissues, it would be difficult to recreate a breast envelope of sufficient size to match the contralateral breast with tissue expansion, even with acellular dermal matrix or total submuscular placement of a tissue expander. The history of radiation would likely cause an element of capsular contracture and decrease symmetry to the contralateral breast with grade 2 ptosis. Both the latissimus dorsi flap and the thoracodorsal artery perforator flap would recruit non-radiated skin to create an adequate envelope, but would not provide adequate volume alone. The deep inferior epigastric perforator (DIEP) flap would provide both envelope and volume.

88
Q

A 53-year-old woman with a BMI of 27 kg/m2, gigantomastia, and grade III ptosis is considering unilateral mastectomy and autologous reconstruction with abdominal tissue. Which of the following is the most appropriate advice regarding the contralateral breast?

A) A contralateral prophylactic mastectomy should be performed to maximize symmetry
B) Contralateral reduction should be accompanied by placement of an implant on that side to address upper pole symmetry with the autologous reconstruction
C) Simultaneous contralateral reduction may be performed with an acceptable risk profile
D) Symmetry is unlikely to be achieved, so a contralateral matching procedure would not be recommended
E) Symmetry with autologous tissue is likely to be achieved without the contralateral matching procedure

A

The correct response is Option C.

Studies have shown that contralateral symmetry procedures performed synchronously with unilateral autologous tissue reconstruction after mastectomy (including reconstructions with a free perforator flap) are acceptable. Situations requiring reduction rather than augmentation or mastopexy seem to be the most suited to this timing. A patient with a lower BMI and gigantomastia is unlikely to achieve symmetry without a contralateral procedure. Autologous tissue reconstruction likely would not need a contralateral implant for upper pole symmetry. An attempt at improvement in symmetry using contralateral surgery would be acceptable should the patient so choose, but recommending a prophylactic mastectomy on the contralateral side solely for symmetry and not for risk reduction may be overly aggressive when other methods such as reduction mammaplasty exist with a likelihood of acceptable postoperative symmetry.

89
Q

Which of the following is the most accurate location of the elliptical skin island of a profunda artery perforator (PAP) flap?

A) Anteromedial thigh with the superior border within the groin crease
B) Inferior buttock with the inferior border within the gluteal fold
C) Lateral hip superior to the iliac crest
D) Middle buttock, from the posterior superior iliac spine to the apex of the greater trochanter
E) Posteromedial thigh with the superior border within the gluteal fold

A

The correct response is Option E.

According to the literature, the skin island of the profunda artery perforator (PAP) flap is inferior to the gluteal fold.1,2 An ellipse of the inferior buttock with the inferior border within the gluteal fold describes the skin island of the inferior gluteal artery perforator (IGAP) free flap.3 An ellipse of the anteromedial thigh with the superior border within the gluteal fold describes the transverse upper gracilis (TUG) flap. An ellipse of the middle buttock, from the posterior superior iliac spine to the apex of the greater trochanter, describes the superior gluteal artery perforator (SGAP) flap.3 An ellipse of the lateral hip superior to the iliac crest describes the Rubens or lateral hip flap.4 The only option that correctly identifies the skin island for the PAP flap is an ellipse of the posteromedial thigh with the superior border within the gluteal fold. The superior marking is within or just below the gluteal fold and the inferior marking is roughly 7 cm below the superior marking. The flap is an ellipse so the scar does not extend outside of the gluteal fold.1,2

90
Q

Nipple-sparing mastectomy is CONTRAINDICATED in which of the following women considering mastectomy and immediate breast reconstruction?

A) A 21-year-old with BRCA1 mutation and grade I breast ptosis
B) A 31-year-old with BRCA2 mutation; subglandular augmentation mammaplasty 1 year ago
C) A 45-year-old with unilateral breast cancer and need for postoperative radiation therapy
D) A 50-year-old with unilateral breast cancer in the tail of Spence
E) A 61-year-old with unilateral breast cancer; tumor-to-nipple distance of 1 cm

A

The correct response is Option E.

Nipple-sparing mastectomy is increasingly prevalent owing to perceived improvement in reconstructive outcome and patient satisfaction. Prevalence of nipple ischemia, which can vary in severity from incomplete, partial-thickness epidermolysis to total nipple loss, ranges from 2 to 60% depending on the definition used. A recent pooled analysis found a cumulative prevalence of 7%. Most cases respond to local wound care; reoperation for total nipple loss is relatively infrequent.

Patient selection remains paramount to successful nipple preservation. Typically, thin, non-smoking patients with small, non-ptotic breasts are considered ideal candidates. Severe macromastia and ptosis may not only increase risk of poor nipple vascularity, but also contribute to nipple malposition. Oncologic determinants of the safety of nipple preservation should also be considered. Noninflammatory cancers located in excess of 2 cm from the nipple can generally be safely extirpated without removal of the nipple. Usually, an intraoperative frozen section biopsy of the retroareolar tissue (“doughnut”) is performed after excision of the main mastectomy specimen to demonstrate nipple margins free of tumor. Patients undergoing prophylactic mastectomy in the setting of high genetic predisposition are also considered good candidates. Successful nipple preservation has also been described in patients with a history of reduction mammaplasty or mastopexy.

Successful nipple-sparing mastectomy with implant reconstruction has been described recently in patients who require postoperative radiation, albeit with higher risk of capsular contracture and nipple malposition. Prior augmentation in the subglandular or subpectoral positions does not contraindicate nipple-sparing mastectomy; rather, the latter group may present opportunities for direct-to-implant reconstruction. Location of a breast cancer in the tail of Spence (axillary extension of breast tissue) should not contraindicate nipple-sparing mastectomy, as it is likely to be far enough from the nipple to safely spare it.

91
Q

A 62-year-old woman with a history of Stage III breast cancer is scheduled for delayed autologous breast reconstruction from the abdominal donor site. She has no other medical problems. BMI is 30 kg/m2. Her mother had a lower extremity deep venous thrombosis in the past. Caprini risk assessment score is 9. Which of the following is the most appropriate method of postoperative VTE risk reduction?

A) Aspirin therapy
B) Early ambulation after surgery
C) Low-molecular-weight heparin therapy
D) Sequential compression device use
E) No VTE prevention is indicated

A

The correct response is Option C.

Venous thromboembolism (VTE) is a disorder with short-term mortality and long-term morbidity. Plastic and reconstructive surgery patients are known to be at high risk for VTE after surgery. Symptomatic VTE occurs with high frequency after post-bariatric body contouring (7.7%), abdominoplasty (5%), and breast or upper body contouring (2.9%). To fully identify VTE risk in surgical patients, individualized patient assessment is advocated. The Caprini risk assessment model (RAM) is a useful and effective tool to stratify surgical patients for VTE risk. For patients with high Caprini scores, a significantly greater likelihood of VTE events is observed. Approximately 11% of patients with Caprini score >8 will have a VTE within 60 days after surgery.

Based upon recommendations from the ASPS VTE Task Force, patients undergoing elective plastic and reconstructive surgical procedures who have Caprini RAM score of 7 or more should have VTE risk reduction strategies employed, such as limiting operating room times, weight reduction, discontinuation of hormone replacement therapy, and early postoperative mobilization. Patients undergoing major plastic and reconstructive operative procedures performed during general anesthesia that last longer than 60 minutes should receive VTE prevention. For patients with Caprini score of 3 to 6, the use of postoperative low-molecular-weight heparin (LMWH) or unfractionated heparin (UH) should be considered. For patients with Caprini score of 3 or more, use of mechanical prophylaxis throughout the duration of chemical prophylaxis for non-ambulatory patients should be considered. For patients with Caprini score of 7 or more, the use of extended LMWH postoperative prophylaxis should be strongly considered.

Aspirin does not decrease the risk of VTE and may increase the risk of perioperative complications.

92
Q

A 42-year-old woman comes to the office for treatment after receiving a diagnosis of cancer of the right breast. She has decided to undergo mastectomy of the right breast. Which of the following is a relative CONTRAINDICATION to nipple-sparing mastectomy?

A) Comedo-type breast tumor
B) Invasive lobular carcinoma
C) Subareolar tumor
D) Tumor location 3 cm from the nipple
E) Tumor size of 2.5 cm

A

The correct response is Option C.

A relative contraindication to nipple-sparing mastectomy is a centrally located tumor. Although various authors have employed different distance criteria, it is generally accepted that patients whose tumors are within 2 cm of the nipple are not candidates for nipple-sparing mastectomy.

Nipple-sparing mastectomy is an appropriate option for high-risk patients undergoing prophylactic mastectomy and for patients diagnosed with breast cancer who meet certain criteria. Those criteria are: tumor size of 3 cm or less, at least 2 cm from the nipple, not multicentric, and with clinically negative nodes.

Comedo carcinoma of the breast is a type of ductal carcinoma in situ. It is considered to be an early stage of breast cancer, is confined to the ducts, and usually does not spread beyond. It is not a contraindication to nipple-sparing mastectomy.

Invasive lobular carcinoma originates from the breast lobules, may form a thickening of the breast tissue rather than a discrete mass, and is often bilateral. As long as it meets the above criteria, it is not a contraindication to nipple-sparing mastectomy.

Inflammatory breast cancer, Paget disease, and tumors infiltrating the skin are also not candidates for skin-sparing or nipple-sparing mastectomy, according to several authors.

In more recent studies, a tumor size of 3 cm or less appears to result in no increase in local or regional recurrence in nipple-sparing mastectomy compared with alternative surgical approaches. A tumor of 2.5 cm is not a contraindication to nipple-sparing mastectomy.

93
Q

Which of the following characteristics is correlated with increased risk of nipple-areola complex necrosis in nipple-sparing mastectomies with immediate reconstruction?

A) Autologous tissue reconstruction
B) Direct to implant reconstruction
C) Patient age
D) Periareolar incision
E) Small breast size

A

The correct response is Option D.

Nipple-sparing mastectomies (NSMs) are becoming more common for both therapeutic and prophylactic mastectomies. Nipple-areola complex (NAC) necrosis can imperil reconstructive efforts, as well as negatively affect patients emotionally. It is important to maximize perfusion to the mastectomy skin flaps and NAC while still performing an oncologically sound procedure. There are multiple different incisions for performing NSM. Periareolar, inframammary-fold, radial, and vertical incisions are the most common. Periareolar incisions are associated with an increased risk of NAC necrosis in NSMs. Type of reconstruction, small breast size, and patient age have not been shown to be linked to increased rates of NAC necrosis.

94
Q

A 52-year-old woman receives a diagnosis of invasive ductal carcinoma of the right breast. Which of the following details from this patient’s history is the strongest risk factor for this diagnosis?

A) Early first pregnancy (less than 30 years)
B) Early menarche (less than 12 years)
C) Early menopause (less than 55 years)
D) Multiple episodes of breast-feeding
E) Remote oral contraceptive use

A

The correct response is Option B.

Early menarche is the highest risk factor for breast cancer of the options listed. Late first pregnancy, late menopause, no breast-feeding, and recent oral contraceptive use are also risk factors for breast cancer but are not as high risk.

95
Q

A 55-year-old woman comes to the office for a second opinion because she is displeased with the results of a recent bilateral mastectomy and breast reconstruction with 800-mL high-profile silicone implants. A photograph is shown. BMI is 35 kg/m2. She repeatedly shows pictures of models with augmented breasts and says that she wants her breasts to be “perkier.” She requests augmentation/mastopexy. Which of the following is the most appropriate next step in management?

A) Augmentation/mastopexy
B) Implant exchange
C) Mastopexy
D) Reassurance
E) Referral to a psychiatrist

A

The correct response is Option D.

The most reasonable approach in this patient is to offer reassurance and reset her expectations. A patient with a BMI of 35 kg/m2 who undergoes mastectomy and implant reconstruction will never look like a model with augmented breasts. This patient clearly has misguided expectations. Any surgical intervention is unlikely to produce the result she is looking for, when in fact she has a very acceptable result as is. Referral of this patient to a psychiatrist will likely upset the patient and undermine her trust.

96
Q

A 42-year-old woman with a 3-cm invasive ductal carcinoma of the right breast is evaluated for breast reconstruction. She has not decided how she wants to manage her contralateral breast. Regarding eliciting a family history, which of the following cancers is associated with a mutation in a breast cancer-susceptibility gene?

A) Colon
B) Esophageal
C) Lung
D) Pancreatic
E) Thyroid

A

The correct response is Option D.

The breast cancer-susceptibility gene types 1 and 2 (BRCA1 and BRCA2) are tumor suppressor genes. Mutations in BRCA1and BRCA2 are associated with hereditary breast and ovarian cancers. Additionally, they can be associated with increased risks of pancreatic and prostate cancer. Thyroid, lung, esophageal, and colon cancer are not associated with increased risks of BRCA1 and BRCA2 mutations.

97
Q

A 35-year-old woman comes to the office for consultation regarding prophylactic mastectomy and breast reconstruction. The patient’s mother and sister were diagnosed with bilateral breast cancer in their premenopausal years. Genetic testing for BRCA mutations is negative. Which of the following best estimates this patient’s lifetime risk of breast cancer?

A) 5%
B) 13%
C) 20%
D) 45%
E) 80%

A

The correct response is Option D.

The cumulative lifetime risk for a 35-year-old woman whose mother and sister had breast cancer is estimated to be approximately 15%. The risk may increase to as high as 45% if those cancers were premenopausal and bilateral. BRCAhereditary cancer is characterized by autosomal dominant genetics with multiple family members in each generation being affected. For patients with BRCA1 mutation, the risk of breast cancer has been estimated to be between 50 and 80% by age 65 years. The risk of developing ovarian cancer has been estimated to be 10% by age 60 years.

98
Q

A 39-year-old woman with a history of fibrocystic breast lesions comes to the office for consultation. She has no family history of breast cancer. Results of routine mammograms have been negative; she has never undergone biopsy. Which of the following is the most appropriate recommendation for this patient regarding managing her risk of breast cancer?

A) Continue to schedule routine mammograms
B) Refer for mastectomy
C) Schedule core needle biopsy
D) Schedule fine-needle aspiration
E) Schedule genetic testing

A

The correct response is Option A.

Studies have shown the fibrocystic changes alone in the breast are not directly linked to an increased risk of breast cancer, so there is no indication for fine-needle aspiration or core biopsy. Cancer risk increases in benign breast disease with increased proliferation and atypical hyperplasia. Even with no family history of breast cancer, it is recommended that the patient continue routine mammograms. Fibrocystic breast disease has not been linked in the literature to an increased risk of mutations of the BRCA genes; therefore, genetic testing is unnecessary.

99
Q

An otherwise healthy 37-year-old woman presents for delayed microsurgical breast reconstruction. Which of the following is associated with use of tamoxifen?

A) Hemodynamic instability
B) Impaired wound healing
C) Increased bleeding
D) Seroma formation
E) Thromboembolic events

A

The correct response is Option E.

Breast cancers that are estrogen receptor positive may be responsive to adjuvant chemotherapy with selective estrogen receptor modulators such as tamoxifen, which can reduce recurrence and mortality. Tamoxifen is associated with thromboembolic events, such as deep venous thrombosis and pulmonary embolism. This prothrombotic effect has been postulated to be secondary to the effect of tamoxifen on estrogen receptors that are abundant within vascular endothelium.

Tamoxifen has been shown to be associated with increased rates of total flap loss and decreased rates of flap salvage when taken within 28 days of microsurgical breast reconstruction, which represents two half-lives of the active metabolite of tamoxifen (N-desmethyl tamoxifen, t1/2=14 days). It has therefore been recommended that in patients undergoing microsurgical breast reconstruction, tamoxifen be held for at least 28 days preoperatively. Some authors have further advised holding the medication postoperatively in addition to preoperatively.

Tamoxifen is not associated with impaired wound healing, increased bleeding, hemodynamic instability, or seroma formation.

100
Q

An otherwise healthy 38-year-old woman undergoes prophylactic bilateral mastectomy and immediate reconstruction with deep inferior epigastric artery perforator (DIEP) free flaps. Intraoperatively, the left DIEP flap appears congested before the conclusion of the case. The left deep inferior epigastric artery and vein (DIEA and DIEV) were anastomosed to the proximal internal mammary vessels. The vascular pedicle is evaluated and each anastomosis appears patent and not kinked; however, the venous congestion persists. Which of the following is the most appropriate management?

A) Anastomose the superficial inferior epigastric vein to an internal mammary vessel perforator
B) Convert to left prosthetic reconstruction
C) Infuse tissue plasminogen activator (tPA) to the DIEA
D) Initiate leech therapy
E) Revise the DIEV anastomosis to the retrograde internal mammary vessel limb

A

The correct response is Option A.

Venous drainage of the lower abdominal skin and subcutaneous tissue occurs primarily through the superficial venous system and secondarily through the deep venous system, with perforating veins interconnecting the two systems. These communicating veins have been identified on computed tomography angiography in approximately 90% of abdominal walls in vivo. The majority of the remaining 10% of patients likely have communicating veins that are too small to visualize or are absent. In these cases of anatomical superficial venous system dominance, venous drainage is dependent on the superficial venous system. A recently published 2012 article by Sbitany et al. demonstrated that the incidence of intraoperative venous congestion secondary to persistent superficial venous system dominance was 0.9% in 1201 muscle-sparing transverse rectus abdominis musculocutaneous and deep inferior epigastric artery perforator free flaps. A free flap that becomes congested after reperfusion in the operating room should be assessed immediately for possible etiologies including twisting, kinking, tension, or vasospasm of the vascular pedicle. If a technical problem is ruled out and the venous anastomosis remains patent, obligatory enhancement of venous drainage with the superficial venous system is necessary to salvage the free flap rather than revision of the original anastamosis. Various methods include an anastomosis of the superficial inferior epigastric vein (SIEV) to the DIEV system or any chest wall vein, including the retrograde limb of the internal mammary vessel, the branch of the internal mammary vessel, or the thoracodorsal system. This requires preemptive planning and sparing of the superficial epigastric vein or SIEV during the dissection of the flap. A vein graft can be utilized if additional length is necessary. Another option is to substitute the DIEV anastomosis with the SIEV.

Tissue plasminogen activator (tPA) would not be indicated in this scenario, as it is used as a thrombolytic and there is no evidence of vascular thrombosis. Revising the DIEV anastomosis would be moot because it is patent and the deep system is being drained. Leech therapy is useful for venous congestion, but primarily as an adjunct after potential surgical etiologies have been addressed. Sacrificing the free flap without first attempting salvage is not warranted, and using a prosthetic would be possible only if prior patient consent were obtained.

101
Q

A 60-year-old woman with breast cancer undergoes a transverse rectus abdominis musculocutaneous flap breast reconstruction after mastectomy. She has no allergies. Weight is 200 lb (91 kg). Estimated blood loss is 200 mL. Duration of the operation is 3 hours and 50 minutes. Administration of cefazolin before skin incision is planned as prophylaxis against surgical site infection. Which of the following is the most appropriate dosage and timing of this injection?

DoseTimingRedosing

A)1g5 minutes priorafter 150 mL of blood loss

B)1g15 minutes priorno

C)1g40 minutes priorno

D)2g15 minutes priorno

E)2g40 minutes priorno

A

The correct response is Option E.

There has been a renewed interest in perioperative antibiotics in recent years toward more appropriate use to decrease surgical site infection (SSI) while decreasing the incidence of resistant bacteria.

Current recommendations are to administer a single perioperative dose of antibiotics against common skin flora (gram positive), usually using a first-generation cephalosporin. However, the following recommendations may be underappreciated:

Cefazolin intravenous:1 g if <80kg; 2g if >80kg

Alternatives:clindamycin 600 to 900mg intravenously;
vancomycin 1 to 1.5g intravenously

In this case, the patient weighs 200lb (91kg), so 2g of cefazolin is the recommended dosage.

Additionally, an important factor is the timing of the administration of the perioperative antibiotics in order to achieve proper skin levels before incision. In one study by Classen et al., published in the New England Journal of Medicine in 1992, the optimal time was between 2 hours before the operation and skin incision, as greater than 2 hours before and any time after skin incision led to marked increases in the relative risk of SSIs. A follow-up by Weber et al. in 2008 narrowed the most appropriate window to between 30 and 59 minutes before skin incision.

Finally, there is the issue of redosing. Current recommendations are to redose if there is excessive blood loss (>1500 mL) or if there are long procedures where one exceeds the half-life of the antibiotic used.

In the clinical scenario described, the most appropriate choice is 2 g of cefazolin, because the patient’s weight is above 80 kg, administered between 30 to 59 minutes before skin incision. Redosing on the basis of blood loss is unnecessary, although one could consider redosing at approximately 4 hours on the basis of half-life.

102
Q

A 42-year-old woman who underwent mastectomy of the right breast 6 months ago is evaluated for delayed autologous breast reconstruction with free tissue transfer. Which of the following medications should be discontinued preoperatively if she is routinely taking it?

A) Diltiazem
B) Fluoxetine
C) Metoprolol
D) Multivitamin
E) Tamoxifen

A

The correct response is Option E.

It is well known that tamoxifen can increase the risk of thrombembolic events. In a retrospective study at MD Anderson Cancer Center, it was shown that patients who received the drug close to the procedure had a significantly higher rate of complications. It is recommended that the patient stop tamoxifen at least 28 days before surgery.

Other listed medications do not have a direct effect on thrombotic complications.

103
Q

When performing immediate breast reconstruction, it is important to reconstruct the lateral inframammary fold. This is because the oncologic extirpation of the breast is carried out to which of the following anatomic landmarks?

A) Anterior edge of the latissimus dorsi muscle
B) Anterior edge of the serratus muscle
C) Lateral edge of the pectoralis major muscle
D) Lateral edge of the pectoralis minor muscle
E) Posterior edge of the serratus muscle

A

The correct response is Option A.

For modified radical and simple mastectomies, the landmarks of dissection are: superiorly to the clavicle, medially to the sternum, inferiorly to the inframammary fold, and laterally to the border of the latissimus dorsi muscle. The pectoralis major muscle fascia is resected with the specimen.

The recreation of the inframammary fold is important for shaping in breast reconstruction and care must be taken to evaluate and repair both the inferior and lateral components of the inframammary fold.

104
Q

A 52-year-old woman is evaluated for breast reconstruction after modified radical mastectomy with adjuvant chemotherapy and radiation therapy 18 months ago. Her last radiation treatment was 8 months ago. BMI is 29 kg/m2. Examination today shows hyperpigmentation of the right chest wall with no redundancy of the mastectomy skin flaps. Her contralateral breast is a D cup with grade III ptosis. Which of the following methods will create the best symmetry for this patient?

A) Deep inferior epigastric perforator flap
B) Gel breast implant and acellular dermal matrix
C) Latissimus dorsi musculocutaneous flap
D) Tissue expander and acellular dermal matrix placement with planned staged exchange for permanent gel implant
E) Tissue expander placement alone with planned staged exchange for permanent gel implant

A

The correct response is Option A.

The deep inferior epigastric perforator flap would give the patient autologous tissue reconstruction with ample tissue for skin resurfacing and soft tissue for volume. In this radiated patient with a tight skin envelope, a tissue expander/implant, with or without acellular dermal matrix, would be difficult to create an appropriately ample skin envelope and the patient would be at higher risk for wound-healing problems and capsular contracture. The latissimus dorsi musculocutaneous flap, although an autologous tissue reconstruction, would have insufficient volume to adequately match this patient’s contralateral side. It would have to be combined with an implant.

105
Q

An otherwise healthy 67-year-old woman with advanced breast cancer is scheduled to undergo mastectomy and immediate reconstruction with a free transverse rectus abdominis musculocutaneous (TRAM) flap. BMI is 35 kg/m2. Which of the following is most appropriate for deep venous thrombosis prophylaxis?

A) Aspirin and intermittent pneumatic compression stockings
B) Elastic compression stockings only
C) Intermittent pneumatic compression stockings only
D) Low-molecular-weight heparin and intermittent pneumatic compression stockings
E) Positioning and early ambulation

A

The correct response is Option D.

This patient is at highest risk for deep venous thrombosis and she will require combination therapy of compression stockings and chemical prophylaxis.

Perhaps the most well-regarded set of guidelines in this matter comes from the American College of Chest Physicians. These guidelines provide treatment recommendations based on a patient’s risk classification. Caprini model offers a very user-friendly method of calculating patient risk factors and categorizing them into low, moderate, high, and highest risks.

The patient in question has risk factors (age, malignancy, and major surgery) that would put her in the highest risk category. In the highest risk category, prophylaxis will require combination therapy or warfarin.

106
Q

A 45-year-old woman comes to the office for consultation regarding mastectomy and immediate breast reconstruction because of recurrent right-sided breast cancer. She underwent breast-conserving therapy and radiation therapy 5 years ago. BMI is 23 kg/m2. The patient is otherwise healthy, and she does not smoke cigarettes. On examination, the breast is soft with obvious fibrosis. Use of which of the following has the highest risk for reconstructive failure in this patient?

A) Deep inferior epigastric artery perforator (DIEP) flap
B) Free transverse rectus abdominis musculocutaneous (TRAM) flap
C) Latissimus flap with prosthesis
D) Pedicled TRAM flap
E) Tissue expander and prosthesis

A

The correct response is Option E.

Pre-reconstruction radiation is typically seen in two groups of patients: those who underwent mastectomy without reconstruction followed by radiation therapy, or those who underwent breast-conserving therapy and radiation with recurrence or new cancer. Although these patients may present with a reasonable skin envelope, complication rates associated with tissue expander/prosthesis have been reported as high as 70%, with a 40% rate of failure or conversion to flap. Additionally, aesthetic outcomes in most patients who completed expander/prosthesis reconstruction after previous radiation therapy were deemed good or very good compared with the majority of non-irradiated patients who achieved excellent results. Tissue expander/prosthesis can be considered as an option for patients with a history of previous irradiation who wish to avoid the scars and recovery of flap-type reconstructions; however, they should be counseled of the high risks of complications.

107
Q

A 34-year-old woman is scheduled to undergo bilateral prophylactic mastectomy because of a strong family history of breast cancer. Reconstruction with free flaps from the upper inner thigh is planned. Which of the following is the pedicle to these flaps?

A) Ascending branch of the lateral femoral circumflex artery
B) Descending branch of the lateral femoral circumflex artery
C) Medial femoral circumflex artery
D) Superficial circumflex iliac artery
E) Superficial femoral artery

A

The correct response is Option C.

The transverse upper gracilis (TUG) flap is a free flap that can be used in breast reconstruction. It is particularly useful in cases where the abdominal donor site is not available or not desired. The flap incorporates the skin and subcutaneous tissues of the upper inner thigh, in the region of the medial thigh lift. The TUG flap is based on the gracilis vessels and perforators from the descending branch of the medial femoral circumflex artery, which forms the dominant pedicle. This arises from the profunda femoris artery. Modifications of this flap have been proposed to increase the volume available for free tissue transfer.

The ascending branch of the lateral femoral circumflex artery provides blood supply to the tensor fascia lata flap.

The descending branch of the lateral femoral circumflex artery forms the pedicle for the anterolateral thigh flap.

The superficial circumflex iliac artery forms the pedicle for the groin flap.

Branches from the superficial femoral artery provide minor pedicles to the distal portion of the gracilis muscle, but are less important and distal to the area of the TUG flap.

108
Q

A 41-year-old woman comes to the office because of an invasive ductal carcinoma of the left breast. On mammography, the tumor is 3 cm from the nipple and measures 4 cm. A left-sided lateral periareolar scar extending from the 12 o’clock to the 3 o’clock position from a previous biopsy is noted. The patient wishes to undergo a nipple-sparing mastectomy. Which of the following findings places the patient at greatest oncologic risk, including risk for de novo or recurrent cancer or inadequate surgical margins, with this procedure?

A) Distance of tumor to nipple
B) Patient age
C) Presence of the periareolar scar
D) Size of tumor
E) Type of tumor

A

The correct response is Option D.

As surgical approaches to breast cancer treatment have evolved, nipple-sparing mastectomy (NSM) has emerged as an alternative to other approaches. It was initially used for prophylactic mastectomies, and patients reported increased satisfaction and body image with nipple-areola complex (NAC) preservation. The role of NSM has been expanded to therapeutic mastectomy, and with that there has been increased research in the oncologic safety of this approach.

Studies have evaluated therapeutic NSM in the context of invasive ductal carcinoma, invasive lobular carcinoma, and ductal carcinoma in situ. The type of cancer does not appear to be associated with the oncologic safety of NSM. Several studies have demonstrated an inverse association between NAC involvement and distance of the tumor from the nipple. While these studies have varied in their distance cutoffs, nipple involvement is reported to be over 50% when the tumor-nipple distance is less than 2 cm, as noted in one study. There is a direct correlation between tumor size and NAC involvement—the same study cited data that when the tumor was greater than 4 cm, the likelihood of nipple involvement was greater than 50%.

One published screening algorithm for plastic surgery includes tumor size less than 3 cm, and tumor location greater than 2 cm from the nipple as criteria for NSM candidacy.

A periareolar scar, if large, may compromise the blood supply to the NAC. Acceptable incisions for NSM, however, include a periareolar incision of 25 to 50%.

109
Q

A 26-year-old woman with a strong family history of breast cancer undergoes genetic testing. She is found to have a deleterious mutation of the BRCA1 gene. Which of the following best describes her lifetime risk for the development of breast cancer when compared with women without this mutation?

A) Two times greater
B) Six times greater
C) Ten times greater
D) Fifteen times greater
E) Twenty times greater

A

The correct response is Option B.

A woman’s lifetime risk of developing breast and/or ovarian cancer is greatly increased if she inherits an altered BRCA1 or BRCA2 gene. Women with an inherited alteration in one of these genes have an increased risk of developing these cancers at a young age (before menopause) and often have multiple close family members with the disease. These women may also have an increased chance of developing colon cancer.

Men with an altered BRCA1 or BRCA2 gene also have an increased risk of breast cancer (primarily if the alteration is in BRCA2) and possibly prostate cancer. Alterations in the BRCA2 gene have also been associated with an increased risk of lymphoma, melanoma, and cancers of the pancreas, gallbladder, bile duct, and stomach in some men and women.

According to estimates of lifetime risk, approximately 13.2% (132 of 1000 individuals) of women in the general population will develop breast cancer, compared with estimates of 36 to 85% (360 to 850 of 1000) of women with an altered BRCA1 or BRCA2 gene. In other words, women with an altered BRCA1 or BRCA2 gene are 3 to 7 times more likely to develop breast cancer than women without alterations in those genes. Lifetime risk estimates of ovarian cancer for women in the general population indicate that 1.7% (17 of 1000) will get ovarian cancer, compared with 16 to 60% (160 to 600 of 1000) of women with altered BRCA1 or BRCA2 genes. No data are available from long-term studies of the general population comparing the cancer risk in women who have a BRCA1 or BRCA2 alteration with women who do not have an alteration in these genes. Therefore, these figures are estimated ranges that may change as more research data are added.

110
Q

A 48-year-old woman undergoes radical resection of the left breast, left hemisternectomy, four-rib resection, and visceral pleurectomy because of invasive inflammatory breast cancer extending through the thoracic wall into the mediastinum and chest. Examination of specimens obtained from intraoperative frozen section biopsies shows no residual disease. Postoperative adjuvant chemotherapy and radiation therapy to the area are planned. An intraoperative photograph of the 17 × 17-cm defect is shown. Which of the following is the most appropriate technique for soft-tissue reconstruction?

A) Left latissimus muscle flap with skin graft

B) Omental flap with skin graft

C) Left rectus turnover flap with skin graft

D) Reverse abdominoplasty flap

E) Right TRAM flap

A

The correct response is Option E.

The most appropriate option for this patient would be a right TRAM flap. The defect encompasses a composite defect of the left side of the sternum, including the left internal mammary artery, four contiguous ribs, and the entire left breast. The question specifically asks for soft-tissue thoracic reconstruction. The analysis of the defect yields a very large soft-tissue requirement. Given this, the most logical choice of flaps to provide this amount of soft tissue on a reliable pedicle would be a right (contralateral) TRAM flap. Furthermore, this flap could also be shaped to provide a breast mound versus all the other choices.

A left latissimus muscle flap with skin graft is not the optimal choice as it would not provide sufficient soft-tissue coverage with its associated skin paddle without undue donor site morbidity in this particular patient with a large defect that spans parasternal and lateral chest wall. As a muscle-only flap with skin graft, however, it may be considered as a backup option in case of primary flap failure.

An omental flap with skin graft is an option for reconstruction of this defect; however, it would not be the most appropriate option in this case as it would necessitate an otherwise unnecessary intra-abdominal procedure and would not reshape a breast mound. It has been shown, however, to be optimal in contaminated areas due to its associated lymphatic vessels and nodes associated with its pedicle, so in different circumstances, this may be a preferred choice.

A left rectus turnover flap with skin graft would not be a good option for two reasons: the left internal mammary artery has been harvested, thereby compromising the superior epigastric vessel on which this flap would be based; and it would not cover the entirety of the defect.

A reverse abdominoplasty flap would not be able to reach the full superior extent of the defect.

111
Q

A 42-year-old woman comes to the office because she is dissatisfied with an obvious step-off between the chest wall and the superior pole of the breast 6 months after she underwent immediate expander reconstruction of the left breast. No further adjuvant therapy was indicated. After full expansion, the tissue expander was removed, and a permanent smooth, round silicone prosthesis was placed. BMI today is 28 kg/m2. Examination shows a well-defined inframammary fold. The volume is matched to the contralateral breast. Which of the following is the most appropriate surgical procedure for correction of this patient’s deformity?

A) Exchange of the silicone prosthesis with a silicone prosthesis of a larger volume and dimension
B) Exchange of the silicone prosthesis with a saline prosthesis of the same volume and dimension
C) Fat grafting to the superior pole and chest wall
D) Placement of acellular dermal matrix as an inferolateral sling
E) Removal of the prosthesis and reconstruction with a deep inferior epigastric perforator flap

A

The correct response is Option C.

Fat grafting has been shown to successfully address acquired contour deformities in breast reconstruction. Changing the prosthesis to a saline prosthesis of the same size and dimension will not address the problem of the step-off between the prosthesis and the chest wall in this slender patient (BMI 20 kg/m2) which is due to decreased soft-tissue coverage in the superior pole. Likewise, increasing the size and dimensions of the silicone prosthesis would not correct this contour deformity and would also lose the symmetry with the contralateral breast. This patient, without excess lower abdominal skin and subcutaneous fat, is not a candidate for a deep inferior epigastric perforator flap. Placement of acellular dermal matrix as an inferolateral sling is typically performed in the initial stage of tissue expansion and reconstruction. It can help define the inframammary fold, which this patient does not need.

112
Q

A 74-year-old woman comes to the office because of a 2-month history of increasing soreness over the left lateral chest wall. History includes bilateral radical mastectomy and adjuvant radiation to the left chest 30 years ago because of left-sided breast cancer. On examination, a small, nonhealing 1 × 2-cm wound is visible in the subaxillary region within an area of dense fibrotic skin. A photograph is shown. Which of the following is the most appropriate next step?

A) Biopsy of the nonhealing wound
B) Coverage with a left latissimus muscle flap and skin grafting
C) LHyperbaric oxygen treatment
D) MRI of the chest wall
E) Oral administration of antibiotics for 7 days

A

The correct response is Option A.

This patient’s presentation is consistent with late radiation tissue injury (LRTI), which occurs in 5 to 15% of long-term cancer survivors who have received radiation and can occur months to years (even decades) after treatment. It can vary significantly with age, dose, and site of treatment. Characteristics include progressive tissue deterioration secondary to decreased vascularity followed by replacement of normal soft-tissue architecture by dense fibrotic tissue until there is insufficient oxygen delivery to sustain normal function. When LTRI of the chest wall progresses to skin breakdown and ulceration, as has happened in this patient, a biopsy is always required to rule out recurrence of the primary tumor or a radiation-induced squamous cell carcinoma or soft-tissue sarcoma.

Hyperbaric oxygen therapy has been proposed as a treatment modality that can improve tissue quality and prevent tissue breakdown in irradiated areas. While most data on this relate to treatment of osteoradionecrosis of the mandible, there have been some reports of its use on the chest wall after irradiation due to breast cancer. However, the question specifically asks for the most appropriate next step, which would be biopsy rather than hyperbaric oxygen therapy.

The principles of management once malignancy has been ruled out include debridement of necrotic tissues (including ribs) and reconstruction with well-vascularized flaps. In this case, a left latissimus muscle flap and skin graft was used for reconstruction after aggressive debridement. No alloplastic material was placed or thoracic cage reconstruction performed. This is common in these types of patients as excessive fibrosis caused by the radiation to the chest wall prevents loss of respiratory efficiency through paradoxical motion.

CT scan or MRI may be useful in evaluating the size, extent, and nature of the problem, but does not take priority over tissue biopsy.

The redness that is visualized on the patient’s skin in the case is related to post-radiation changes and telangiectasias, which are common in LTRI, and is not infection. Therefore, antibiotics would be unnecessary.

113
Q

A 33-year-old woman comes to the office with a 6-cm rapidly growing tumor of the left breast. She wears a size 36C brassiere. The tumor has a bluish hue and skin veins are dilated. A phyllodes tumor is diagnosed, and surgical excision is planned. Which of the following is the most appropriate surgical procedure to treat this patient?

A) Excision with 1-cm margin
B) Excision with 2-cm margin
C) Excisional biopsy
D) Modified radical mastectomy
E) Radical mastectomy

A

The correct response is Option A.

Phyllodes are large benign tumors that occur primarily in the perimenopausal patient. Previously, they were referred to as cystosarcoma phyllodes, a term coined in 1838 because the tumors are fleshy and have a gross leaf-like intracanalicular growth pattern. However, this is a misnomer because these tumors do not behave like sarcomas and are rarely malignant. The histologic characteristics that separate fibroadenomas from phyllodes tumors are not well defined and have been somewhat controversial. Nevertheless, phyllodes tumors typically are large fibroadenomas that histologically have more stromal cellularity than that seen in the typical fibroadenoma. The classification of benign versus malignant phyllodes tumors is not sharply delineated, and the term borderline lesion may be more appropriate. Borderline lesions have more mitoses per high-power field and moderate nuclear pleomorphism. They have a tendency to recur after local excision but do not demonstrate true malignant behavior. When metastases of a phyllodes tumor have been reported, there have been obvious sarcomatous elements such as liposarcoma or rhabdomyosarcoma in the lesion.

The surgical treatment of phyllodes tumors has recently been redefined. In the past, simple or radical mastectomies were recommended for the treatment of large phyllodes tumors. Currently, most surgeons perform more conservative surgery. Several clinical studies have recommended the excision of tumors with 1-cm clear margins or mastectomy if breast conservation is impossible.

114
Q

A 41-year-old man is referred to the office because of a mass on his left breast that has been growing rapidly for 1 month. Examination of the specimen obtained on core needle biopsy confirms invasive mammary carcinoma. Genetic testing results are positive for BRCA2. In addition to an increased risk of male breast cancer, which of the following other types of cancer is most likely to be associated with this patient?

A) Colon
B) Lung
C) Pancreatic
D) Renal
E) Thyroid

A

The correct response is Option C.

Men with harmful BRCA1 mutations also have an increased risk of breast cancer and, possibly, pancreatic cancer, testicular cancer, and early onset prostate cancer. However, male breast cancer, pancreatic cancer, and prostate cancer appear to be more strongly associated with BRCA2 gene mutations. Colon, lung, renal, and thyroid cancers are not associated with the BRCA1 and BRCA2 gene mutations.

115
Q

A 40-year-old woman is referred for reconstruction following mastectomy for a peripherally located ductal carcinoma in situ. A nipple-sparing mastectomy with immediate, single-stage prosthesis reconstruction with acellular dermal matrix is planned. Which of the following interventions is most appropriate to ensure that no residual cancer exists?

A) Chemotherapy
B) Ductal washing
C) Radiation therapy
D) Retroareolar frozen section
E) Sentinel lymph node evaluation

A

The correct response is Option D.

More plastic surgeons are performing reconstruction for women pursuing prophylactic mastectomy, which is requested quite frequently to avoid cancer recurrence and to achieve optimal aesthetic outcome. Exclusion criteria for nipple-sparing mastectomy include tumors greater than 3 cm, clinical invasion of the nipple-areola complex, tumors within 2 cm of the nipple, evidence of multicentric disease, positive intraoperative retroareolar frozen section, or nodal disease. If carcinoma is found in the retroareolar tissue, the nipple-areola complex must be removed. A patient who would require sentinel lymph node evaluation, radiation therapy, or chemotherapy would not be an appropriate candidate for nipple-sparing mastectomy. Annual mammography is recommended for any patient with a history of breast cancer and is not specific to the issue of nipple-sparing mastectomy. Ductal washing is not relevant for this pathology.

116
Q

An otherwise healthy 52-year-old woman is scheduled to undergo bilateral breast reconstruction with abdominal perforator flaps. She does not smoke cigarettes. Which of the following is the most appropriate antibiotic prophylaxis regimen?

A) Preoperative antibiotics
B) Preoperative chlorhexidine bath
C) Preoperative and intraoperative antibiotics
D) Preoperative, intraoperative, and postoperative antibiotics for 7 days
E) Preoperative, intraoperative, and postoperative antibiotics until all drains are removed

A

The correct response is Option C.

For a healthy patient undergoing a lengthy autologous breast reconstruction, the most appropriate regimen of antibiotic prophylaxis consists of a preoperative dose of intravenous antibiotic and intraoperative antibiotics to ?maintain therapeutic levels during operation and, at most, a few hours after closure? (Mangram, et al). Although preoperative chlorhexidine bathing is supported by some studies and strong theoretical rationale, it is not nearly as effective as antibiotic prophylaxis on its own. A single dose of preoperative antibiotic intravenously is appropriate for short procedures; however, therapeutic levels of antibiotic should be maintained throughout the lengthy surgery described. Continued administration of prophylactic antibiotics for 7 days following surgery or until drains are removed has not been proven to reduce the incidence of surgical site infections. This regimen also promotes drug resistance and adverse patient reactions.

117
Q

A 12-year-old girl with Beckwith-Wiedemann syndrome develops profound breast enlargement at the onset of puberty. Physical examination shows two distinct masses in the right breast and severe hyperplasia consuming the left breast. A photograph is shown. Which of the following is the most appropriate surgical management?

A) Hormone suppression therapy
B) Right lumpectomies; left mastectomy with skin reduction with application of nipple as skin graft
C) Right simple mastectomy; left mastectomy with skin reduction with application of nipple as skin graft
D) Right simple mastectomy; left simple mastectomy with sentinel lymph node biopsy
E) Right total mastectomy; left modified radical mastectomy

A

The correct response is Option B.

Epithelial hyperplasia is a benign pathological process. Therefore, modified radical mastectomy or sentinel lymph node biopsy would not be indicated in the absence of cancer. Mastectomy on the right is not indicated because the masses are discrete and separate from the normal breast parenchyma. As such, a viable breast mound can be preserved in this 12-year-old girl. Because of the expansive nature of her hyperplasia on the left, skin reduction is required to ensure smooth contour of her skin flaps. The nipple may be spared because of the benign disease. Reconstruction can be performed in a delayed fashion after development is complete. An adjustable expander/prosthesis can also provide a reasonable breast mound until definitive reconstruction is performed.

Hormone suppression therapy may temporarily slow the growth of the hyperplasia but could adversely affect this child’s normal development. It would not comprise definitive management.

118
Q

A 52-year-old woman with breast cancer undergoes right mastectomy and reconstruction with a free transverse rectus abdominis musculocutaneous (TRAM) flap. The procedure is uneventful. In the recovery room, the patient’s husband says that she has been smoking one pack of cigarettes daily up to the day of surgery. Which of the following postoperative complications is most likely to occur?

A) Flap hematoma
B) Mastectomy skin loss
C) Microvascular thrombosis
D) Partial flap loss
E) Superficial infection

A

The correct response is Option B.

Patients who smoke cigarettes and who undergo breast reconstruction with a free flap have a higher rate of mastectomy skin loss. In addition, they have a high rate of donor site abdominal flap necrosis, umbilical necrosis, and hernias. There is no increase in microsurgical complications, flap-related complications (partial flap loss or fat necrosis), infections, or hematomas.

Current recommendations are for patients to stop smoking at least 4 weeks prior to breast reconstruction. Patients who quit smoking prior to this period have a lower risk of perioperative complications when compared to active smokers.

119
Q

A 53-year-old woman is referred for consultation regarding breast reconstruction following mastectomy. Autologous breast reconstruction options are discussed. Which of the following is an advantage of the pedicled transverse rectus abdominis musculocutaneous (TRAM) flap over the free TRAM flap?

A) Decreased complications in patients with a history of smoking
B) Decreased incidence of complete flap loss
C) Decreased risk of abdominal hernia
D) Decreased risk of fat necrosis
E) Ease of insetting and flap shaping

A

The correct response is Option B.

There is considerable controversy regarding the merits of the pedicled TRAM, free TRAM, free deep inferior epigastric perforator (DIEP), and free superficial inferior epigastric perforator (SIEP) flap reconstructions. Reconstruction with each flap has distinct advantages and disadvantages. Currently, the literature does not clearly favor the use of one procedure over the others.

The pedicled TRAM flap is characterized by a shorter operative time and a decreased risk of complete flap loss when compared to the free flaps. However, the pedicled TRAM flap shows a higher incidence of fat necrosis with partial flap loss, as well as an increased length of stay. Free flaps offer ease of flap shaping and insetting, as well as a decreased risk of abdominal hernia and abdominal wall weakness. Free flaps are more suitable for patients who are diabetic, are overweight, or smoke cigarettes.

Deciding which particular procedure to perform requires assessment on a case-by-case basis and consideration of the surgeon’s level of comfort with the different techniques.

120
Q

A 45-year-old woman with a T2 N0 M0 invasive ductal carcinoma in the inferior pole of the left breast is scheduled to undergo segmental mastectomy followed by radiation therapy. She currently wears a size 36E brassiere and wants to have any cup size from a C to D. After segmental resection of the tumor, which of the following procedures is most likely to yield the best cosmetic result in this patient?

A) Bilateral reduction mammaplasty
B) Primary closure of the left breast and reduction mammaplasty of the right breast
C) Reconstruction of the left breast with a latissimus dorsi musculocutaneous flap and reduction mammaplasty of the right breast
D) Reconstruction of the left breast with a transverse rectus abdominis musculocutaneous flap and mastopexy of the right breast
E) A single-stage breast reconstruction with placement of a silicone prosthesis in the left breast and reduction mammaplasty of the right breast

A

The correct response is Option A.

In a patient with large breasts, in whom a partial mastectomy is required, reduction mammaplasty is an appropriate management. This procedure will potentially relieve symptoms of macromastia, reduce the amount of breast tissue present in both breasts, and offer the best aesthetic outcome. Implantation of a prosthesis in a breast that will undergo radiation therapy increases the risk for complications and would likely lead to a less symmetrical result. The latissimus dorsi or transverse rectus abdominis musculocutaneous (TRAM) flaps could be used for immediate partial reconstruction, but they are ideal for delayed reconstruction of partial mastectomy defects. Reduction mammaplasty does not preclude future reconstruction options, but a latissimus flap reconstruction would. With a T2 tumor, a significant portion of the lower pole of the breast is removed to obtain clear margins. Even though the patient described has moderate-to-large breasts, there is a high likelihood that she will develop a deformity of the left breast and asymmetry with the right breast if no reconstruction is performed.

121
Q

A 46-year-old woman with ductal carcinoma in situ is scheduled to undergo right mastectomy. Immediate reconstruction with a tissue expander and acellular dermal matrix (ADM) is planned. Which of the following is an expected outcome with use of ADM?

A) Decreased formation of seroma
B) Decreased incidence of hematoma
C) Decreased risk of infection
D) Increased capsule contracture
E) Increased initial fill of the expander

A

The correct response is Option E.

According to Sbitany, et al, acellular dermal matrix (ADM) allows for a greater initial fill of saline. This potentially improves cosmetic outcome, as it better capitalizes on preserved mastectomy skin for reconstruction. Sbitany, et al, concluded that ADM-assisted prosthesis breast reconstruction has a safety profile no worse than that of complete submuscular coverage but offers the benefit of fewer expansions and the potential for more predictable secondary revisions.

ADM has enhanced prosthesis-based reconstruction and remains useful in immediate prosthetic breast reconstruction. However, it has been found to have higher rates of postoperative seroma and infection. It has also been reported to decrease capsule contracture.

122
Q

A 42-year-old woman is evaluated because of an invasive cancer of the right breast. Which of the following best approximates the likelihood that this patient’s cancer is associated with the BRCA1 or BRCA2 genes?

A) 1%
B) 10%
C) 25%
D) 40%
E) 75%

A

The correct response is Option B.

Among familial breast cancers, 5 to 10% are considered to be hereditary. These familial breast cancers are linked to specific mutations on a cancer susceptibility gene. The breast cancer susceptibility genes (BRCA) belong to a class of genes known as tumor suppressors. In normal cells, BRCA1 and BRCA2 genes stabilize the DNA and prevent uncontrolled cell growth.

A woman’s lifetime risk of developing breast and/or ovarian cancer is greatly increased if she inherits a mutation on BRCA1 or BRCA2 genes. BRCA1- and BRCA2-related breast cancers occur in younger women and are often associated with estrogen receptor-negative tumors.

123
Q

A 37-year-old woman comes to the office for consultation regarding left breast reconstruction after mastectomy to treat breast cancer. Chemotherapy and radiation therapy are planned postoperatively. She wears a size 34D brassiere. Height is 5 ft 6 in (168 cm), and weight is 160 lb (73 kg). BMI is 25.8 kg/m2. She does not want abdominal scars. Autologous reconstruction with a transverse musculocutaneous gracilis (TMG) flap is planned. Which of the following is a disadvantage of using a TMG flap for this patient’s reconstruction?

A) Difficulty of flap harvest
B) High risk of donor site morbidity
C) High risk of fat necrosis
D) Small flap size
E) Variable vascular anatomy

A

The correct response is Option D.

For breast reconstruction, the TMG flap offers a valuable alternative therapy compared with other standard flaps from the lower abdomen, such as the transverse rectus abdominis musculocutaneous (TRAM) or DIEP flaps. The amount of tissue that can be transferred, however, is limited. The largest flap harvested in one large series weighed 420 g and measured 30 × 10 cm. The flap usually offers enough volume to reconstruct small- to mid-sized breasts. The major advantage of this flap as compared with other flaps, such as the gluteal flap or the perforator latissimus flap, is its constant vascular anatomy. Flap perfusion is always reliable, with low rates of fat necrosis and tissue similar in consistency with breast tissue. The donor scar is inconspicuous. The incision is comparable to incisions for a thigh lift and is well hidden. A distortion of the labia majora is typically not observed. However, as in thigh lifts, a lowering of the scar remains a possible problem. Functional donor-site morbidity after TMG flap harvest itself is low. After having clinically established the transverse approach for routine procedures, harvesting one flap is rapid and usually takes no longer than 30 minutes.

124
Q

A 43-year-old Caucasian woman is referred to the office because of a mass on her right breast that has been rapidly growing for 8 weeks. Physical examination shows a 4.5-cm, freely movable mass in the right breast. No axillary adenopathy or nipple discharge is noted. Which of the following is the most likely nonepithelial neoplasm in this patient?

A) Fibroadenoma
B) Hamartoma
C) Lipoma
D) Phyllodes tumor
E) Primary breast lymphoma

A

The correct response is Option D.

Primary breast lymphoma is rare; it constitutes less than 0.6% of all breast malignancies. Phyllodes tumors represent about 1% of tumors in the breast; they are the most commonly occurring nonepithelial neoplasm of the breast. The tumor has a smooth, sharply demarcated texture and is generally freely movable. It is a relatively large tumor, with an average size of 5 cm. A hamartoma is the most common benign tumor of the lung. Hamartomas of the breast are benign tumors composed primarily of dense fibrous tissues with variable amounts of fat and associated ducts. Eighty-five percent of phyllodes tumors are benign, and 15% are malignant. There is no race predilection, but phyllodes tumors occur almost exclusively in women. In young women under age 25 years, asymmetric, tender, and fibrocystic tissues usually point to a fibroadenoma or circumscribed fibrocystic mass. Lipoma of the breast causes diagnostic and therapeutic uncertainty. Clinically, it may be difficult to distinguish a lipoma from other conditions. Fine-needle aspiration cytology is often not helpful. Both mammography and ultrasonography results are often negative. MRI may be overread, but it is useful as a diagnostic tool. This neoplasm can be seen in men and women.

125
Q

A 60-year-old woman receives low-molecular-weight heparin (LMWH) 40 U subcutaneously 1 hour before undergoing breast reconstruction using a unilateral transverse rectus abdominis musculocutaneous (TRAM) flap. Weight is 185 lb (84 kg); BMI is 32 kg/m2. Which of the following is the most likely effect of the LMWH on perioperative risks in this patient?

A ) Decreased risk of flap failure

B ) Decreased risk of postoperative hematoma

C ) Decreased risk of pulmonary thromboembolism

D ) Increased risk of blood transfusion

E ) Increased risk of infection

A

The correct response is Option C.

There are no uniform standards or guidelines for the routine use of chemoprophylaxis of venous thromboembolism in plastic surgery. At a minimum, lower extremity mechanical compression devices should be used on all patients undergoing general anesthesia. The patient described has multiple risk factors for developing deep venous thrombosis (DVT) and pulmonary embolism, such as an age over 50, prolonged surgery time, malignancy, and being overweight.

In such high-risk patients, data support the use of chemoprophylaxis with either standard heparin or low-molecular-weight heparin given subcutaneously. As the risk of developing DVT begins with anesthesia induction, it is generally recommended that heparin therapy be started before surgery. Out of concern for bleeding, some surgeons start heparin therapy in the early postoperative period. While more data are needed to clarify the optimal start time of therapy, it is clear that starting therapy before surgical incision is generally safe when dosed appropriately.

In large retrospective studies, chemoprophylaxis patients were not more likely to require blood transfusion, though they do demonstrate a slightly greater decrease in postoperative hemoglobin as compared with control patients. There is clearly a decreased risk of postoperative DVT and pulmonary embolism, both clinically apparent and asymptomatic. The data regarding postoperative hematoma are less clear, with some studies showing no increase in €œtake-backs € to the operating room for hematoma with chemoprophylaxis. One study did show an increase in hematomas in oncologic breast surgery with LMWH versus standard, unfractionated heparin. There is no association between flap survival or infection and subcutaneous heparin use.

Until reliable prospective, randomized data of sufficient study size are available to demonstrate optimal treatment, the studies seem to support the routine use of mechanical and perioperative subcutaneous heparin prophylaxis in high-risk patients.

126
Q

A 53-year-old woman comes to the office because of ulcerated tissue 6 weeks after undergoing radiation therapy for breast cancer. She underwent mastectomy 1 year ago. Analysis of the radiated tissue is most likely to show an increase in which of the following?

A ) Acute inflammatory response

B ) Cytokines and growth factors

C ) Neutrophil function

D ) Tissue oxygenation

E ) Vessel thrombosis

A

The correct response is Option E.

Analysis of radiated tissue will show increased vessel thrombosis. Ionizing radiation directly damages the genome or injures the DNA through free radical production. In acute radiation injury, the skin becomes erythematous and edematous with the dilation of fine blood vessels, endothelial edema, and lymphatic obliteration. Although perfusion of the skin assessed with fluorescein injection appears normal, tissue oxygenation is inadequate. Healing is impaired with slowed fibroblast proliferation and impairment of the acute inflammatory response. Fibroblast defects are the main problem in the inhibited healing of chronic radiation injury. Phagocytosis and bacteriocidal metabolic functions in neutrophils are also impaired. This effect increases after therapy, which cannot be a direct effect of radiation on the neutrophils because their lifespan is too short. The local wound environment and irradiated tissue do not prime the neutrophils with the appropriate cytokines and growth factors needed for activation. This may lead to an increased incidence of postoperative infections in patients with previous radiation.

Recent studies have shown promise in the treatment of radiation tissue damage with lipoaspirate transplantation. Adipose-derived stem cells have been hypothesized to target damaged areas, release angiogenic factors, form new vessels, and increase tissue oxygenation.

127
Q

A 45-year-old woman who has breast cancer comes to the office for consultation regarding bilateral breast reconstruction. Reconstruction using autologous abdominal tissue is considered. The risk of abdominal morbidity is discussed. Which of the following flap techniques is most likely to result in the lowest level of overall abdominal morbidity?

A ) Deep inferior epigastric artery perforator

B ) Free muscle-sparing transverse rectus abdominis musculocutaneous (TRAM)

C ) Free TRAM

D ) Pedicled TRAM

E ) Superficial inferior epigastric artery flap

A

The correct response is Option E.

The superficial inferior epigastric artery (SIEA) flap results in the lowest level of overall abdominal morbidity, as the technique used in harvesting this flap leaves the abdominal fascia intact. These vessels are only present in less than one third of patients, and only one half of those patients will have vessels of sufficient diameter to support a free tissue transfer. The SIEA flap is also associated with a higher frequency of total flap loss, in addition to a higher incidence of fatty necrosis.

The deep inferior epigastric artery perforator (DIEAP or DIEP) flap involves dissection of one or two (occasionally more) perforators through the rectus muscle to the inferior epigastric vessels. Although this technique does not include any rectus muscle or sheet/fasica within the flap itself, it does involve moderate-level trauma to those organs and can cause abdominal wall morbidity.

The free TRAM or free muscle-sparing TRAM techniques are free tissue transfer variants of the TRAM, whereby a small amount of the rectus is taken with the flap. In the more advanced MS free TRAM technique, the amount of muscle taken is only enough to allow safe transfer of the perforators. Although this technique is less invasive than the pedicled TRAM, it does still involve removal of a variable portion of the rectus muscle and fascia.

A pedicled TRAM flap technique involves transferring of the flap based on the superficial epigastric vessel that runs within the rectus muscle. Therefore, the entirety of the rectus (unilateral or bilateral) is elevated out of its native abdominal wall location.

Many publications have compared the other modes of breast reconstruction with reference to abdominal wall morbidity. This area remains controversial. The general consensus remains that

in a bilateral reconstruction, pedicled TRAM flaps are associated with higher levels of overall abdominal morbidity (hernias, bulges, weakness, intolerance to exercise, etc) when compared with the use of MS free TRAM, DIEP, or SIEA flaps. The use of SIEA flaps results in minimal to no abdominal wall morbidity.

128
Q

A 45-year-old man comes to the office for consultation regarding breast cancer after undergoing gene testing and learning that he is a carrier of the BRCA2 gene mutation. He has a strong family history of breast, prostate, and ovarian cancers. Which of the following best represents his lifetime risk for developing breast cancer?

A ) 6%

B ) 15%

C ) 30%

D ) 60%

E ) 85%

A

The correct response is Option A.

Since the identification of the first breast cancer gene in 1990, the field of molecular breast cancer testing has grown enormously. It is estimated that approximately 5% to 10% of patients diagnosed with breast cancer last year carry highly penetrant cancer susceptibility genes. It is important to identify these patients early, as the presence of genetic mutations or other high-risk factors may alter management strategies for patients, both prior and subsequent to the diagnosis of breast cancer.

Male breast cancer accounts for less than 1% of all breast cancer cases in the United States and is more common in men with a family history of breast cancer. The relationship between male breast cancer and a deleterious BRCA2 mutation has been well established. In the largest study to date, the lifetime risk of developing breast cancer in a BRCA2 male carrier is approximately 7%. The correlation with BRCA1 and male breast cancer is less clear, but it seems that the presence of BRCA1 mutation confers a lower lifetime risk of developing breast cancer than a BRCA2 mutation. The relative risk of developing breast cancer is highest for men in their 30s and 40s, and it decreases with increasing age.

The lifetime risk for the development of breast cancer in female carriers of BRCA1 and BRCA2 mutations is significantly higher than for males. Female BRCA1 carriers have an 85% risk of developing breast cancer and a 62% risk of developing ovarian cancer. Female BRCA2 carriers have an 85% risk of developing breast cancer and a 25% risk of developing ovarian cancer.

129
Q

A 21-year-old woman comes to the office for consultation regarding a palpable lump in the left breast that she first noticed 4 months ago. Phyllodes tumor of the breast is diagnosed. Which of the following is the most appropriate management at this time?

A ) Observation and incisional biopsy

B ) Neoadjuvant chemotherapy

C ) Local excision and annual surveillance

D ) Wide local excision and radiation therapy

E ) Mastectomy with immediate reconstruction

A

The correct response is Option C.

Phyllodes tumors of the breast were once called cystosarcoma phyllodes because of the fleshy nature of the tumor. Because most tumors are benign, the name may be misleading. Thus, the favored terminology is now phyllodes tumor.

Phyllodes tumor is the most commonly occurring nonepithelial neoplasm of the breast but represents only about 1% of tumors in the breast. It has a sharply demarcated, smooth texture and is typically freely movable. It is a relatively large tumor; the average size is 5 cm. However, lesions greater than 30 cm have been reported. Approximately 90% of phyllodes tumors are benign and approximately 10% are malignant.

In considering excision, the tumor-to-breast ratio should be small enough to allow for segmental excision of the tumor and possible reduction mammaplasty technique (eg, inferior pedicle for a superior tumor or a superior pedicle for an inferior lesion). Annual surveillance and follow-up is recommended because the pathologic appearance does not always predict the clinical behavior.

130
Q

A 29-year-old woman who is positive for the BRCA1 gene underwent bilateral mastectomies and immediate reconstruction with placement of tissue expanders. After completion of the second session of tissue expansion, she has severe pain in the chest wall. There is no evidence of hematoma or infection. Which of the following is the most appropriate next step in management?

A ) Administer pain medication

B ) Apply a warm compress to the breasts

C ) Plan for return to the operating room

D ) Reassure the patient

E ) Withdraw fluid from the tissue expander

A

The correct response is Option E.

Physical signs and symptoms should be closely monitored during tissue expansion. Pain and rapidly blanching skin under tension are evidence of excessive expansion at that setting. The patient €™s pain immediately after expansion (or even in a slightly delayed manner) is indicative of ischemic tissue with poor oxygen tension. Neural tissue has been shown to have the lowest tolerance to ischemia, hence in part explaining the pain. The most appropriate step in management is to withdraw enough fluid to relieve pain. This amount can be dramatically small at times; a few milliliters can correct the problem.

Other measures to relieve pain, such as administering pain medication and applying warm compresses, can be attempted after withdrawing fluid, if necessary. Adding more fluid is inappropriate. There is no need at this stage to return to the operating room.

131
Q

A 48-year-old woman undergoes skin-sparing mastectomy followed by immediate breast reconstruction with implantation of subpectoral prostheses. Which of the following best describes the resulting effect on pectoralis torque strength in this patient?

A ) Decreased on the operated side

B ) Decreased on the side of hand dominance

C ) Increased on the operated side

D ) Increased on the side of hand dominance

E ) No effect

A

The correct response is Option A.

Much attention has been given to abdominal wall function and complications following autologous breast reconstruction utilizing abdominal wall tissue in the form of transverse rectus abdominis musculocutaneous (TRAM), free TRAM, muscle-sparing TRAM, and deep inferior epigastric perforator free tissue transfers.

De Haan and colleagues examined the effect of immediate subpectoral prosthetic breast reconstruction following skin-sparing mastectomy on pectoralis function by measuring bilateral isometric arm strength in patients undergoing unilateral breast reconstruction. After correcting for the effect of hand dominance, the authors found a statistically significant decrease in torque strength on the operated side by 20.1% compared to the nonoperated side. This loss represents approximately half of the torque needed by healthy subjects over 50 years of age to rise from a chair and was therefore considered substantial.

132
Q

An 18-year-old man with Klinefelter syndrome is referred to the office because of overdeveloped breasts. Diagnostic workup testing is ordered. The likelihood of breast cancer is increased if testing shows which of the following results?

A ) Decreased level of plasma follicle-stimulating hormone

B ) Decreased level of plasma luteinizing hormone

C ) High number of progesterone receptors in the mesenchymal breast tissue

D ) Increased level of plasma testosterone

E ) Low number of estrogen receptors in the mesenchymal breast tissue

A

The correct response is Option C.

Klinefelter syndrome is a relatively rare disorder with an incidence of approximately 1:100,000, yet it represents a very important subset of patients with gynecomastia. Approximately one half of patients with typical Klinefelter syndrome (47,XXY) and one third of those with the mosaic variety (46,XY/47,XXY) develop gynecomastia. Those exhibiting gynecomastia have elevated plasma follicle-stimulating and luteinizing hormone levels, along with decreased plasma testosterone levels, suggesting a hormonal link with gynecomastia in the Klinefelter population.

The presence of estrogen and progesterone receptors in elevated concentration in patients with Klinefelter syndrome provides a potential mechanism by which these patients develop breast neoplasms. The absence of elevated estrogen and progesterone receptors in patients with idiopathic gynecomastia might help to clarify why these patients rarely develop breast malignancies.

Gynecomastia has many identifiable causes, although most cases are idiopathic. Recent studies have shown strong evidence for the estrogen-stimulating effects of breast tissue development and support for an inhibitory androgenic effect. Decreases in the androgen-to-estrogen ratio have also been associated with development of gynecomastia. No clear etiologic classification has been suggested based solely on hormonal influences. Instead, most physicians accept an arbitrary classification based on physiologic, pathologic, pharmacologic, and idiopathic causes. Idiopathic causes (25%) are the most common.

133
Q

A 36-year-old woman with a confirmed diagnosis of ductal carcinoma in situ is considering skin-sparing mastectomy and immediate reconstruction. The patient should be informed that her risk of developing recurrent ipsilateral breast cancer over the next four years is approximately which of the following?

A ) 2%

B ) 5%

C ) 10%

D ) 15%

E ) 20%

A

The correct response is Option A.

Several studies support skin-sparing mastectomy as an oncologically safe technique, based on the absence of breast ductal epithelium at the margin of the native skin flaps. A recurrence rate of less than 2% at 45 months is quoted in a study by Slavin and colleagues.

134
Q

A 32-year-old woman with multifocal ductal carcinoma in situ in her right breast is scheduled to undergo mastectomy. There is no family history of breast cancer. Results of testing for the BRCA gene mutation are negative. MRI of the left breast shows no abnormalities. A discussion is held with the patient about prophylactic mastectomy on the contralateral breast. Which of the following is the principal factor in the determination of whether to perform bilateral mastectomy?

A ) Medical clearance

B ) Need for radiation

C ) Patient wishes

D ) Reconstructive options

E ) Tumor burden

A

The correct response is Option C.

In the scenario described, the most important consideration is the patient €™s choice. In several studies, patients who underwent prophylactic mastectomy at the suggestion of the physician were generally more regretful than those who underwent the procedure following a patient-initiated discussion. The lack of perioperative emotional support also affected these patients. Lack of nipple sensation and inability to breast-feed are two factors leading to patients €™ regret about their decision.

Factors influencing a patient €™s decision to choose bilateral mastectomies include BRCA gene status, family history, previous biopsies, dense breasts, and hard-to-read mammograms. Reconstructive options are important to explore with the patient, but ultimately the patient €™s wishes are most predictive of patient satisfaction in prophylactic mastectomy. Radiation and tumor burden do not affect the treatment of the contralateral breast.

135
Q

A 39-year €‘old woman is referred for consultation regarding reconstruction of the left breast two years after mastectomy for breast cancer. Implantation of a silicone prosthesis and reconstruction with a pedicled latissimus dorsi musculocutaneous flap are planned. This patient will most likely show deficits in which of the following shoulder functions?

(A) Extension and abduction

(B) Extension and adduction

(C) Flexion and abduction

(D) Flexion and adduction

(E) No deficits

A

The correct response is Option B.

Transfer of the latissimus dorsi musculocutaneous flap is associated with deficits in extension and adduction.

The latissimus muscle acts synergistically with six other muscles in the shoulder. The primary contribution of the muscle is in shoulder extension, adduction, and medial rotation. There are definite biomechanical changes that occur in the shoulder girdle with latissimus muscle transfer. The synergistic action of the teres major muscle compensates for loss of the latissimus muscle. This leads to teres major hypertrophy in the long term. A study in 1986 evaluated a series of patients with latissimus muscle transfer and found a decrease in total shoulder strength up to 34% compared to the contralateral side. When tested specifically for the latissimus muscle function, the average weakness was 18%. Active range of motion at the shoulder with extension and adduction was decreased by 5% and 0%, respectively.

In 1992, one study used isometric, isotonic, and isokinetic strength tests for evaluation after latissimus transfer. The authors concluded that forced extension is weaker only when the arms are in 60 degrees of flexion. There was no loss of range of motion. Another study done in 1995 reexamined change in muscle power and endurance after latissimus transfer. The authors concluded that women who underwent unilateral pedicled latissimus transfer showed a deficit of power and endurance in shoulder extension and adduction. This was seen in three specific work activities: ladder climbing, overhead painting, and pushing up from a chair. A more rapid onset of fatigue during prolonged activities involving these motions, including swimming, is noted.

136
Q

A 16-year-old girl is brought to the office by her parents because she has had worsening pain in the right breast (shown) for the past eight months. Ten years ago, she sustained a full-thickness burn to the back, right chest, and abdomen requiring excision and split-thickness skin grafting. Physical examination shows contracture of the scar and lateral expansion of the breast. After release of the burn scar contractures, a defect of 200 cm2 is created. Which of the following is most appropriate for coverage of this defect?

(A) Dermal regeneration template (Integra)

(B) Latissimus dorsi musculocutaneous flap

(C) Meshed split-thickness skin graft

(D) Nonmeshed thin split-thickness skin graft

(E) Z-plasty release of the constricted scar

A

The correct response is Option A.

The patient described is suffering from pain caused by a developing breast that is constricted under a nonexpanding split-thickness skin graft placed at the time of the burn excision. The burn scar contractures are first released to allow expansion of the underlying breast. The decision for coverage material is then based on what will most likely permit further expansion of the breast.

Dermal regeneration template (Integra) has been shown to allow secondary expansion and is the most appropriate choice for the scenario described. The wound is covered with an expandable dermal layer, which is then covered again by a split-thickness skin graft, leaving a minimal donor scar.

A Z-plasty would be inadequate to release this large scar.

Full-thickness skin grafting would allow greater breast expansion and less secondary burn scar contracture, but a graft of the size needed would be difficult to obtain.

A latissimus dorsi flap would not be the best option for the patient described because she has a history of burns on the back. Also, a latissimus dorsi flap would be bulky, and the patient described does not require additional volume.

Nonmeshed thin split-thickness skin grafting would give a superior aesthetic result compared with meshed split-thickness skin grafting; however, both would result in secondary burn scar contracture, which would not allow further expansion of the breast.

137
Q

When performing a delayed autologous breast reconstruction, which of the following is the most appropriate placement of the skin paddle on the chest wall for optimal aesthetic appearance?

(A) Inferior and lateral

(B) Inferior and medial

(C) Superior and lateral

(D) Superior and medial

A

The correct response is Option A.

The most advantageous orientation of a skin paddle in the scenario described is inferior and lateral. This placement creates a natural ptotic breast shape. Placement of the skin paddle in the medial or superior position results in an unaesthetic pattern with scars that may be difficult to hide in clothing. Placement of the skin paddle centrally, within the old mastectomy scar, also results in a less aesthetic contour.

When considering aesthetic subunits of the breast during reconstruction, the ideal skin paddle placement and shape is areolar or expanded areolar for an immediate reconstruction. For delayed reconstruction, the ideal aesthetic subunits are lower outer crescent, lower half of the breast, inferolateral crescent, or entire breast.

138
Q

A 40-year-old woman comes to the office for consultation regarding breast reconstruction two years after undergoing modified radical mastectomy for breast cancer. Physical examination shows atrophy of the lower lateral pectoralis major muscle. Transection of which of the following nerves during mastectomy is the most likely cause of the atrophy?

(A) Intercostobrachial

(B) Lateral intercostal

(C) Long thoracic

(D) Medial pectoral

(E) Thoracodorsal

A

The correct response is Option D.

The pectoralis major muscle is innervated by branches from the brachial plexus. The lateral cord gives off the lateral pectoral nerve, which crosses the axillary artery and vein and pierces the deep surface of the muscle. It supplies the medial and superior portions of the muscle. The medial cord gives off the medial pectoral nerve, which enters the deep surface of the pectoralis minor and then passes around the edge of the lateral pectoralis major muscle. At this point, this nerve is vulnerable to division during mastectomy. The medial and lateral pectoral nerves are named for the brachial plexus cord of their origin, not for the portion of the muscle that they supply.

The second intercostobrachial nerve supplies the skin of the axilla and inner arm. It is often transected or transiently injured during axillary dissection. The lateral intercostal nerves supply the breast, and the fourth intercostal nerve supplies nipple and areola sensation. The long thoracic nerve comes off the lateral and posterior cords and supplies the serratus anterior. Division of this nerve results in a winged scapula. The thoracodorsal nerve comes from the posterior cord and supplies the latissimus dorsi muscle.

139
Q

A 43-year-old woman is scheduled for reconstruction of the right breast with a latissimus dorsi flap. She is at greatest risk for which of the following complications?

(A) Chronic chest wall pain

(B) Dorsal wound dehiscence

(C) Partial flap necrosis

(D) Seroma

(E) Stiffness in the ipsilateral shoulder

A

The correct response is Option D.

The most common complication in the scenario described is seroma formation, which is estimated to be as high as 35% to 60%. Other less common complications include chest wall pain (6%), dorsal wound dehiscence (4%), partial flap necrosis (1% €“7%), and shoulder stiffness (1%).

140
Q

A 58-year-old nulligravid woman who is scheduled to undergo bilateral prophylactic mastectomy comes to the office for consultation regarding immediate breast reconstruction. She works full time as a fitness instructor. The patient currently wears a size 34B brassiere and wants her brassiere size to be increased to a C cup, but she wants to make sure that scarring is minimized. Height is 5 ft 5 in and weight is 120 lb. Physical examination of the abdomen shows paucity of extra tissue. Which of the following is the most appropriate breast reconstruction procedure for this patient.

(A) Bilateral autogenous reconstruction

(B) Bilateral tissue expansion followed by implantation of prostheses

(C) Delayed breast reconstruction after pathology is confirmed

(D) Single-stage reconstruction with prostheses

(E) TRAM flap followed by implantation of prostheses

A

The correct response is Option B.

In the thin patient described, autogenous tissue transfer often requires implantation of prostheses to provide enough volume to create a breast mound. This can be done alone or in combination with either the latissimus dorsi myocutaneous flap or the TRAM flap. A recent study suggests that there may be less risk of recipient site morbidity in reconstruction with a TRAM flap with prostheses placed or completely filled subsequently compared with immediate placement of prostheses.

Bilateral latissimus dorsi or TRAM flaps may not be the first choice for a fitness instructor who will most likely require prostheses. The recovery time and the associated donor site morbidity may prevent her from returning to the gym as an instructor. Tissue expander placement and placement of prostheses can often be done through the same incisions used for extirpation and as such have the least amount of visible scar. No additional tissue is used to create the breast mound. Immediate breast reconstruction can offer some psychological benefits for the woman and may be associated with fewer surgeries overall. Prostheses placed without an expanded pocket may not be as reliable in reconstruction of moderate-sized breasts.

141
Q

A 55-year €‘old woman undergoes modified radical mastectomy with immediate first-stage reconstruction of the right breast with a tissue expander. Before the second stage to exchange the tissue expander with a permanent prosthesis is initiated, pathology results from analysis of tissue from the right breast indicate metastatic carcinoma of four axillary lymph nodes. Radiation therapy is recommended. Which of the following interventions will result in the best long-term appearance of the reconstructed breast?

(A) Complete the tissue expansion before radiation and exchange the tissue expander with a prosthesis after radiation

(B) Deflate the tissue expander before radiation and reinflate the tissue expander and exchange with a prosthesis after radiation

(C) Remove the tissue expander and reconstruct the breast with a TRAM flap before radiation

(D) Remove the tissue expander before radiation; after radiation, reinsert and expand a tissue expander and then exchange with a prosthesis

(E) Remove the tissue expander before radiation and reconstruct the breast with a TRAM flap after radiation

A

The correct response is Option E.

Breast reconstruction after radiation therapy is best accomplished with autologous tissue such as a TRAM flap. Any reconstruction method that uses prostheses or tissue expanders after radiation therapy has a higher risk of complications and poor cosmetic outcome when compared with reconstruction using autologous tissue. Radiation increases the risk of infection and exposure of the prosthesis, leading to failure of breast reconstruction procedures using prostheses. Radiation also increases the risk for capsular contracture, resulting in a poor aesthetic result.

Breasts that are reconstructed with a TRAM flap and subsequently radiated are prone to distortion, shrinkage, and fibrosis, leading to a poor aesthetic outcome. Therefore, the best long-term results are obtained when the breast is reconstructed with a TRAM flap after radiation therapy has been completed.

142
Q

A 45-year-old woman with T3 N0 invasive ductal carcinoma in the inferior pole of the left breast is scheduled to undergo segmental mastectomy and subsequent radiation therapy. She currently wears a size 36DDD brassiere and is willing to accept any cup size from C to DDD. Which of the following interventions will yield the best cosmetic result in this patient?

(A) Bilateral reduction mammaplasty

(B) Implantation of a prosthesis in the left breast and mastopexy of the right breast

(C) Latissimus dorsi musculocutaneous flap reconstruction of the left breast and mastopexy of the right breast

(D) TRAM reconstruction of the defect

(E) No reconstruction

A

The correct response is Option A.

In a patient with large breasts, in whom a partial mastectomy is required, reduction mammaplasty is a very good option. This procedure will potentially relieve symptoms of macromastia, reduce the amount of breast tissue present in both breasts, and offer the best aesthetic outcome.

Implantation of a prosthesis in a breast that will undergo radiation therapy increases the risk for complications and would likely lead to a less symmetrical result.

The latissimus dorsi (or TRAM) flap could be used for immediate partial reconstruction but is ideal for delayed reconstruction of partial mastectomy defects. Reduction mammaplasty does not preclude future reconstruction options, but a latissimus flap reconstruction would.

With a T3 tumor, a significant portion of the lower pole of the breast is removed to obtain clear margins. Even though the patient described has moderate to large breasts, there is a high likelihood that she will develop a deformity of the left breast and asymmetry with the right breast if no reconstruction is performed.

143
Q

A 45-year-old woman who underwent reconstruction of the left breast with a TRAM flap followed by radiation two years ago is evaluated for a second reconstruction procedure on the left breast (shown). She requests that only reconstruction with her own tissues be performed and is opposed to surgery of the right breast to restore symmetry. On physical examination, the breast is shrunken and firm. After removal of the unsalvageable left breast, which of the following is the most appropriate reconstruction technique at this time?
(A) Deep inferior epigastric perforator (DIEP) flap
(B) Free TRAM flap
(C) Latissimus dorsi myocutaneous flap
(D) Superior gluteal artery perforator (SGAP) flap
(E) Thoracoepigastric flap

A

The correct response is Option D.

When there is near-total loss of a TRAM flap for breast reconstruction, it is important that a second breast reconstruction procedure be successful, with special consideration for safety. When there is insufficient lower abdominal tissue for tissue expansion and transfer of a second flap, neither a free TRAM flap nor a deep inferior epigastric perforator (DIEP) flap recruiting soft tissue in the same lower abdominal areas is an option. A latissimus dorsi myocutaneous flap is a reliable source of autogenous tissue; however, considering the size of the contralateral breast in this patient, a latissimus dorsi myocutaneous flap would need to be supplemented by a permanent expander implant to provide sufficient volume. A staged breast reconstruction with tissue expander would also require permanent implantation of a prosthesis upon completion of the expansion process. A gluteus free flap is considered when the latissimus dorsi flap is not sufficient or available and the patient requests autogenous tissue breast reconstruction. The free sensate superior gluteal artery perforator (SGAP) flap is a free sensate flap based on the superior gluteal artery perforator. It has an abundance of adipose tissue (specimens up to 760 g), even in thin patients, a relatively long vascular pedicle, a discrete scar, improved projection compared with the DIEP and TRAM flaps, and preservation of the entire gluteus maximus muscle with extremely low donor site morbidity.

144
Q

A 40-year-old woman is scheduled to undergo reconstruction of the right breast via a free TRAM flap. She has smoked two packs of cigarettes daily for the past eight years. This patient=s smoking history increases her risk of which of the following postoperative complications?

(A) Hematoma

(B) Mastectomy flap necrosis

(C) Seroma

(D) TRAM flap loss

(E) Vessel thrombosis

A

The correct response is Option B.

One study found no significant increased risk of flap loss, vessel thrombosis, or fat necrosis in free TRAM flaps performed on smokers. However, smokers did have an increased risk over their nonsmoking counterparts in the area of mastectomy skin flap necrosis, abdominal hernia, and abdominal flap necrosis.

Another study of 569 free TRAM flaps found that smokers had an increased rate of fat necrosis, wound infection, abdominal flap necrosis, and mastectomy flap necrosis. There was no correlation with free flap loss, thrombosis, hematoma, or seroma.

By using the deep inferior epigastric vessels, which are the primary blood supply to the TRAM flap, risks to the flap itself are minimized even in smokers. But smoking continues to increase the risk of donor site morbidity and native breast skin necrosis.

145
Q

A 48-year-old woman comes to the office for consultation regarding reconstruction of the right breast after mastectomy because of cancer. The patient is concerned about maximizing the aesthetic result and minimizing any donor site deformity. Physical examination shows a well-healed chest wall and a B-cup left breast with Grade 3 ptosis. Soft-tissue reconstruction with a superior gluteal artery perforator (SGAP) free flap is planned. Which of the following is a disadvantage of this procedure?

(A) Difficulty molding the gluteal fat

(B) Gait dysfunction

(C) Inability to fully hide the donor scar

(D) Inability to provide a sensate flap

(E) Lack of abundant soft tissue

A

The correct response is Option A.

In soft-tissue reconstruction with a superior gluteal artery perforator (SGAP) free flap, the firmer consistency of gluteal fat causes difficulty in molding tissue, particularly in breast reconstruction. Secondary procedures may be necessary to obtain the desired shape.

Gait dysfunction is extremely uncommon after a SGAP flap if care is taken to avoid injury to the motor branch. The scar is generally well hidden by clothing and the donor site deformities are generally minimal. Even in thin patients there is an abundance of adipose tissue in the gluteal area. Although not commonly done, a sensate branch of the nervi clunium superiores can provide sensation to the flap.

146
Q

A healthy 27-year-old woman comes to the office for follow-up examination three months after undergoing bilateral prophylactic mastectomy and reconstruction with saline prostheses. She is concerned about the incision on her left breast because it is slightly swollen and warm to the touch. Physical examination shows mild erythema of the left breast and normal healing of the right breast. Which of the following is the most appropriate initial management?
(A) Observation
(B) Oral antibiotic therapy
(C) Open lavage of the implant pocket
(D) Removal of the prosthesis
(E) Exchange of the prosthesis

A

The correct response is Option B.

In a situation such as this, in which a patient has a minor infection and no actual or threatened exposure of the prosthesis, antibiotics alone are the treatment of choice. Follow-up certainly would be necessary because a mild infection could become severe if not properly treated. Severe infections around the prostheses or exposure of the prosthesis certainly dictates more aggressive therapy. In a prophylactic situation in which a patient has not had radiation therapy to the chest wall, infection of the prosthesis has been shown to be less likely than in the presence of radiation.

147
Q

In reconstruction mammaplasty after mastectomy, which of the following is the most significant advantage of self-filling osmotic tissue expanders over conventional tissue expanders?
(A) Anatomic shape
(B) Incidence of infection
(C) Permanent implantation
(D) Short filling phase
(E) Subcutaneous placement

A

The correct response is Option B.

Because there is no need for external fillings, the risk of infection and painful injections is obviated with self-filling osmotic tissue expanders, which contain osmotic-active hydrogel vinyl pyrrolidine and methylmethacrylate (developed by Wiese and produced by Osmed in Elman, Germany). The second-generation expander has a silicone membrane with small pores enclosing the osmotic hydrogel. This membrane reduces expansion speed, and final volume is accurately defined. It has been successfully used in delayed reconstruction mammaplasty, correction of breast anomalies (i.e., tubular breast, micromastia, etc.), and coverage of defects after excision of tumors, scars, burns, and alopecia.

As with conventional tissue expanders, self-filling osmotic expanders are not meant to be permanent, require submuscular placement, have a filling phase of 40 to 60 days, and require replacement with a permanent prosthesis in four to six months. The anatomically shaped osmotic expander is prone to rotation, resulting in an inferior result.

148
Q

For reconstruction mammaplasty, which of the following is an advantage of the extended latissimus dorsi flap over the standard latissimus dorsi myocutaneous flap?
(A) Better flap perfusion
(B) Decreased need for breast implant
(C) Fewer donor-site seroma
(D) Less sacrifice of latissimus dorsi muscle
(E) Smaller donor-site scar

A

The correct response is Option B.

The latissimus dorsi (LD) myocutaneous flap was one of the first methods of breast reconstruction ever described. However, with the increasing popularity of transverse rectus abdominis myocutaneous (TRAM) flap breast reconstruction, for many, the LD flap has become a secondary choice for autologous breast reconstruction. One reason for this has been that the standard LD flap alone often does not provide sufficient volume for breast reconstruction. In fact, it is a common practice to combine the LD flap with a prosthesis to achieve adequate breast volume.

Variations of the LD flap have been described to increase its volume and avoid the addition of a prosthesis. The first “extended” LD (ELD) flap was described by Hokin in 1983 and included lumbar fat extensions of the LD flap. Others have used the buried de-epithelialized LD myocutaneous flap for breast reconstruction. McCraw and Papp modified this technique by using a fleur-de-lis skin paddle design to carry additional fat on the surface of the LD muscle, creating the totally autogenous LD breast reconstruction.

The design of the ELD flap has evolved to include the parascapular and scapular “fat fascia” in addition to the lumbar fat for additional volume. The main advantage of the ELD flap is that it can provide autogenous tissue to replace breast volume without an implant and with acceptable donor site contour and scar. Because the ELD flap transports additional tissue from the back so a breast prosthesis is not required, this avoids all potential problems associated with implants, e.g., development of capsular contracture and deformation of the reconstructed breast.

With ELD flaps, donor site complications, including seroma formation and wound necrosis, can be a significant problem.

149
Q

A 36-year-old woman comes to the office for consultation regarding breast reconstruction one year after she underwent right modified radical mastectomy. The procedure was followed by six weeks of radiation therapy. She has no history of other surgical procedures or serious medical illnesses. Height is 5 ft 4 in tall and weight is 135 lb. She wears a size 32B brassiere. Which of the following is the LEAST appropriate breast reconstruction procedure for this patient?
(A) Extended latissimus dorsi flap
(B) Latissimus dorsi flap with saline-filled prosthesis
(C) Superior gluteal artery perforator (SGAP) flap
(D) TRAM flap
(E) Two-staged reconstruction with a tissue expander and saline-filled prosthesis

A

The correct response is Option E.

Radiation therapy has significant negative effects on the outcome of breast reconstruction with prostheses. There is a significant increase in capsular contracture and other complications, which are unrelated to implant type.

Totally autologous reconstruction is the best option for managing the radiated breast. The extended latissimus flap might have adequate volume in this patient (size 32B brassiere) without use of a prosthesis. Even if a prosthesis is required, the new nonradiated skin and muscle of a latissimus flap are advantageous for softness and thickness. The superior gluteal artery perforator (SGAP) and TRAM flaps would also bring in new nonradiated tissue and foreign bodies would not be placed in the breast.

Although the final outcome of a two-stage reconstruction with tissue expanders and prostheses can be acceptable in the radiated field, it is generally not as good as those obtainable with autologous tissue alone.

150
Q

A 60-year-old woman with breast cancer is scheduled to undergo modified radical mastectomy followed by postoperative radiation therapy. Which of the following techniques will yield the most natural appearance of the breast with the fewest complications?

(A) Delayed reconstruction with an autologous tissue flap
(B) Delayed reconstruction with a tissue expander followed by implantation of a prosthesis
(C) Immediate reconstruction with an autologous tissue flap
(D) Immediate reconstruction with a tissue expander followed by implantation of a prosthesis

A

The correct response is Option A.

Reconstruction of the breast with a delayed (nonradiated) autologous tissue flap will yield the best breast appearance if postmastectomy chest radiation therapy is needed.

Radiation therapy negatively affects reconstructive outcomes with implants, causing a marked increase in capsular contracture and other complications. These complications are unrelated to the type of implant.

Totally autologous reconstruction is the best option for reconstruction of a radiated breast. In implant reconstruction of a radiated breast, the latissimus flap can be used to salvage tissue with periprosthetic contractures, particularly in a patient who is not a good candidate for a TRAM flap. The final outcome is not as good as that obtained with autologous tissue alone but can be acceptable.

In one study, patients who had immediate reconstruction with a TRAM flap followed by radiation therapy were compared with patients who had radiation therapy followed by delayed reconstruction with a TRAM flap. The study found that the incidence of late complications was significantly higher in the immediate reconstruction group than in the delayed reconstruction group. In fact, 28% of patients in the immediate reconstruction group required an additional flap to correct a distorted contour caused by flap shrinkage or severe flap contraction. These findings indicate that delayed reconstruction is preferred in patients who are candidates for breast reconstruction with a free TRAM flap and need postmastectomy radiation therapy.

151
Q

A 56-year-old woman comes to the office for routine follow-up 10 days after undergoing skin-sparing right modified radical mastectomy and immediate breast reconstruction with a subpectoral tissue expander. Physical examination shows a 2 _ 3-cm area of frank necrosis of the lower lateral mastectomy skin flap. The tissue expander in this area is not covered by muscle, but it is not exposed and there are no signs of infection. Which of the following is the most appropriate next step in management?

(A) Admission to the hospital for intravenous administration of antibiotics and observation
(B) Debridement of necrotic tissue and primary closure
(C) Debridement of necrotic tissue and removal of the tissue expander
(D) Initiation of tissue expansion by instillation of 100 mL of saline into the tissue expander
(E) Oral administration of an antibiotic and follow-up in one week

A

The correct response is Option B.

For this patient, the most appropriate next step is to debride the necrotic tissue of the mastectomy skin flap using sterile technique and then perform primary closure of the resulting wound. This management minimizes the risk of infection and of exposure of the tissue expander.
Oral or intravenous administration of an antibiotic does not prevent eventual exposure and infection of a breast tissue expander that is covered with nonviable soft tissue. Initiation of tissue expansion in a mastectomy skin flap with frank necrosis risks dehiscence of the mastectomy wound and exposure of the tissue expander.

Removal of the tissue expander is not necessary because it is not infected at this time. If the skin flap necrosis can be debrided while the sterility of the tissue expander is maintained, the resulting wound can undergo primary closure and the breast reconstruction can be preserved. If the pectoralis major or serratus anterior muscle were covering the tissue expander in the necrotic area of the skin flap, this necrotic tissue could be managed nonoperatively by allowing it to slough and by performing regular dressing changes while secondary healing occurs.

152
Q

A 14-year-old girl has absence of the nipple and lack of development of the right breast. The left breast has normal shape and normal nipple-areola complex and fits a B-cup brassiere. Family history includes normal breast development in the parents and siblings. On physical examination, both pectoralis muscles are present and fully developed. No abnormalities of the hands are noted. Which of the following is the most likely diagnosis?

(A) Anterior thoracic hypoplasia
(B) Congenital absence of the breast
(C) Hypoplasia of the breast
(D) Poland syndrome
(E) Tubular breast deformity

A

The correct response is Option B.

Congenital absence of the breast is defined by the absence of the nipple and mammary gland. This rare genetic condition is highly heterogeneous in presentation and inheritance.

Anterior thoracic hypoplasia is defined by unilateral sinking of the anterior chest wall, hypoplasia of the breast, superior location of the nipple-areola complex, and normal pectoralis muscles. In hypoplasia of the breast, both nipples are present. In Poland syndrome, the defining feature is partial or complete aplasia of the sternocostal head of the pectoralis muscle and deformity of the breast and upper extremity. Tubular breast deformity involves constriction at the base of the breast, hypoplasia of the breast, and herniation of tissue into the nipple-areola complex.

153
Q

A 13-year-old girl is brought to the office by her parents because her left breast is not developing. On examination, both nipples and areolae are present. The breasts are asymmetric; the left breast is considerably smaller. The left anterior axillary fold is absent. This patient is most likely to have which of the following additional developmental differences?

(A) Ambiguous genitalia
(B) Craniosynostosis
(C) Microtia
(D) Pectus excavatum
(E) Syndactyly

A

The correct response is Option E.

This patient has Poland syndrome, which is characterized by unilateral breast or nipple hypoplasia, unilateral absence of the sternal head of the pectoralis major muscle, absence of the pectoralis minor muscle, and ipsilateral syndactyly or hypoplasia of the ipsilateral extremity. In severe Poland syndrome, rib anomalies also occur.

Ambiguous genitalia, craniosynostosis, microtia, and pectus excavatum are not associated with Poland syndrome.

154
Q

Six hours after undergoing breast reconstruction with a free TRAM flap, a 35-year-old woman has swelling of the reconstructed area and bluish discoloration of the skin. A photograph is shown above. Capillary refill time is increased. Findings on Doppler ultrasonography of the flap are within normal limits.

Which of the following is the most appropriate next step in management?

(A) Fluid resuscitation
(B) Anticoagulant therapy
(C) Release of sutures and observation
(D) Operative exploration

A

The correct response is Option D.
This 35-year-old woman who underwent breast reconstruction now exhibits signs of venous outflow obstruction, including increased capillary refill time, dark or bluish discoloration of the skin, and swelling of the affected area. In addition, complete venous obstruction does not always manifest as abnormal findings on Doppler ultrasonography, and the Doppler signal may indeed fall within normal limits in many affected patients. Therefore, in the presence of these clinical signs, the only appropriate option is to perform repeat operative exploration.

Fluid resuscitation and anticoagulant therapy are not appropriate treatment of venous obstruction in patients who have undergone free tissue transfer. In the same way, release of the sutures will not resolve the obstruction.

155
Q

A 56-year-old woman with recurrent cancer of the right breast and no evidence of distant metastatic disease is scheduled to undergo completion mastectomy five years after undergoing lumpectomy (segmental mastectomy) for stage II carcinoma with radiation. In this patient, risk of complications is highest with which of the following methods of immediate breast reconstruction?

(A) Free superior gluteal artery perforator (SGAP) flap
(B) Free TRAM flap
(C) Latissimus dorsi myocutaneous flap with implant
(D) Pedicled TRAM flap
(E) Tissue expansion

A

The correct response is Option E.
In a patient with a previously irradiated breast, reconstruction with a tissue expander followed by a permanent implant (expander/implant) has a higher rate of complications than does reconstruction with any autologous tissue.

The latissimus dorsi myocutaneous flap, pedicled TRAM flap, free TRAM flap, and free SGAP flap are autologous methods of breast reconstruction that bring in new unradiated tissue to reconstruct the breast.

One study showed that previous radiation exposure was a significant risk factor for major complications after breast reconstruction with an expander/implant but not after reconstruction with a TRAM flap. Another study showed that breast cancer patients who underwent radiation and reconstruction with an expander/implant had a significantly higher rate of complications than patients who had radiation and breast reconstruction with a TRAM flap. These findings remained the same whether radiation exposure occurred before or after breast reconstruction.

156
Q

Two years after undergoing modified radical mastectomy on the right, a 36-year-old woman desires reconstruction with an implant. Physical examination shows minimal bulk in the inferior and lateral portions of the right pectoralis major muscle.

The most likely cause is denervation of which of the following nerves during mastectomy?

(A) Fourth intercostal
(B) Lateral pectoral
(C) Long thoracic
(D) Medial pectoral
(E) Thoracodorsal

A

The correct response is Option D.

The minimal bulk in the inferior and lateral portions of the right pectoralis major muscle in this patient most likely results from denervation of the medial pectoral nerve, which is injured often during mastectomy. The pectoralis major is innervated by the medial and lateral pectoral nerves, named for their respective cords of origin from the brachial plexus. The lateral nerve, which arises from C5-6, actually supplies the medial portion of the pectoralis major muscle, and the medial nerve, derived from C8-T1, supplies the lower and lateral sternal portion of the pectoralis major muscle and the pectoralis minor muscle.

Because injury to the medial pectoral nerve results in severe atrophy of the lower half of the muscle, implant coverage will be sparse in this region. It is important for the plastic surgeon to assess the status of the pectoralis muscle before attempting implant augmentation; this can be accomplished by having the patient place her hands on her hips and contract her chest muscles.

The fourth intercostal nerve supplies sensation to the nipple-areola complex. The long thoracic nerve, commonly referred to as the long thoracic nerve of Bell, supplies motor innervation to the serratus muscle, and injury may result in winging of the scapula. The thoracodorsal nerve provides innervation to the latissimus dorsi muscle.

157
Q

Which of the following is the primary advantage of performing breast reconstruction with an extended latissimus dorsi myocutaneous flap instead of the standard latissimus dorsi myocutaneous flap?

(A) Decreased donor site scarring
(B) Decreased incidence of seroma at the donor site
(C) Decreased need for adjuvant breast implantation
(D) Decreased need for division of the humeral tendon attachment
(E) Decreased risk for flap loss

A

The correct response is Option C.

The primary advantage of the extended latissimus dorsi flap is the autogenous tissue it provides, which can be used to enhance breast volume without the need for an implant. The standard latissimus dorsi myocutaneous flap was one of the first created for breast reconstruction. However, with the emergence of the TRAM flap, the latissimus dorsi flap has become a secondary choice for autologous reconstruction for many surgeons because it often does not provide sufficient volume and is, instead, often used in combination with an implant.

Variations of the latissimus dorsi flap have been created to increase volume and obviate the need for an implant. The first extended latissimus dorsi flap, described in 1983, was designed to include fat and fascia from the parascapular and scapular regions and fat from the lumbar spine. This transfer of additional tissue from the back eliminates the need for an implant; therefore, potential complications associated with implants, such as the development of capsular contracture and deformation of the reconstructed breast, are avoided. The technique was later modified by creating a fleur-de-lis skin paddle, which carries additional fat on the surface of the latissimus dorsi muscle and is thus a totally autogenous latissimus dorsi flap.

The contour and scarring of the donor site following flap transfer are acceptable. However, other complications, such as seroma formation and wound necrosis, can be a significant concern. Division of the humeral tendon allows for
increased mobility of the flap. Partial flap and/or fat necrosis are more commonly associated with the extended flap than the standard flap.

158
Q

A 36-year-old woman with carcinoma of the breast is scheduled to undergo modified radical mastectomy followed by postoperative radiation therapy. Which of the following operative techniques is most likely to provide the best aesthetic reconstruction?

(A) Immediate reconstruction with autologous tissue
(B) Immediate reconstruction with autologous tissue and an implant
(C) Immediate reconstruction with an expander and a saline-filled implant
(D) Delayed reconstruction with autologous tissue
(E) Delayed reconstruction with an expander and a silicone gel-filled implant

A

The correct response is Option D.

Delayed reconstruction with autologous tissue will provide the best aesthetic reconstruction in this patient. Transfer of autologous tissue alone is the best method in any patient who has previously undergone radiation therapy. In the absence of autologous reconstruction, a latissimus dorsi flap can be used, especially in patients who are not good candidates for TRAM flap reconstruction or as a salvage technique following periprosthetic contracture in patients who underwent implant reconstruction following radiation therapy.

One recent study evaluated patients who underwent immediate TRAM flap reconstruction followed by radiation therapy and compared them with patients who had radiation therapy first followed by TRAM flap reconstruction. Although the incidence of early complications did not differ significantly between the two groups, late complications were much more common in the immediate reconstruction group than in the delayed reconstruction group. In addition, 28% of those patients undergoing immediate reconstruction experienced severe shrinkage and contraction of the flap and subsequently required an additional procedure for correction. Therefore, in patients who require radiation therapy following mastectomy, it is recommended that reconstruction is delayed until radiation therapy has been completed.

Use of an implant, whether saline- or silicone gel-filled, is associated with a significant increase in capsular contracture and other complications in patients who have previously undergone radiation therapy.

159
Q

In a 30-year-old woman who carries the BRCA-2 genetic mutation, the lifetime risk of development of breast carcinoma is closest to what percentage?

(A) 10%
(B) 20%
(C) 40%
(D) 60%
(E) 100%

A

The correct response is Option D.

In persons who carry the BRCA-1 or BRCA-2 genetic mutation, the lifetime risk for development of breast carcinoma varies from 50% to 85%. Although initial studies showed a lifetime risk of 85%, more recent studies have reported that the risk is actually closer to 56%. Genetic testing for BRCA mutations is now available, and patients who carry a mutation may wish to undergo bilateral mastectomy and reconstruction.

The BRCA genes are autosomal dominant genes whose loci code for proteins; these proteins facilitate DNA repair and thus suppress tumor growth. Although only 5% to 10% of all cancerous tumors have been shown to result from the BRCA genes, a mutation of either gene results in a marked increase in the incidence of carcinoma.

BRCA-1, whose locus lies on chromosome 17, has also been linked to carcinoma of the ovaries, prostate, and colon. More than 100 different mutations of this gene have been reported. The BRCA-2 locus is located on chromosome 13. This mutation is associated with an increased risk for carcinoma of the breast, but not with ovarian carcinoma. According to the results of recent studies, administration of tamoxifen appears to decrease the risk for breast carcinoma in patients with the BRCA-2 mutation.

160
Q

In a 48-year-old woman who recently underwent bilateral reduction mammaplasty, histologic evaluation of resected tissue shows findings consistent with invasive ductal carcinoma. Which of the following factors will best determine the most appropriate next step in the management of this patient?

(A) Age of the patient
(B) Initial tumor margins
(C) Location of the tumor
(D) Total volume of tissue resected
(E) Tumor size

A

The correct response is Option B.
In this patient who has invasive ductal carcinoma, the initial tumor margins will determine the most appropriate next step in management. If the tumor was not completely excised during the original reduction mammaplasty procedure, completion mastectomy is recommended. Because of the potential for shifting of the tissues following reduction, possible tumor seeding, and residual lymphatic spread, this procedure is advocated for nearly all patients who did not undergo complete tumor excision at the time of breast reduction. Even if the tumor is excised completely with adequate margins, the axillary region should be evaluated because chemotherapy and radiation therapy may still be required.

Occult breast carcinoma has been identified in as many as 2% of women undergoing reduction mammaplasty. In addition, occult carcinoma was found in 4.6% of women who underwent a “balancing reduction” following mastectomy. There is some discrepancy in the incidence of occult carcinoma related to the inclusion or exclusion of in situ lesions. Women who have occult malignancies identified during breast reduction are more likely to be younger and to have lobular tumors without palpable lymph nodes.

The age of the patient, total volume of tissue resected, location of the tumor, and tumor size have not been shown to influence treatment options independent of the surgical margins obtained at the time of reduction mammaplasty.

161
Q

A 45-year-old woman is scheduled to undergo mastectomy of the right breast followed by reconstruction using a free TRAM flap. She has a 15 pack/year history of cigarette smoking. This patient is at increased risk for development of each of the following postoperative complications EXCEPT

(A) abdominal flap necrosis
(B) fat necrosis
(C) hernia
(D) mastectomy skin flap necrosis

A

The correct response is Option B.

The free TRAM flap is frequently advocated for breast reconstruction in high-risk patients, including those who smoke, because of its enhanced blood supply; however, patients who smoke are still at increased risk for development of complications. One large retrospective study showed that patients who smoked were at greater risk for developing hernia and necrosis of the mastectomy skin flap and abdominal flap when compared with nonsmokers undergoing breast reconstruction with the free TRAM flap. Because patients who had a 10 pack/year or greater history of smoking were at greatest risk for perioperative complications, it has been suggested that reconstruction should be delayed until the patient has stopped smoking for at least four weeks. Studies have shown no significant increase in the rate of fat necrosis, flap loss, or vessel thrombosis in patients who smoked when compared with nonsmokers.

162
Q

A 36-year-old woman desires breast reconstruction one year after undergoing right modified radical mastectomy followed by radiation therapy. She is 5 ft 4 in tall, weighs 135 lb, and is otherwise healthy. The left cup size of her bra is 32B.

Which of the following is the most appropriate reconstructive option in this patient?

(A) Reconstruction with a latissimus dorsi flap and a saline-filled implant
(B) Reconstruction with a TRAM flap
(C) One-stage reconstruction with a silicone gel-filled implant
(D) Two-stage reconstruction with a tissue expander and a saline-filled implant
(E) Two-stage reconstruction with a tissue expander and a silicone gel-filled implant

A

The correct response is Option B.

Reconstruction with a TRAM flap is the most appropriate choice for this 36-year-old woman. Transfer of autologous tissue alone is the best method in any patient who has previously undergone radiation therapy. Use of an implant, whether saline- or silicone gel-filled, is associated with a significant increase in capsular contracture and other complications in previously irradiated patients. The latissimus dorsi flap is reserved for those patients who are not good candidates for TRAM flap reconstruction or as a salvage technique following periprosthetic contracture in patients who underwent implant reconstruction following radiation therapy.

163
Q

A 55-year-old woman is scheduled to undergo right mastectomy for management of breast carcinoma. She has a 30 pack/year history of cigarette smoking. She would like to undergo breast reconstruction using a free TRAM flap at the time of the mastectomy procedure.

This patient would be at significantly increased risk for development of which of the following complications?

(A) Anastomotic thrombosis
(B) Fat necrosis
(C) Mastectomy skin flap necrosis
(D) Partial TRAM flap loss
(E) Wound infection

A

The correct response is Option C.

When compared with nonsmokers, patients who smoke are at increased risk for necrosis of the mastectomy skin flap or abdominal flap and hernia development after breast reconstruction using a TRAM flap. Patients who have a smoking history of greater than 10 pack/years are at particularly high risk for the development of these complications. However, the risk can be decreased significantly if a delay procedure is performed or if the patient stops smoking at least four weeks before the procedure.

TRAM flap reconstruction is not associated with an increased risk for vessel thrombosis, fat necrosis, TRAM flap loss, or wound infection in patients who smoke.

164
Q
A